SlideShare a Scribd company logo
Yuli Kusumawati, S.T., M.T.
CATATAN KULIAH
EKONOMI MINERAL
CATATAN KULIAH
EKONOMI MINERAL
YULI KUSUMAWATI, S.T., M.T.
Allah, tidakada Tuhan selain Dia.
YangMaha Hidup, yangterus-menerus mengurus (makhluk-Nya),
tidakmengantukdan tidaktidur.
Milik-Nya apa yangada di langit dan apa yangada di bumi.
Tidakada yangmemberi syafaat di sisi-Nya tanpa izin-Nya.
Dia mengetahui apa yangada di hadapan mereka dan apa yangada di belakangmereka
dan mereka tidakmengetahui sesuatu apapun tentangilmu-Nya melainkan apa yangDia kehendaki.
Kursi-Nya meliputi langit dan bumi. Dan Dia tidakmerasa berat memelihara keduanya,
dan Dia Maha Tinggi, Maha Besar.
(QS. Al Baqarah : 255)
iii
DAFTAR ISI
1. DEFINISI DAN KONSEP DASAR 1
1.1. Ekonomi teknik dan pengambilankeputusaninvestasi 1
1.2. Karakteristikindustri pertambangan 2
1.3. Konsep nilai waktu uang 3
1.4. Konsep kesetaraan 4
1.5. Aliran kas 5
2. KONSEP BIAYA 6
2.1. Pengertian biaya 6
2.2. Klasifikasi biaya 6
2.2.1. Biaya berdasarkanwaktu 6
2.2.2. Biaya berdasarkansifat penggunaannya 6
2.2.3. Biaya berdasarkanproduk 7
2.2.4. Biaya berdasarkanvolume produksi 8
2.3. Biaya dalam usahapertambangan 9
2.3.1. Biaya kapital 9
2.3.2. Biaya operasi 10
3. KONSEP DAN RUMUSAN BUNGA 12
3.1. Bunga sederhana dan bungamajemuk 12
3.2. Suku bunga nominal dan suku bungaefektif 13
3.3. Minimum Attractive RateofReturn (MARR) 13
3.4. Waktu lipat dua 14
3.5. Rumus bunga majemuk 14
3.5.1. Single payment compound-amount 14
3.5.2. Single payment present-worth 15
3.5.3. Uniform series compound-amount 15
3.5.4. Sinking-fund deposit 15
3.5.5. Capital-recovery 16
3.5.6. Uniform series present-worth 16
3.5.7. Uniform gradient-series 16
4. INDIKATOR KELAYAKAN INVESTASI 18
4.1. Net Present Value (NPV) 18
4.2. Internal Rate ofReturn(ROR) 19
4.3. Benefit-Cost Ratio (BCR) 19
4.4. Present Value Ratio (PVR) 20
4.5. PaybackPeriod (PBP) 20
5. ANALISIS MUTUALLY EXCLUSIVE 22
5.1. Analisis mutually exclusive dengan umur alternatifsama 22
5.2. Analisis mutually exclusive dengan umur alternatifberbeda 24
6. ANALISIS NON MUTUALLY EXCLUSIVE 27
6.1. Analisis non-mutually exclusivedenganumur alternatifsama 27
6.2. Analisis non-mutually exclusivedenganumur alternatifberbeda 29
7. DEPRESIASI,DEPLESI, AMORTISASI 32
7.1. Depresiasi 32
7.1.1. Straigth line depreciation (SLD) 33
iv
7.1.2. Double decliningbalance depreciation (DDBD) 34
7.1.3. Sum of years digits depreciation (SOYD) 35
7.1.4. Sinkingfund depreciation (SFD) 36
7.1.5. Unit ofproduction depreciation (UPD) 38
7.2. Deplesi 39
7.2.1. Cost depletion 40
7.2.2. Percentage depletion 40
7.2.3. Pemilihan metode deplesi 41
7.3. Amortisasi 41
8. PENGARUH PAJAK DALAM ANALISIS LIFE CYCLE COSTING 43
8.1. Pengertian pajak 43
8.2. Cashflow setelah pajak 44
9. ANALISIS PENGGANTIAN 53
9.1. Konsep penggantian 53
9.2. Umur ekonomis 53
9.3. Membandingkansewaatau beli 54
10. ANALISIS RESIKO DAN KETIDAKPASTIAN 59
10.1. Analisis sensitiv itas 59
10.1.1. Analisis sensitiv itas variabel tunggal 60
10.1.2. Analisis sensitiv itas dengan pendekatan rentang 62
10.2. Analisis probabilitas 62
10.2.1. Teori probabilitas 62
10.2.2. Pohon keputusan 63
v
PENGANTAR
Alhamdulillahirabbil’alamiin, berkat kuasa dan kasih sayang ALLAH SWT akhirnya Catatan Kuliah Ekonomi Mineral ini
bisa diselesaikan.
Catatan Kuliah Ekonomi Mineral ini diperuntukkan bagi mahasiswa teknik tingkat diploma maupun sarjana yang
ingin mendapatkan pemahaman mengenai prinsip-prinsip dasar dan metodologi ekonomi teknik dalam proses
pengambilan keputusan investasi pada kegiatan perencanaan dan pengembangan proyek-proyek keteknikan,
khususnya pada industri pertambangan dan pengelolaan sumberdayamineral.
Materi dalam catatan kuliah ini merupakan rangkuman dari berbagai referensi yang disajikan secara ringkas dan
mudah dipahami namun tetap mencakup esensi dari setiap teori yang berkaitan dengan ekonomi teknik.
Disampingitu disajikan pula contoh soal sederhanauntukmembantu pemahaman materi.
Mengingat keterbatasan yang ada, maka catatan kuliah ini masih banyak kekurangan dan perlu perbaikan. Oleh
karena itu masukan dan saran sangat diharapkan untuk penyempurnannya. Tak lupa penyusun menghaturkan
terima kasih kepada semua pihak yang telah membantu dalam menyelesaikan catatan kuliah ini. Selain itu
penghargaan yang besar penyusun berikan kepada suami dan anak-anak yang tidak surut atas memberikan doa,
semangat, dan dukungan.
Mudah-mudahan catatan kuliah ini menjadi amal kebaikan penyusun dan dapat memberikan manfaat yang
sebesar-besarnya.
Bandung, Juli 2014
Yuli Kusumawati S.T., M.T.
Yuli Kusumawati, Catatan Kuliah Ekonomi Mineral - 1
1. DEFINISI DAN KONSEP DASAR
1.1. Ekonomi teknik dan pengambilan keputusan investasi
Ekonomi teknik adalah ilmu yang berkaitan dengan teknik analisis kuantitatif yang berguna untuk memilih satu
alternatifyanglebih baikdari beberapaalternatifyangsecarateknis layak.
Prosedur dalam analisis ekonomi teknikadalah:
1. Pengenalan masalah, perumusan, dan evaluasi.
2. Merancangalternatifyanglayak.
3. Menyusun net cashflow dari masing-masingalternatif.
4. Pemilihan kriteria.
5. Menganalisis danmembandingkan semuaalternatif.
6. Memilih alternatifyangtepat.
7. Memonitor kinerja dan mengevaluasi hasilnya.
Aplikasi prinsip-prinsip ekonomi teknik tidak hanya berguna dalam analisis kelayakan dari aspek finansial
terhadap proyek-proyek keteknikan tetapi juga membantu dalam pengambilan keputusan investasi pada
umumnya.
Investasi merupakan penukaran sejumlah dana dengan kemungkinan perolehan 100% (karena telah dikuasai)
dengan jumlah dana yang lebih besar tetapi kemungkinan perolehannya kurang dari 100%. Investasi diperlukan
untuk memulai suatu usaha atau mempertahankan dan meningkatkan kapasitas produksi suatu usaha yang
sedangberjalan.
Inv estasi memiliki dua faktor, yaitu:
1. Waktu
2. Resiko
Tujuan dilakukannya investasi adalah untuk memperoleh nilai lebih/keuntungan di masa depan dari modal yang
diinvestasikan. Modal yang dimaksud dapat berupa uang, barang modal, tanah, bangunan, teknologi,
ataupun sesuatu yang tidak riil, misalnya hak paten atau kemampuan manajerial. Dalam bidang
pertambangan, kapital umumnyaberupadeposit bahan tambangdan modal.
Keputusan investasi modal akan mempunyai dampak jangka pendek dan jangka panjang bagi kelangsungan
perusahaan untuk dapat berkompetisi ataupun untuk tetap berproduksi. Keputusan investasi yang salah tidak
saja dapat mengurangi keuntungan perusahaan tetapi juga dapat menghentikan kegiatan perusahaan sama
sekali.
Keputusaninvestasi modal mempunyai duabentuktindakan utama, yaitu :
1. Mengalokasikan sejumlah modal untukinvestasi proyektertentu atauuntukmenyediakan asset produksi.
Tujuan:
 Memahami konsep ekonomi teknikdan aplikasinyadalam pengambilan keputusan investasi.
 Memahami karakteristikindustri pertambangan.
 Memahami konsep nilai waktu uangdankesetaraan.
 Memahami konsep aliran kas.
Yuli Kusumawati, Catatan Kuliah Ekonomi Mineral - 2
2. Memperoleh sejumlah modal untukmeningkatkannilai perusahaan.
Ada lima langkah pentingdalam pengambilan keputusan investasi, yaitu :
1. Mendefinisikan masalah
2. Menganalisis masalah
3. Mengembangkan alternatifsolusi
4. Memutuskan solusi yangterbaik
5. Mengubah keputusanmenjadi tindakanyangefektif
Analisis inv estasi diklasifikasikan menjadi tiga, yaitu :
1. Analisis ekonomi
Evaluasi terhadap kemakmuran relatifdari situasi-situasi investasi dari sudut pandang laba dan ongkos, yaitu
studi ev aluasi terhadapkeuntungandari alternatifproyek.
2. Analisis finansial
Analisis finansial adalah evaluasi terhadap cara pendanaan terhadap investasi yang diusulkan. Terdapat
beberapa alternatif metode untuk pendanaan, di antaranya adalah dengan dana pribadi atau
perusahaan, meminjam dari bank, atau menawarkansaham padapublik.
Analisis finansial terbagi lagi menjadi tiga pengambilan keputusan penting yang harus dilakukan oleh
sebuah perusahaan,yaitu :
a. Keputusandeviden
b. Keputusankeuangan
c. Keputusaninvestasi
Ketiga keputusan ini salingterkait satu sama lain dan mempengaruhi analisis inv estasi secara keseluruhan.
3. Analisis intangible
Evaluasi terhadap faktor-faktor yang mempengaruhi investasi tetapi sukar diukur secara kuantitatif. Contoh
faktor intangible antara lain perijinan, pertimbangan keamanan, opini publik, pertimbangan politik, faktor
ekologi dan lingkungan, danketidakpastiankondisi peraturan pajak.
Ketiga jenis analisis investasi ini mutlak harus dilakukan, karena sering terjadi suatu alternatif yang hasil analisis
ekonominya baik, ternyata tidak cukup baik jika dilihat dari analisis finansial dan intangible-nya. Sebagai
contoh, suatu proyek ditolak dari analisis finansial bila dana internalnya tidak mencukupi untuk membiayai
proyek dan juga tidak dapat memperoleh pinjaman dana dari luar dengan tingkat bunga yang sesuai. Faktor
intangible yang dapat mengakibatkan suatu proyek ditolak secara analisis ekonomi pada umumnya adalah
karena opini publik dan masalah lingkungan (polusi udara, tanah, dan air). Karenanya pengaruh analisis
finansial dan intangible terhadap analisis ekonomi harus diperhatikan secara seksama karena analisis-analisis
tersebut mempunyai pengaruh yangsangat besar terhadap pemilihan alternatifinvestasi.
1.2. Karakteristik industri pertambangan
Industri pertambangan merupakan bagian dari dunia industri umum namun industri pertambangan mempunyai
karakteristik khusus yang berbeda dengan industri lainnya. Pemahaman tentang karakteristik khusus ini penting
untukmelakukan analisis kelayakansuatuproyek/investasi tambang.
KarakteristikIndustri pertambangan antara lain:
1. Modal Besar
Besarnya modal yang dibutuhkan untuk industri tambang bervariasi, tergantung dari jenis bahan tambang,
metode penambangan, skala penambangan, lokasi dan parameter lainnya.
Yuli Kusumawati, Catatan Kuliah Ekonomi Mineral - 3
2. Periode Pra Produksi yangPanjang
Lama periode pra produksi tergantungdari metode penambangan, metode pengolahan, ukuran dan letak
deposit, kompleksitas operasi, dan kendala lingkungan. Periode pra produksi ini berkisar antara 3 – 12 tahun.
Periode pra produksi yang panjang akan berdampak terhadap besar modal yang dibutuhkan dan
terhadaptingkat pengembalian modal.
3. Beresiko Tinggi
Disamping resiko yang berhubungan dengan kebutuhan modal yang besar serta masa pra produksi yang
lama, terdapat resiko lain yang mempengaruhi keputusan investasi pada industri tambang, yaitu: resiko
geologi, resiko engineeringdan konstruksi, reiko ekonomi, resiko politik, dan resiko pasar mineral.
4. Sumber daya alam yangtakdapat diperbaharui (nonrenewable resources)
Implikasi dari sumber daya alam yang tidak dapat diperbaharui ini adalahterhadap pendapatan utama
perusahaan yang diperoleh dari penjualan bahan tambang, yang mengakibatkan umur tambang
tergantung dari jumlah cadangan dan tingkat produksi sehingga dibutuhkan eksplorasi kontinyu untuk
menemukan deposit baru.
5. Mendorongpertumbuhan ekonomi
Dikarenakan letak aktivitas penambangan banyak terdapat di daerah terpencil, hal ini akan dapat
memberikan dampak positif terhadap aktivitas ekonomi masyarakat setempat sehingga dapat mendorong
pertumbuhan ekonomi di daerahtersebut.
6. Dampakterhadap lingkungan
Kegiatan eksploitasi bahan tambangakan mengubah bentangalam sehingga berdampak buruk terhadap
keadaan lingkungan. Oleh karena itu tingkat kepedulian industri tambang terhadap lingkungan harus
tinggi. Reklamasi merupakan salah satu upaya untuk mengurangi dampak lingkungan dari kegiatan
penambangandanpengolahan.
7. Sifat indestructibility ofproduct
Konsekuensi dari sifat ini adalah munculnya pasar sekunder dan dapat mengurangi prosentase kebutuhan
akan bijih/bahan tambang. Daur ulang logam sering dipertimbangkan lebih menguntungkan
dibandingkan menambangbijih untukdijadikan logam.
1.3. Konsep nilai waktu uang
Konsep nilai waktu uang (time value of money) mengacu pada biaya kesempatan (opportunity cost) dalam
menghasilkan pendapatan pada saat sekarang. Bila sejumlah dana tidak digunakan untuk membiayai suatu
proyek, maka dana tersebut akan dapat digunakan untuk membiayai proyek lain. Dengan kata lain jika suatu
dana ditanamkan seluruhnya pada suatu proyek maka akan menghilangkan kesempatan untuk mendapatkan
penghasilan (return) dari proyeklain yangtidakdipilih.
Nilai sesuatu (termasuk uang) berubah seiring dengan berjalannya waktu. Perjalanan waktu pada umumnya
mengurangi nilai kecuali pada benda antik dan benda yang mempunyai umur layanan (service life) tak
terbatas, seperti tanah.
Beberapa hal yangmenyebabkan nilai uangdipengaruhi oleh waktu adalah:
1. Uang memiliki purchasingpower (dayabeli).
Secara umum uang digunakan untuk melakukan transaksi, sehingga semua komoditi harus dinilai
berdasarkan nilai tukarnyaterhadapuang.
Daya beli uangmenurun disebabkan oleh:
 Inflasi
 Perubahan polasupply(pasokan) dandemand (permintaan)
Yuli Kusumawati, Catatan Kuliah Ekonomi Mineral - 4
 Perubahan struktur ekonomi
Sebagai contoh harga premium sekarang Rp 6.500/liter, artinya 1 liter premium nilainya Rp 6.500.Beberapa
tahun yang lalu harga premium masih Rp 4.500/liter, atau dikatakan nilai uang mengalami penurunan
terhadappremium.
2. Uangmemiliki earningpower (kemampuan menghasilkan).
Sejumlah uang yang disimpan di bank akan bertambah dengan adanya bunga. Sebagaimana meminjam
barang, bunga(interest) dianggapsebagai sewaatas pemakaianuangtersebut.
1.4. Konsep kesetaraan
Konsep kesetaraan (equivalence) menyatakan bahwa sejumlah uang pada satu waktu tertentu setara nilainya
dengan uang dalam jumlah yangberbeda pada waktu yang berbeda apabila diberikan discount rate tertentu.
Konsep kesetaraan dapat digunakan untuk mengevaluasi rencana investasi. Untuk keperluan tersebut maka
seluruh pendapatan dan pengeluaran proyek di masa yang akan datang dihitung nilainya pada saat sekarang
(present worth).
Sebagai contoh uang sejumlah Rp 10 juta yang disimpan di bank dengan bunga 20%/tahun dapat diambil
dalam beberapacara, antaralain:
a. Disimpan hari ini dan baru diambil tahun depan dengan jumlah Rp 12 juta, dengan rincian pokok simpanan
Rp 10 juta ditambahbungaRp 2 juta.
b. Diambil bunganya saja Rp 2 juta tiap tahun selama lima tahun, dan pada akhir tahun kelima pokok
simpanannya diambil. Jumlah uang yang diperoleh selama lima tahun tersebut adalah Rp 20 juta, dengan
rincian bunga Rp 2 juta x 5 = Rp 10 juta dan pokoksimpanan Rp 10 juta.
c. Diambil dalam jumlah yang sama setiap tahunnya selama sepuluh tahun sebesar Rp 2,385 juta. Jumlah
uangyangdiperoleh selama sepuluhtahun tersebut adalah Rp23,85 juta, yaituRp 2,385 juta x 10.
Ketiga jumlah nominal uang tersebut yaitu Rp 12 juta, Rp 20 juta, dan Rp 23,85 juta pada dasarnya setara
dengan Rp 10 juta padasaatsekarang. Kesetaraantersebut terjadi dengan penggunaanbunga 20% per tahun.
1.5. Aliran kas
Aliran kas (cashflow) adalah aliran uang yang terdiri atas cash-in (uang masuk) dan cash-out (uang keluar) yang
terjadi selama periode operasi tertentu, misalnya bulan atau tahun. Cashflow digunakan untuk
menggambarkan pergerakanuangsuatu perusahaanpada satuan periode tertentu.
Aliran kas terdiri dari:
 Uangmasuk(cash in), umumnya berasal dari penjualan produkatau manfaat(benefit) yangterukur.
 Uangkeluar (cash out), merupakankumulatifdari biaya-biaya(cost) yangdikeluarkan.
Net cashflow adalah jumlah uang masuk dikurangi jumlah uang keluar. Net cashflow yang negatif artinya defisit,
yaitu cash-out lebih besar daripada cash-in.
Discounted cashflow adalah nilai cashflow yang dibawa ke nilai sekarang (present worth) sesuai dengan
discount rate yang diberikan. Istilah discount mengacu pada pengurangan nilai, yaitu jika sejumlah tertentu
uangdimasa datangdihitungnilainyasekarangmaka nilai uangtersebut akanlebih kecil (berkurang).
Cashflow dapat digambarkan dalam bentuk:
1. Tabel yangterdiri palingsedikit tiga kolom, yaitu kolom waktu, kolom pengeluaran, dankolom pendapatan.
Yuli Kusumawati, Catatan Kuliah Ekonomi Mineral - 5
2. Diagram yang ditunjukkan dengan anak panah yang mengarah ke atas untuk cash-in dan anak panah
yangmengarah ke bawahuntukmenggambarkancash-out.
Contoh 1-1:
Pada tahun awal (tahun ke-0) biaya yang dikeluarkan adalah Rp 10 juta. Tahun ke-1 dikeluarkan lagi biaya Rp 2
juta selain diterima pendapatan (revenue) Rp 3 juta. Pada tahun ke-2, 3, 4, dan 5 diperoleh pendapatan
masing-masing Rp 5 juta. Gambarkan undiscounted cashflow perusahaan tersebut dalam bentuk tabel dan
diagram.
Jawab:
Tabel undiscountedcashflow:
Tahun Pendapatan(cash-inflow) Pengeluaran (cash-outflow) Net cashflow CummulativeCashflow
0 0 10.000.000 -10.000.000 -10.000.000
1 3.000.000 2.000.000 1.000.000 -9.000.000
2 5.000.000 0 5.000.000 -4.000.000
3 5.000.000 0 5.000.000 1.000.000
4 5.000.000 0 5.000.000 6.000.000
5 5.000.000 0 5.000.000 11.000.000
Jumlah 23.000.000 12.000.000 11.000.000
Diagram cashflow:
Dari tabel di atas terlihat pada akhir tahun ke-5, proyek tersebut menghasilkan keuntungan bersih sebesar Rp
11.000.000.
Secara umum cash flow dikelompokkan menjadi:
1. Cashflow uniform, adalah cashflow dimana besar cash-in dan cash-out tiap periode adalah seragam
(tetap).
2. Cashflow gradient uniform, adalah cashflow dimana besar cash-in atau cash-out bertambah atau
berkurangdengan gradien tetap.
3. Cashflow seri geometrik, adalah cashflow dimana besar cash-in atau cash-out bertambah atau berkurang
dengan presentase tetap.
10 2 3 4 5
10.000.0000 2.000.000
3.000.000 5.000.000 5.000.000 5.000.000 5.000.000
Yuli Kusumawati, Catatan Kuliah Ekonomi Mineral - 6
2. KONSEP BIAYA
2.1. Pengertian biaya
Biaya adalah jumlah uang yang harus dikeluarkan untuk memproduksi sesuatu (cost of production) atau harga
yangharus dikeluarkan untukmendapatkansesuatu(supply price).
Konsep biaya dalam analisis ekonomi teknik perlu dipahami karena semua analisis yang dilakukan didasarkan
atas perkiraan arus kas yang dikembangkan berdasarkan perkiraan biaya (cost estimate) dan proyeksi
pendapatan(revenue projection).
2.2. Klasifikasi biaya
Biaya dapatkelompokkan berdasarkanklasifikasi berikut (Giatman, Ekonomi Teknik):
a. Berdasarkan waktunya, dibedakanmenjadi:
 Biaya masa lalu
 Biaya perkiraan
 Biaya sebenarnya
b. Berdasarkan sifat penggunaannya, dibedakanmenjadi:
 Biaya inv estasi
 Biaya operasi
 Biaya perawatan
c. Berdasarkan produknya, dibedakanmenjadi:
 Biaya produksi/pabrikasi
 Biaya komersial
d. Berdasarkan v olume produk, dibedakanmenjadi:
 Biaya tetap
 Biaya tidaktetap
 Biaya semi v ariable
2.2.1. Biaya berdasarkan waktu
Biaya berdasarkanwaktu dibedakanmenjadi:
1) Biaya masa lalu (hystorical cost), yaitu biaya yang secara riil telah dikeluarkan yang dibuktikan dengan
catatan historis pengeluarankegiatan.
Biaya historis digunakan sebagai dasar untuk:
 Penyusunan/perkiraan biayamasa datang.
 Pertanggungjawabanatau audit terhadapbiaya-biayayangtelah dikeluarkan.
2) Biaya perkiraan (predictive cost), yaitu perkiraan biaya yang akan dikeluarkan bila suatu kegiatan
dilaksanakan.
Biaya perkiraan digunakan untuk:
 Memperkirakan pemakaianbiayadalam merealisasikan suatukegiatan.
 Menganalisis efektivitas dan efisiensi biaya terhadapkegiatanyangakandilaksanakan.
Tujuan:
 Memahami konsep biayadan klasifikasinya secara umum.
 Mampu menghitungbiaya produksi denganmetode yangsesuai.
 Mampu menganalisis biayaproduksi sehingga bisa memperbaiki efisiensinya.
 Mengetahui komponen biaya yangadadalam usaha pertambangan.
Yuli Kusumawati, Catatan Kuliah Ekonomi Mineral - 7
3) Biaya aktual (actual cost), yaitu biaya yang sebenarnya dikeluarkan. Biaya ini perlu diperhitungkan jika
rentang waktu antara pembelian dengan proses produksi atau penjualan cukup lama sehingga
terpengaruh dengan perubahan harga pasar.
Biaya aktual dibedakanmenjadi:
 Biaya riil (expense), yaitu biaya yangdikeluarkansecara riil.
 Biaya semu (sunk cost), yaitu biaya yang ditanggung tetapi tidak pernah dikeluarkan secara riil. Misalnya,
selisih harga pembukuanaset yangakandilikuidasi denganharga pasar.
 Biaya kesempatan (opportunity cost), yaitu biaya yang ditanggung akibat kelalaian dalam
memanfaatkanpeluanguntukmendapatkan keuntungan yanglebih besar.
Metode perhitunganbiayaaktual antaralain:
 First in first out (FIFO)
 Last in first out (LIFO)
 Metode rata-rata(average method)
 Metode harga standar (standard price method)
2.2.2. Biaya berdasarkan sifat penggunaannya
Biaya berdasarkansifat penggunaannyadikelompokkan menjadi tiga, yaitu:
1) Biaya investasi (investment cost), yaitu biaya yang ditanamkan untuk membangun dan mengembangkan
suatu usaha sehingga siap beroperasi. Biaya ini bisanya dikeluarkan pada awal kegiatan dengan jumlah
yangrelatifbesar dan mempunyai dampakjangka panjanguntukkeberlangsunganusahatersebut.
Contoh:
 Pembelian tanah atauhakpenggunaan lahan.
 Pembangunanpabrik, gedung, dan infrastuktur pendukung.
 Pengadaan mesin danperalatan produksi.
 Pengadaan alat transportasi
 Pengadaan perabotan kantor dan sarana pendukungadministrasi.
 Pendidikan dan pelatihan sumberdayamanusia.
2) Biaya operasi (operational cost), yaitu biaya yang dikeluarkan untuk menjalankan kegiatan usaha. Biaya ini
biasanya dikeluarkan secara rutin atau berkala dalam jumlah yang relatif sama sesuai dengan jadwal
kegiatan/produksi.
Biaya operasi digunakan antara lain untuk:
 Pembelian bahanbaku
 Pembelian bahanpendukung
 Pembayaran upahkaryawan
 Pengeluaran untukkeperluanorganisasi danadministrasi.
3) Biaya perawatan (maintenance cost), yaitu biaya yang digunakan untuk menjamin kinerja fasilitas atau
peralatan agar selalu prima dan siapuntukdigunakan.
Sifat pengeluaranini umumnya dibedakan menjadi dua, yaitu:
a. Biaya perawatan rutin (preventive maintenance)
b. Biaya perawatan mendadak(curative maintenance)
2.2.3. Biaya berdasarkan produk
Biaya berdasarkanproduk dikelompokkan menjadi dua, yaitu:
1) Biaya produksi (production cost) atau biaya pabrikasi (factory cost), yaitu biaya yang langsung berkaitan
dengan proses produksi.
Biaya produksi terdiri dari komponen berikut:
Yuli Kusumawati, Catatan Kuliah Ekonomi Mineral - 8
a. Biaya utama (prime cost), yaitu biaya untuk bahan langsung dan tenaga kerja langsung yang
berhubungan denganproses produksi.
b. Biaya overhead, yaitu biaya untuk bahan tak langsung, tenaga kerja tak langsung, dan biaya tak
langsunglainnya.
c. Biaya konversi (conversion cost), yaitu biaya pengubahan bahan bahan baku menjadi barang jadi.
Biaya bn biaya overheaddapat digabungke dalam kelompokbiaya konversi.
2) Biaya komersial (commercial cost)
Biaya komersial adalah biayadi luar biayaproduksi yangdigunakan untukkeperluanpenjualan produk.
Biaya komersial meliputi:
a. Biaya umum dan administrasi (general and administration cost), contoh gaji karyawan dan pimpinan,
biaya untukkeperluanadministrasi, dantelekomunikasi.
b. Biaya pemasaran(marketingcost), meliputi biayadistribusi, danpromosi.
c. Pajak usaha dan perusahaan (company tax), meliputi semua pajak dan retribusi yang dikeluarkan
perusahaan.
Tujuan perhitunganbiaya berdasarkan produkantaralain:
 Memproyeksikan biayaproduksi danharga jual produk.
 Mengetahui komposisi komponen biaya produksi dan biayakeseluruhanproduk.
 Untuk menganalisis struktur biaya produk yang ideal guna memperbaiki struktur pembiayaan melalui
konsep pusat-pusatpembiayaan.
2.2.4. Biaya berdasarkan volume produksi
Biaya berdasarkanvolume produksi dibedakanmenjadi:
1) Biaya tetap (fixed cost), yaitu biaya yang dikeluarkan dengan jumlah yang relatif sama walaupun volume
produksi berubah dalam batas tertentu. Contoh:gaji karyawan tetap, biaya penyusutan, pajak bumi dan
bangunan, biaya sewa, biayalistrikuntukpenerangan, telepon,dan air.
2) Biaya tidak tetap (variable cost), yaitu biaya yang besarnya berubah secara proporsional dengan jumlah
produk yang dibuat. Contoh: biaya bahan baku, tenaga kerja langsung yang digaji berdasarkan v olume
kerja, biaya penyusutan yangtergantungdenganvolume produksi.
3) Biaya semi variable (semi variable cost), yaitu biaya yang berubah tidak proporsional terhadap volume
produksi. Contoh: perubahan volume produksi yang melampaui kapasitas mesin, sehingga diperlukan biaya
penambahan kapasitas mesin, atau biayaperbaikan mesin.
Contoh 2-1:
PT Insan Kreatif mendapat pesanan bangku untuk suatu sekolah. Di pasaran harga bangku (S) sesuai dengan
spesifikasi teknis yang diajukan sekolah itu adalah Rp 450.000/unit. Jika bangku itu dibuat sendiri maka diperlukan
biaya-biayasebagai berikut:
Harga kayu = Rp 950.000/m3
Paku = Rp 20.000/kg
Bahan finishing = Rp 100.000/lt
Upah tukang = Rp 80.000/orang/hari
Upah pembantutukang = Rp 50.000/orang/hari
Satu unit bangku membutuhkan 0,20 m3 kayu, 0,01kg paku, bahan finishing 0,1lt, dan diselesaikan selama 0,5
hari kerja oleh tukang ditambah1 hari kerja oleh pembantunya.Selain itu perusahaan memerlukan bengkel kerja
dengan biayasekitar Rp 1,5 juta dan pengadaan peralatankerja Rp 2,2 juta.
Yuli Kusumawati, Catatan Kuliah Ekonomi Mineral - 9
Kelompokkan biaya berdasarkan biaya tetap dan biaya variable. Hitunglah berapa jumlah minimal pesanan
bangku yangharus diterima perusahaan untukmencapai titikimpas (breakev enpoint/BEP)?
Jawab:
Biaya tetap:
Membangun bengkel kerja = Rp 1.500.000
Pengadaan alat kerja = Rp 2.200.000
Jumlah fixed cost (FC) = Rp 3.700.000
Biaya v ariable per unit bangku:
Bahan kayu = 0,2 m3 x Rp 950.000/m3 = Rp 190.000
Paku = 0,01kg x Rp 20.000/kg = Rp200
Bahan finishing = 0,1 lt x Rp 100.000/lt = Rp 10.000
Upah tukang = 0,5 hari x Rp 80.000/orang/hari = Rp 40.000
Upah pembantutukang = 1 hari x Rp 50.000/orang/hari = Rp 50.000
Jumlah v ariable cost (VC) = Rp 290.200
Jumlah produksi minimal untukmencapai titikimpas (BEP) adalah:
𝐁𝐄𝐏 =
𝐅𝐂
𝐒 − 𝐕𝐂
BEP =
Rp 3.700.000
Rp 450.000− Rp 290.200
= 23,15 ≈ 24 unit
2.3. Biaya dalam usaha pertambangan
Secara umum biaya dalam usahapertambangan dibedakan menjadi dua, yaitu:
1. Biaya kapital (biayainvestasi), terdiri dari duakomponen yaitu:
1) Biaya kapital tetap
2) Modal kerja
2. Biaya operasi, terdiri dari tiga komonen yaitu:
1) Biaya operasi langsung
2) Biaya operasi taklangsung
3) Biaya ov erhead
2.3.1. Biaya kapital
Biaya kapital (capital cost) adalah jumlah biaya yang dibutuhkan untuk membuat suatu endapan bahan galian
yangberada di dalam bumi menjadi produk tambang yangdapat dijual.
Biaya kapital terdiri dari dua komponen penting, yaitu:
1) Biaya kapital tetap (initial investment)
Jika tambang yang akandikerjakan merupakan tambang baru, maka biaya tetap biasanya terdiri dari
komponen-komponen berikut:
 Land Acquisition (pembebasanlahan), biayanyatergantungkepada luas dan lokasi lahan.
 Konstruksi pra-penambangan(pengupasantanahpenutup, dan sebagainya).
 Pembangunantambang/masa konstruksi
 Analisa DampakLingkungan.
 Peralatan tambang, bangunan, saranalain.
 Peralatan pabrik, bangunan, sarana lain.
 Sarana penunjang(jalan, listrik, perumahan, sarana olahraga, instalasi air, dansebagainya).
 Jasa perancangan dan konsultasi.
Yuli Kusumawati, Catatan Kuliah Ekonomi Mineral - 10
 Contingency.
2) Modal kerja (workingcapital).
Modal kerja adalah biaya yang digunakan untuk memulai produksi sebelum perusahaan mendapatkan
uang dan hasil penjualan produknya. Besarnya modal kerja umumnya adalah 25% dari biaya operasi atau
mencukupi kebutuhan operasi selama 3-6 bulan.
Modal kerja umumnya terdiri dari komponen-komponen biaya sebagai berikut:
a. Persediaan
 Bahan baku, berupa dimana cadangan endapan mineral/bijih yang ekonomis yang belum
dilaksanakan proses penambangan.
 Suku cadang, yangberguna apabilaterjadi ker usakan padaalat-alat penambangan.
 Supplies, merupakan stock dari suatu perusahan tambang yang berupa perlengkapan habis sekali
pakai seperti perlengkapankantor, bahan bakar, bahanpelumas, dll.
 Bahan dalam proses (materialsin process), berupa endapan mineral/bijih yang sedang atau dalam
proses penambangan ataudalam proses pengolahan (mineral dressing).
 Bahan jadi/produk tambang, merupakan bahan galian/bijih yang telah melalui proses pengolahan
yangsiap dijual.
b. Piutangdagang
Piutang dagang merupakan suatu modal kerja yang dapat ditarik sewaktu-waktu dari pihak
kedua sesuai dengan perjanjian dagang (seperti pembayaran diakhir transaksi penjualan komoditas
dagang).
c. Hutangdagang
Merupakan modal kerja yang diperoleh dari or ang lain dalam bentuk pinjaman yang bernilai
ekonomis, yang harus dibayar oleh kita apabila telah jatuh tempo sesuai dengan perjanjian daga ng
kedua belah pihak
d. Kas, dan lain-lain.
Merupakan cadangan uang yang disimpan yang berguna untuk membiayai kehidupan tambang
sehari-hari, dengan periode waktu yangrelatifsingkat.
2.3.2. Biaya Operasi
Biaya operasi(operating cost) adalah segala macam biaya yang harus dikeluarkanagar proyek penambangan
dapat beroperasi/berjalan dengan normal. Dalam suatu operasi penambangan, keseluruhan biaya
penambangan terdiri dari banyak komponenbiaya yang merupakan akibat dari masing-masing tahap
kegiatan. Besar kecilnya biaya penambanganakantergantung pada perancangan teknis sistem
penambangan, jenis dan jumlahalat yangdigunakan.
Aspek teknis dan aspek ekonomis tidak dapat berjalan sendiri-sendiri, keduanyaakanselalu saling
mempengaruhi. Perkiraan biaya investasi alatakan tergantung pada jumlah alat yang dipergunakan dan
kapasitas alat yang dipilih. Demikian pula biaya produksi merupakan fungsi dari kapasitas alat yang dipakai.
Biaya penambangan yang rendah dapat dicapai jika rancangan teknis dapat dioptimalkan dengan
memperhatikan pemilihan dan jumlah alat yang akan digunakan, yaitu yang dapat memberikan biaya produksi
per ton yangrendah.
Secara umum biaya operasi dibagi menjadi tiga komponen biaya, yaitu:
1) Biaya operasi langsung
Yuli Kusumawati, Catatan Kuliah Ekonomi Mineral - 11
Biayaoperasi langsung merupakan biaya utama dan berkaitan langsungdenganproduk yang dihasilkan.
Walaupun komponen biaya operasi langsung dari satu tambang ke tambang yang lain bervariasi, akan
tetapi padaumumnyaterdiri dari:
 Upah pekerja (pekerja lapangan, pengawas lapangan, dan sebagainya)
 Bahan bakar (bahan bakar, oli, dan sebagainya)
 Royalti.
 Persiapan daerah produksi/permukaan kerja
2) Biaya operasi taklangsung
Biaya operasi tak langsung adalah pengeluaran-pengeluaran yang tak terpengaruholeh produksi yang
dihasilkan. Biaya operasi langsung terdiri dari:
 Gaji pekerja (administrasi, keamanan, teknisi, jurubayar, petugas kantor, bengkel dan sebagainya).
 Asuransi.
 Penyusutanalat.
 Pajak.
 Reklamasi daerah bekas tambang.
 Perjalanan bisnis, rapat, sumbangan-sumbangan.
 Keperluan kantor.
 Humas dan sebagainya.
3) Biaya ov erhead
Biaya overhead dapat/tidak dapat dimasukkan sebagai ko mponen biaya operasi tetapi biaya-biaya ini
berpengaruh terhadap total biaya produksi walaupun umumnya mencerminkan biaya-biaya diluar
tambang/biaya-biayaperusahaan. Overheadbiasanya dikelompokkanmenjadi:
a. Penjualan.
b. Administrasi kantor pusat.
Yuli Kusumawati, Catatan Kuliah Ekonomi Mineral - 12
3. KONSEP DAN RUMUSAN BUNGA
3.1. Bunga sederhana dan bunga majemuk
Bunga (interest) adalah uang yang dibayarkkan atas penggunaan sejumlah pinjaman. Bunga juga bisa berarti
uang yang diperoleh dari investasi sejumlah modal tertentu sebagai bentuk kompensasi resiko yang timbul dari
inv estasi tersebut.
Tingkat/suku bunga (interest rate) adalah rasio antara bunga yang dibayarkan terhadap pokok dalam suatu
periode waktu.
Periode bunga (interest period) adalahinterval waktuyangdijadikandasar dalam perhitunganbunga.
Secara umum perhitungan bungadikelompokkan menjadi dua, yaitu:
1. Bunga sederhana (simple interest), yaitu bunga yang dihitung dari pokok uang/pinjaman selama periode
pinjaman.
I = P i n
Dimana:
I = bunga (interest)
P = pokok (principal)
i = tingkat/suku bunga (interest rate)
n = periode pinjaman
Sehingga jumlah uang/pinjaman padaakhir periode (F) adalah:
F = Pokok + Bunga
= P + P i n
= P (1 + i n)
Contoh 3-1:
Seorang petani meminjam uang di KUD sebesar Rp 10.000.000 dengan bunga 10%/tahun dengan
perhitungan bungasederhana. Berapahutangyangharus dibayar pada akhir tahunkeempat?
Jawab:
F = Rp 10.000.000 x (1 +(10% x 4))
= Rp 14.000.000
2. Bunga majemuk (compound interest), yaitu bunga yang dihasilkan dari pokok pinjaman dibungakan lagi
bersama-samadenganpokokpinjaman.
I = P(1 + i)n-1 . i
Sehingga jumlah uang/pinjaman padaakhir periode (F) adalah:
F = Pokok + Bunga
= P + P(1 + i)n-1 . i
= P (1 + i)n
Tujuan:
 Memahami konsep bunga sederhanadan bunga majemuk.
 Mampu menghitungsuku bunganominal, periode, danefektif.
 Mampu menerapkan rumus bunga majemuk yang sesuai untuk menghitung kesetaraan nilai uangpada
berbagai titikwaktu.
Yuli Kusumawati, Catatan Kuliah Ekonomi Mineral - 13
Contoh 3-2:
Seorang karyawan meminjam uang di Bank sebesar Rp 10.000.000 dengan bunga 10%/tahun dengan
perhitungan bungamajemuk. Berapa hutangyangharus dibayar pada akhir tahun keempat?
Jawab:
F = Rp 10.000.000 (1 + 10%)4
= Rp 14.641.000
Dari kedua contoh di atas ternyata bunga majemuk nilainya lebih tinggi dari bunga sederhana. Dalam
pembahasan pemilihanalternatifatauevaluasi rencanainvestasi digunakan bunga majemuk.
3.2. Suku bunga nominal dan suku bunga efektif
Suku bunga nominal (nominal rate) per tahun (r) adalah perkalian suku bunga per periode (i) dengan jumlah
pemajemukan dalam setahun (m).
r = i. m
Suku bunga efektif (effectiv e rate) disimbolkan E adalah suku bunga sesungguhnya dalam satu tahun yang
tepat dibayarkanterhadapsejumlah uangyangdisimpan atau dipinjam.
E = (1 + i)m -1 atau E = (1+ (r/m))m – 1 atau E = (F/P)1/n– 1
Contoh 3-3:
Bank ABC membebankan suku bunga sebesar 1,25 % per bulan pada semua saldo rekening yang belum
terbayar. Berapakah suku bunga nominal dansuku bunga efekti per tahunnya?
Jawab:
Suku bunga periode (i) = 1,25% per bulan
Jumlah pemajemukan/th (m) = 12
Suku bunga nominal (r) = 1,25% x 12 = 15% per tahun
Suku bunga efektif(E) = (1 + (0,15/12))12 – 1 = 16,18% per tahun
Tabel 3.1. Tingkat suku bunga efektifper tahun dari berbagai suku bunga nominal.
3.3. Minimum Attractive Rate of Return (MARR)
MARR adalah laju pengembalian minimum yang dikehendaki oleh investor.MARR tergantung pada lingkungan,
jenis kegiatan, tujuan dan kebijakan organisasi, dan tingkat resiko masing-masingproyek.
Pertimbangandalam menentukanMARR antaralain:
a. Jika perusahaan beroperasi dengan modal pinjaman, maka besarnya MARR harus lebih besar daripada
bunga yangdibayarkan padapinjamanitu.
Yuli Kusumawati, Catatan Kuliah Ekonomi Mineral - 14
b. Jika modal datang dari berbagai sumber (dana-dana yangberasal dari perusahaan sendiri, hutang jangka
panjang maupun jangka pendek, dan sumber-sumber keuangan lainnya), maka penentuan biaya modal
rata-ratabisa dipakai sebagai basis untukmenentukan besarnyaMARR.
c. Besarnya MARR harus setara dengan kecepatan pertumbuhan dari kekayaan perusahaan yang telah
ditetapkanoleh pimpinan perusahaan.
d. MARR untukproyekyangresikonya tinggi harus lebih besar daripada proyekyangresikonya rendah.
e. Perusahaan yang lebih bonafide (lebih banyak kesempatan mendapatkan proyek) biasanya menetapkan
MARR lebih tinggi daripadaperusahaansejenis yangbelum punyanama.
3.4. Waktu lipat dua
Waktu lipat dua (doubling time) adalah waktu yang diperlukan agar uang yang ditanamkan hari ini menjadi
dua kali lipat. Untukmemperkirakan waktuini dengan cepat digunakan angka ajaib(magic number) 72.
Waktu lipat dua = 72 / (suku bunga majemuk per periode x 100)
Akan tetapi penggunaan magic number ini akan salah jika periode pemajemukan kurangdari tiga (n≤3) atau
suku bunga (i) di atas 30%
Contoh 3-4:
Berapa tahun uang yang disimpan di bank menjadi dua kali lipat jika suku bunga yang berlaku adalah 8% per
tahun?
Jawab:
n = 72/(8% x 100) = 9 tahun
3.5. Rumus bunga majemuk
Notasi dalam rumus bunga:
i = suku bunga tiap periode bunga
n = jumlah periode bunga
P = nilai uangsekarang
F = nilai uangmendatang
A = pembayaranseri tahunan yangseragam
G = pembayaranseri gradient
3.5.1. Single payment compound-amount
Digunakan untuk menentukan nilai mendatang (F) dari sejumlah uang pada saat sekarang (P) selama periode
(n) tertentu dan padatingkat suku bunga(i) tertentu.
Contoh 3-5:
Seorang ayah mendepositokan uang Rp 100 juta sekarang di bank dengan tingkat bunga 12%/ tahun. Uang
tersebut akan diambil pada akhir tahun kelima, saat anaknya akan masuk universitas. Berapa jumlah uang yang
akan diperoleh sangayah?
Yuli Kusumawati, Catatan Kuliah Ekonomi Mineral - 15
Jawab:
F = Rp 100 juta (1 + 0,12)5
= Rp 100 juta (F/P 12%, 5)
= Rp 100 juta (1,7623)= Rp 176.230.000
3.5.2. Single payment present-worth
Digunakan untuk menentukan nilai sekarang dari sejumlah uang yang diterima pada waktu mendatang selama
periode tertentu dan tingkatbungatertentu.
Contoh 3-6:
Biaya masukSMA fav orit lima tahun mendatang diperkirakan Rp 30 juta. Untuk keperluan tersebut, seorang ayah
berniat membuka tabungan pendidikan yang memberikan bunga 18% per tahun. Berapa uang yang harus
disetorkan sekarang?
Jawab:
P = Rp 30 juta (1/(1 + 0,18)5)
= Rp 30 juta (P/F 18%, 5)
= Rp 30 juta (0,4371)= Rp 13.113.000
3.5.3. Uniform series compound-amount
Digunakan untuk menentukan nilai mendatang (F) dari suatu rangkaian pembayaran yang seragam (A) yang
terjadi pada setiapakhir periode tertentudantingkat bunga tertentu.
Contoh 3-7:
Seorang PNS menabung Rp 10 juta per tahun untuk keperluan investasi saat dia pensiun lima tahun yang akan
datang. Jika suku bunga yang diberikan bank 6% per tahun, berapa total uang yang dia kumpulkan sampai
akhir tahun ke-5?
Jawab:
F = Rp 10 juta ((1 + 0,06)5 – 1)/ 0,06
= Rp 10 juta (F/A6%, 5)
= Rp 10 juta (5,6371)= Rp 56.371.000
3.5.4. Sinking-fund deposit
Digunakan untuk menghitung besarnya pembayaran seri yang seragam pada tiap akhir tahun dari sejumlah
uangpada masa mendatangpadaakhir periode tertentudantingkat suku bunga tertentu.
Contoh 3-8:
Jika kita ingin memiliki uang $ 1000 pada akhir tahun keenam, berapa uang yang harus kita depositokan setiap
tahunnya, jika suku bunga deposito itu 10%/tahun?
A
A = Rp 10 jt/th
A A AA
0 21
F= ?
3 4 5
Yuli Kusumawati, Catatan Kuliah Ekonomi Mineral - 16
Jawab:
A = $1000 (0,1/((1 + 0,1)6 -1))
= $ 1000 (A/F10%,6)
= $ 1000 (0,12961)= $ 129,61
3.5.5. Capital-recovery
Digunakan untuk menentukan nilai aliran kas yang seragam setiap akhir tahun selama periode tertentu dengan
suku bunga tertentu dari nilai sekarang.
Contoh 3-9:
Untuk ekspansi bisnisnya seorang pengusaha meminjam uang di bank sebesar Rp 100 juta dengan suku bunga
5% per tahun untuk jangka waktu 10 tahun. Berapa angsuran yang dibayar pengusaha tersebut setiap
tahunnya?
Jawab:
A = Rp 100 juta {(0,05(1+0,05)10) / ((1+0,05)10 – 1)}
= Rp 100 juta (A/P5%,10)
= Rp 100 juta (0,1295)= Rp 12.950.000/tahun
3.5.6. Uniform series present-worth
Digunakan untuk menentukan nilai sekarang dari sejumlah pembayaran yang seragam setiap akhir tahun
selama periode tertentu dan suku bunga tertentu.
Contoh 3-10:
Sebuah dealer menawarkan mobil dengan uang muka Rp 10 juta dan sisanya dapat diangsur Rp 2 juta/bulan
selama 100 bulan. Bila bunga yang diberlakukan dealer tersebut adalah 1%/bulan, berapa harga mobil itu jika
dibayar tunai saat ini?
Jawab:
Nilai sekarangdari total angsuran:
P = Rp 2 juta (P/A1%,100)
= Rp 2 juta (63,029)
= Rp 126.038.000
Harga mobil tersebut = uangmuka +total angsuran
H = Rp 10 juta + Rp 126.038.000
= Rp 136.038.000
3.5.7. Uniform gradient-series
Digunakan untukmenghitungpembayaran yangnaikatauturun secarakonstan padasetiapperiode.
A = B + g {(1/i)-(n/(1+i)n – 1))}
A = B + g (A/G i,n)
A
A = ?
A A AA
0 ..1
F= $1000
.. .. 6
A
A = ?
A A AA
0 ..1
P=Rp 100 jt
.. .. 10
A
A= 2jt/bln
A A AA
0 ..1
P=?
.. .. 100
Yuli Kusumawati, Catatan Kuliah Ekonomi Mineral - 17
Contoh 3-11:
Biaya operasi dan pemeliharaan suatu mesin produksi dari tahun ke-1 sampai tahun ke-5 berturut-turut adalah
Rp 5 juta, Rp 7,5 juta, Rp 10 juta, Rp 12,5 juta, dan Rp 15 juta. Berapa biaya tersebut per tahunnya jika diketahui
tingkat suku bunga adalah 20% per tahun.
Jawab:
A = B + g (A/G i,n)
= Rp 5 juta + Rp 2,5 juta (A/G 20%,5)
= Rp 5 juta + Rp 2,5 juta (1,6045)
= Rp 9.011.250
Sama dengan diagram aliran kas:
Tabel 3.2. Rumus aljabar dankonversi rumus bungamajemuk
Yuli Kusumawati, Catatan Kuliah Ekonomi Mineral - 18
4. INDIKATOR-INDIKATOR KELAYAKAN INVESTASI
Kelayakan suatu investasi (proyek) dari aspek ekonomi dapat dianalisis berdasarkan manfaat (benefit) dan
biaya (cost). Suatu proyek dikatakan layak untuk dikerjakan jika manfaat yang diberikan lebih besar daripada
biaya yang dikeluarkan. Untuk menilai keekonomian suatu proyek perlu dilihat seluruh pengeluaran dan
pendapatan sepanjang umur proyek tersebut (life cycle analysis), bukan hanya dengan melihat biaya per
satuan produksi di tahun tertentu.
Indikator keuntungan yangdigunakandalam pengambilan keputusan seyogyanyamempunyai ciri-ciri:
1. Dapat dengan tepat dalam membandingkan dan mengelompokkan kesanggupan memberikan
keuntungan (profitability) dari kesempatan-kesempatan penanamanmodal.
2. Mencerminkan nilai waktu dari modal perusahaan dan secara realistis merupakan masukan bagi
kebijaksanaan fiskal dari perusahaan, termasukinvestasi kembali di masa yangakan datang.
3. Dapat menunjukkan keuntungan meskipun sekecil-kecilnya.
4. Mencakup pernyataan-pernyataankuantitatifdari resiko.
5. Menggambarkan faktor-faktor lain, seperti hasil-hasil gabungan, resiko, dan kekayaan perusahaan bila
mungkin.
Beberapa indikator yangdigunakan untukmenilai kelayakan proyek/investasi antara lain:
a. Net present v alue (NPV)
b. Rate of return (ROR)
c. Present v alue ratio (PVR) atauBenefit-cost ratio (BCR)
d. Paybackperiod (PBP)
Pada dasarnya untukanalisis inv estasi tunggal, semua indikator tersebut akan memberikan hasil keputusan yang
konsisten satu sama lain, tetapi informasi spesifik yang dihasilkan tentu akan berbeda. Sehingga dalam
prakteknya beberapa indikator sering digunakan secara bersamaan untuk mendapatkan gambaran yang lebih
komprehensif.
4.1. Net present value (NPV)
NPV adalah perbedaan antara total present worth dari total penerimaan (revenue) atau penghematan (saving)
dan present worth dari pengeluaransepanjangumur proyekpadadiscountrate yangdiberikan.
NPV = ∑ PW revenue - ∑ PW cost
NPV menunjukkan keuntunganproyeksecara absolut.Suatuproyekdikatakan layak(feasible) apabilaNPV>0.
Cara menghitung NPV bukan trial and error, memperhitungkan nilai waktu uang, menggunakan discount rate
sama dengan MARR, danbisa mempertimbangkanresiko.
Contoh 4-1:
Suatu proyekmemiliki aliran kas sebagai berikut:
Tahun ke- 0 1 2 3 4 5
Aliran kas (x Rp 1.000.000) -100 20 30 20 40 40
Tujuan:
 Mampu memahami konsep analisis kelayakan investasi tunggal.
 Mampu menganalisis kelayakaninvestasi tunggal berdasarkan indikator NPV, ROR, PVR, BCR, dan PBP
Yuli Kusumawati, Catatan Kuliah Ekonomi Mineral - 19
Analisis kelayakan proyek tersebut berdasarkan NPVjika diketahui MARR = 10%.
Jawab:
NPV = ∑ PW rev enue - ∑ PW cost
= 20(P/F,10%,1)+30(P/F,10%,2)+20(P/F,10%,3)+40(P/F,10%,4)+40(P/F,10%,5)-100
= 20(0,9091)+30(0,8264)+20(0,7513)+40(0,6830)+40(0,6209)-100
= 10,156
Karena NPV>0, maka proyektersebut layak.
4.2. Rate of return (ROR)
ROR adalah discount rate (i) yang menyebabkan NPV = 0.ROR merupakan laju perolehan per tahun yang
dihasilkan oleh inv estasi suatuproyek(menunjukkankeuntungan secara relatifterhadap skala investasi proyek).
Cara menghitung ROR adalah coba-coba (trial and error), memperhitungkan nilai waktu uang, lebih
dipengaruhi cashflow awal, tidak tergantung nilai absolut cashflow, tidak bisa dihitung jika semua cashflow
negatifatau positifatau belum balikmodal, dan bisadiperoleh nilai ganda.
Suatu proyekdikatakan layakjika ROR>MARR.
Contoh 4-2:
Suatu proyekmemiliki aliran kas sebagai berikut:
Tahun ke- 0 1 2 3 4 5
Aliran kas (x Rp 1.000.000) -100 20 30 20 40 40
Analisis kelayakan proyektersebut berdasarkan ROR jika diketahui MARR = 10%.
Jawab:
ROR adalahnilai i yangmenghasilkan NPV=0, diperoleh dengan cara coba-coba:
Untuki = 10% diperoleh NPV = 10,156 (perhitungannya seperti contoh4.1)
Untuki = 15%, nilai NPVadalah:
NPV = 20(P/F,15%,1)+30(P/F,15%,2)+20(P/F,15%,3)+40(P/F,15%,4)+40(P/F,15%,5)-100
= 20(0,8696)+30(0,7561)+20(0,6575)+40(0,5718)+40(0,4972)-100
= -4,015
Selanjutnyadicari nilai RORdengan interpolasi:
ROR−10%
10 ,156 −0
=
15%−10%
10,156−(−4,015)
ROR = 10% + (15%-10%)((10,156/(10,156+4,02))
=13,58%
Karena ROR>MARR, maka proyektersebutlayak.
4.3. Present value ratio (PVR)
PVR adalah perbandingan antara NPV dengan nilai absolut investasi yang dikeluarkan (PW net cashflow
negatif).
𝐏𝐕𝐑 =
𝐍𝐏𝐕
│𝐏𝐖 𝐧𝐞𝐭 𝐂𝐅 (−)│
Proyekdikatakan layakjika PVR>0.
ROR=?10%
0
-4,015
10,156
15%
Yuli Kusumawati, Catatan Kuliah Ekonomi Mineral - 20
Contoh 4-3:
Suatu proyekmemiliki aliran kas sebagai berikut:
Tahun ke- 0 1 2 3 4 5
Aliran kas (x Rp 1.000.000) -100 20 30 20 40 40
Analisis kelayakan proyektersebut berdasarkan PVR jika diketahui MARR = 10%.
Jawab:
PVR = NPV/│PW net CF (-)│
= 10,156/│-100│= 0,10
Karena PVR>0, maka proyektersebut layak.
4.4. Benefit-cost ratio (BCR)
BCR adalah perbandinganantara discountedbenefit (savingataurevenue) dengan investasi.
𝐁𝐂𝐑 =
𝐏𝐖 𝐧𝐞𝐭 𝐂𝐅(+)
│𝐏𝐖 𝐧𝐞𝐭 𝐂𝐅 (−)│
Proyekdikatakan layakjika BCR>1.
HubunganPVR danBCR adalah:
BCR = PVR + 1 atau PVR = BCR -1
Contoh 4-4:
Suatu proyekmemiliki aliran kas sebagai berikut:
Tahun ke- 0 1 2 3 4 5
Aliran kas (x Rp 1.000.000) -100 20 30 20 40 40
Analisis kelayakan proyektersebut berdasarkan BCR jika diketahui MARR = 10%.
Jawab:
BCR = PW net CF(+)/│PW net CF (-)│
= 20(P/F,10%,1)+30(P/F,10%,2)+20(P/F,10%,3)+40(P/F,10%,4)+40(P/F,10%,5) / │-100│
= 110,156/100
= 1,10
Atau
BCR = PVR + 1
= 0,10 + 1 = 1,10
Karena BCR>1, maka proyektersebut layak.
4.5. Payback period (PBP)
PBP adalah jangka waktu yang diperlukan untuk mengembalikan modal investasi yang ditanam (investasi
mencapai titik impas). Simple PBP adalah PBP dengan discount rate sama dengan nol (undiscounted).
Kelemahan PBP adalah tidak mempersoalkan keuntungan investasi.PBP dihitung dengan cara interpolasi
berdasarkan aliran kas kumulatif.Diterima atau tidaknya suatu proyek pada periode pengembalian tertentu
tergantung pada investor. Proyek akan diterima jika PBP lebih singkat dari periode terpendek yang diinginkan
inv estor.
Yuli Kusumawati, Catatan Kuliah Ekonomi Mineral - 21
Contoh 4-5:
Suatu proyekmemiliki aliran kas sebagai berikut:
Tahun ke- 0 1 2 3 4 5
Aliran kas (x Rp 1.000.000) -100 20 30 20 40 40
Analisis kelayakan proyektersebut berdasarkan undisconteddandiscounted PBP jika diketahui MARR = 10%.
Jawab:
a. Simple PBP
Terlebih dulu menghitungaliran kas kumulatifnya:
Tahun ke- 0 1 2 3 4 5
Aliran kas (x Rp 1 juta) -100 20 30 20 40 40
Aliran kas kumulatif(x Rp 1 juta) -100 -80 -50 -30 10 50
Terlihat bahwa periode pengembalian investasi proyek tersebut antara tahun ke-3 dan ke-4, sehingga PBP
dapat dihitungdenganinterpolasi:
𝐏𝐁𝐏 = 𝐭𝐚𝐡𝐮𝐧 𝐂𝐮𝐦. 𝐂𝐅(−)𝐭𝐞𝐫𝐚𝐤𝐡𝐢𝐫 + │
𝐂𝐮𝐦. 𝐂𝐅(−)𝐭𝐞𝐫𝐚𝐤𝐡𝐢𝐫
𝐂𝐅(+) 𝐭𝐡 𝐛𝐞𝐫𝐢𝐤𝐮𝐭𝐧𝐲𝐚
│
PBP = 3 + │
−30
40
│ = 3 + 0,75= 3,75 tahun.
b. Discounted PBP denganMARR 10%
Terlebih dulu menghitungaliran kas diskonto dankumulatifnya:
DCF th-1 = 20(P/F,10%,1) = 20(0,9091) = 18,182
DCF th-2 = 30(P/F,10%,2) = 30(0,8264) = 24,792
DCF th-3 = 20(P/F,10%,3) = 20(0,7513) = 15,026
DCF th-4 = 40(P/F,10%,4) = 40(0,6830) = 27,320
DCF th-5 = 40(P/F,10%,5) = 40(0,6209) = 24,836
Tahun ke- 0 1 2 3 4 5
Discounted CF (x Rp 1 juta) -100 18,182 24,792 15,026 27,320 24,836
CummulatifDCF (x Rp 1 juta) -100 -81,818 -57,026 -42,000 -14,680 10,156
Terlihat bahwa periode pengembalian investasi proyek tersebut antara tahun ke-4 dan ke-5, sehingga PBP
dapat dihitungdenganinterpolasi:
𝐏𝐁𝐏 = 𝐭𝐚𝐡𝐮𝐧 𝐂𝐮𝐦. 𝐂𝐅(−)𝐭𝐞𝐫𝐚𝐤𝐡𝐢𝐫 + │
𝐂𝐮𝐦. 𝐂𝐅(−)𝐭𝐞𝐫𝐚𝐤𝐡𝐢𝐫
𝐂𝐅(+) 𝐭𝐡 𝐛𝐞𝐫𝐢𝐤𝐮𝐭𝐧𝐲𝐚
│
PBP = 4 + │
−14,680
24,836
│= 4 + 0,59 = 4,59 tahun.
Yuli Kusumawati, Catatan Kuliah Ekonomi Mineral - 22
5. ANALISIS MUTUALLY EXCLUSIVE
Proyek dapat didefinisikan sebagai suatu kegiatan ekonomi dengan tujuan tertentu. Untuk mencapai tujuan
tersebut terdapat berbagai alternatif yang dapat dipilih. Pertimbangan untuk memilih alternatif yang terbaik
adalah yang membutuhkan biaya minimum atau yang menghasilkan keuntungan maksimum atau yangwaktu
pengembalianinvestasinya palingsingkat.
Secara umum investasi dibedakanmenjadi dua, yaitu:
a. Investasi yang menghasilkan pendapatan (revenue) atau penghematan (saving). Pertimbangan yang
digunakan untukmenilai kelayakaninvestasi ini adalah keuntungan yangmaksimum.
b. Investasi yang tidak menghasilkan revenue atau saving. Pertimbangan yang digunakan untuk menilai
kelayakan investasi ini adalahbiayayangminimum.
Analisis mutually exclusive adalah menganalisis beberapa alternatif dimana hanya ada satu alternatif yang akan
dipilih, yaitu alternatif yang akan menghasilkan pelayanan (service) terbaik, atau yang akan memperbaiki,
memperluas, mengembangkan operasi yang ada, atau yang akan menghasilkan pemasukan (income)
terbesar. Alternatif yang dipilih adalah yang mempunyaiROR, NPV, BCR, atau PVR yang paling tinggi
dibandingkan alternatif yang lain. Akan tetapi terkadang suatu alternatif mempunyai ROR yang lebih tinggi
dibanding alternatif yang lain tetapi NPV nya lebih rendah. Untuk kondisi tersebut maka indikator kelayakan
tersebut harus dihitungnilai inkrementalnya.
5.1. Analisis mutually exclusive alternatif dengan umur sama
Jika terdapat beberapa alternatif proyek/investasi yang mempunyai umur investasi yang sama, maka prosedur
analisis mutually exclusive alternatiftersebut adalahsebagai berikut:
1. Urutkan alternatifdari yangmempunyai investasi awal terendah ke tertinggi.
2. Buat cashflow inkremental (∆CF), yaitu alternatif yang investasi awalnya lebih tinggi (penantang/challenger)
dikurangi alternatifyanginvestasi awalnyalebih rendah (petahana/defender).
3. Tentukan nilai NPVinkremental, ROR incremental, BCR inkremental, PVR inkremental.
4. Analisis dengan kriteria sebagai berikut:
a. Jika ∆NPV> 0, maka pilih alternatifdengan investasi yanglebih besar (penantang) dansebaliknya.
b. Jika ∆ROR> MARR, maka pilih alternatifdenganinvestasi yanglebih besar (penantang) dan sebaliknya.
c. Jika ∆BCR> 1, maka pilih alternatifdengan investasi yanglebih besar (penantang) dan sebaliknya.
d. Jika ∆PVR> 0, maka pilih alternatifdengan investasi yanglebih besar (penantang) dan sebaliknya.
Contoh 5-1:
Untuk memenuhi kebutuhan listrik di area penambangan, investor bermaksud membeli generator listrik. Ada tiga
alternatifgenerator listrik yangditawarkanoleh penjual, denganbiaya dan manfaat sebagai berikut:
Merekgenerator X Y Z
Harga (x Rp 1 juta) 17 25 28
Biaya operasi dan perawatan (x Rp 1 juta) 8 6 5
Nilai manfaat/tahun (x Rp 1 juta) 13 13 13
Nilai sisa (x Rp 1 juta) 5 8 14
Tujuan:
 Mampu memahami konsep pemilihanalternatifinvestasi secara mutually exclusive.
 Mampu melakukananalisis mutually exclusiveberdasarkanindikator NPV, ROR,BCR, dan PVR.
Yuli Kusumawati, Catatan Kuliah Ekonomi Mineral - 23
Ketiga generator tersebut dibeli pada tahun ini (tahun ke-0) dan akan digunakan pada tahun depan (tahun ke-
1) serta memiliki umur pemakaian yang sama yaitu 5 tahun.Jika hanya satu generator yang akan dibeli,
tentukan generator manayangpalingmenguntungkan dengan asumsi MARR 14%/tahun.
Jawab:
Diagram cash flow masing-masingalternatifadalahsebagai berikut:
Alternatif X:
Alternatif Y:
Alternatif Z:
Kemudian urutkan alternatif dari yang mempunyai investasi terkecil sampai yang mempunyai investasi terbesar,
dengan cara membuat tabel cashflow masing-masingalternatifdaninkrementalnya:
Tahun ke- X Y Z Y-X Z-Y
0 -17 -25 -28 -8 -3
1 5 7 8 2 1
2 5 7 8 2 1
3 5 7 8 2 1
4 5 7 8 2 1
5 10 15 22 5 7
Selajutnya dianalisis berdasarkannilai inkrementalnya:
Bandingkan generator X dengangenerator Y:
∆NPVY-X @ 14% =- 8 + 2 (P/A,14%,4) + 5 (P/F,14%,5) = - 8 + 2(2,914) + 5(0,519) = 0,423 juta
Karena NPVY-X> 0, maka pilih Y (alternatifdengan investasi awal lebih besar).
Lalu bandingkangenerator Y dengan generator Z:
∆NPVZ-Y @ 14% = - 3 + 1 (P/A,14%,4) + 7 (P/F,14%,5) = - 3 + 1(2,914) + 7(0,519) = 3,547 juta
Karena ∆NPVZ-Y > 0, maka pilih Z (alternatifdenganinvestasi awal lebih besar).
∆RORY-X (dihitungdengancara coba-coba)= 15,9%
Karena ∆RORY-X> MARR, makapilih Y (alternatifdengan investasi awal lebih besar).
Lalu bandingkangenerator Y dengan generator Z:
∆RORZ-Y (dihitungdengan caracoba-coba) = 42,1%
Karena ∆RORZ-Y> MARR, maka pilih Z (alternatifdenganinvestasi awal lebih besar).
CX = 17 juta
1
OCX = 8 juta
IX = 13 juta
OCX = 8 juta
IX = 13 juta
LX = 5 juta
...........................0 4
OCX = 8 juta
IX = 13 juta
5
CFX0 = -17 juta CFX1 = 5 juta CFX4 = 5 juta CFX5 = 10 juta
Cy = 25 juta
1
OCy = 6 juta
Iy = 13 juta
OCy = 6 juta
Iy = 13 juta
Ly = 8 juta
...........................0 4
OCy = 6 juta
Iy = 13 juta
5
CFY0 = -25 juta CFY1 = 7 juta CFY4 = 7 juta CFY5 = 15 juta
CZ = 28 juta
1
OCZ = 5 juta
IZ = 13 juta
OCZ = 5 juta
IZ = 13 juta
LZ = 14 juta
...........................0 4
OCZ = 5 juta
IZ = 13 juta
5
CFZ0 = -28 juta CFZ1 = 8 juta CFZ4 = 8 juta CFZ5 = 22 juta
Yuli Kusumawati, Catatan Kuliah Ekonomi Mineral - 24
∆PVRY-X @ 14% = (∆NPVY-X @ 14%)/│PW net ∆CFY-X (-)│ = 0,423 / 8 = 0,052
Karena ∆PVRY-X> 0, maka pilih Y (alternatifdengan investasi awal lebih besar).
Lalu bandingkangenerator Y dengan generator Z:
∆PVRZ-Y @ 14% = (∆NPVZ-Y @ 14%)/│PW net ∆CFZ-Y (-)│ =3,547 / 3 = 1,182
Karena ∆PVRZ-Y> 0, maka pilih Z (alternatifdengan investasi lebih besar).
∆BCRY-X @ 14% = ∆PVRY-X @ 14% + 1 = 0,052 + 1 = 1,052
Karena BCRY-X>1, maka pilih Y (alternatifdenganinvestasi awal lebih besar).
Lalu bandingkangenerator Y dengan generator Z:
∆BCRZ-Y @ 14% = ∆PVRZ-Y @ 14% + 1 = 1,182 + 1 = 2,182
Karena ∆BCRZ-Y>1, maka pilih Z (alternatifdengan investasi awal lebih besar).
Hasil perhitungan dirangkum dalam tabel berikut ini:
Tahun ke- CF X CF Y CF Z CF (Y-X) CF (Z-Y)
0 -17 -25 -28 -8 -3
1 5 7 8 2 1
2 5 7 8 2 1
3 5 7 8 2 1
4 5 7 8 2 1
5 10 15 22 5 7
NPV 2,76 3,19 6,74 0,4 3,5
ROR 19,9% 18,6% 22,1% 15,9% 42,1%
PVR 0,16 0,13 0,24 0,05 1,18
BCR 1,16 1,13 1,24 1,05 2,18
Terbukti dengan menganalisis secara inkremental maka semua indikator tersebut memberikan hasil yang
sama/konsisten (generator Z palinglayakuntukdipilih).
5.2. Analisis mutually exclusive alternatif dengan umur berbeda
Seringkali alternatif-alternatif yangada memiliki umur yang berbeda, sehingga diperlukan basis umur yang sama
untukbisa menganalisisnya.
Perhatikan diagram cashflow dari dua alternatifberikut ini:
Alternatif A:
Alternatif B:
Ada tiga metode untukmenyamakan basis umur alternatif, yaitu:
a. Berdasarkan kelipatanpersekutuanterkecil dari masing-masingumur alternatif.
Kelemahan metode ini adalah jika diperoleh hasil kelipatan persekutuan terkecil yang besar maka akan
terlalu banyakdummy reinvestment(investasi ulang semu).
Alternatif A:
Alternatif B:
CA
2
OCA2
0 1
OCA1
CB
2
OCB2
0 31
OCB1 OCB3
LA
LB
LA
CA
2
OCA2
0 1
OCA1
LB
CB
2
OCB2
0 31
OCB1 OCB3
LA
CA
4
OCA2
2 3
OCA1
LB
CB
5
OCB2
3 64
OCB1 OCB3
LA
CA
6
OCA2
4 5
OCA1
Yuli Kusumawati, Catatan Kuliah Ekonomi Mineral - 25
b. Berdasarkan umur alternatifterpanjang
Alternatif dengan umur lebih pendek akan ditambahkan investasi ulang semu sampai periodenya sama
dengan alternatifdenganumur terpanjang.
Jika menggunakan alat yang sama maka nilai sisa pada investasi ulang semu adalah nilai buku pada
periode yangterpotong.
Jika menggunakan alatbaru (CA ≠ C’A) padainvestasi ulangsemu, maka nilai LA ≠ L’A.
Alternatif A:
Alternatif B:
c. Berdasarkan umur alternatifterpendek.
Pada alternatif yang dikurangi umurnya, maka nilai sisa yang baru sama dengan nilai buku yang terpotong
(L’B> LB).
Alternatif A:
Alternatif B:
Contoh 5-1:
Seorang investor dihadapkan pada pilihan untuk menjual bukit karst miliknya seharga Rp 150 juta atau
mengembangkan lahan tersebutdengan investasi Rp 300 juta pada tahun ke-0 dan Rp 400 juta pada tahun ke-1
akan menghasilkan keuntungan tetap dari tahun ke-2 sampai tahun ke-10 sebesar Rp 200 juta. Jika MARR yang
ditetapkanadalah15%, keputusanmanayangpalingmenguntungkan secara ekonomi?
Jawab:
Diagram cash flow masing-masing alternatifadalah:
Alternatif A (mengembangkan lahan):
Alternatif B (lahan dijual):
Diagram inkremental cashflow (alternatifA – alternatifB) adalah:
∆NPVA-B = - 450 - 400(P/F,15%,1) +200(P/A,15%,9)(P/F,15%,1)
= - 450 – 400(0,870) + 200(4,772)(0,870)
= Rp 32,3 juta
Keputusanpilih alternatifA(∆NPVA-B> 0, pilih alternatifdenganinvestasi awal lebih besar)
LA
CA
2
OCA2
0 1
OCA1
L’A
C’A
2 3
OCA3
LB
CB
2
OCB2
0 31
OCB1 OCB3
LA
CA
2
OCA2
0 1
OCA1
L’B
CA
2
OCB2
0 1
OCB1
20 31 64 75 108 9
200-300 200-400 200200 200200 200200 200
20 31 64 75 108 9
-150 -- -- -- -- -
20 31 64 75 108 9
200-450 200-400 200200 200200 200200 200
Yuli Kusumawati, Catatan Kuliah Ekonomi Mineral - 26
∆RORA-Bdihitungdengancara coba-coba:
∆NPVA-B @15% = Rp 32,3 juta
∆NPVA-B @ 20% = - 450 - 400(P/F,20%,1) +200(P/A,20%,9)(P/F,20%,1)
= - 450 - 400(0,833) + 200(4,031)(0,833)
= Rp -111,6 juta
Selanjutnyadicari nilai ∆RORA-B denganinterpolasi:
(∆RORA-B-15%)/(32,3-0) = (20%-15%)/(32,3-(-111,6))
∆RORA-B = 15% + (20%-15%)((32,3/(32,3+111,6))
=16%
Karena ∆RORA-B>MARR, maka pilih alternatifA.
∆PVRA-B = ∆NPVA-B /│PW net ∆CF (-)│
= 32,3 /│(-450 + -400(P/F,15%,1)│
= 0,04
Karena ∆PVRA-B> 0, maka pilih alternatifA.
∆BCRA-B = ∆PVRA-B + 1
= 0,04 + 1
= 1,04
Karena ∆BCRA-B>1, maka pilih alternatifA.
∆RORA-B
=?
15%
0
-111,6
32,3
20%
Yuli Kusumawati, Catatan Kuliah Ekonomi Mineral - 27
6. ANALISIS NON-MUTUALLY EXCLUSIVE
Analisis non-mutually exclusive adalah analisis terhadap beberapa alternatif dimana dari beberapa alternatif
tersebut dapat dipilih lebih dari satu alternatif, sesuai dengan tersedianya modal atau anggaran untuk investasi
tersebut.
Tujuan dari analisis non-mutually exclusive adalah untuk memaksimalkan keuntungan kumulatif yang dapat
dihasilkan dari investasi (cummulative net value atau cummulative future worth profit) berdasarkan kombinasi
beberapa alternatif.
Aplikasi analisis non-mutually exclusive antara lain pemilihan alternatif riset dan pengembangan, pemilihan
metode eksplorasi, serta meranking prospekpengeboran padaindustri minyak.
Ada dua teknikyangdigunakan untukmeranking proyeknon-muatually exclusive, yaitu:
a. Growth rate ofreturn (growth ROR), yaitusuatutingkat suku bungamajemukdimana investasi tumbuh.
b. Analisis rasio, baikmenggunakan PVR atauBCR.
Individual ROR dan individual NPV tidak bisa digunakan untuk meranking proyek non-mutually exclusive, karena
individual NPV maupun RORyang besar belum tentu mencerminkan alternatif terbaik.Analisis menggunakan net
value analysis (NPV,NAV,NFV) pada proyek non-mutually exclusive digunakan untuk memilih kombinasi proyek
yangmenghasilkan nilai bersih kumulatifterbesar.
6.1. Analisis non-mutually exclusive dengan umur alternatif sama
Contoh 6-1:
Seorangpengusahadihadapkanpadaempatalternatifpembelian mesin seperti berikut:
Mesin A Mesin B Mesin C
Biaya inv estasi (xRp1000) 10.000 25.000 35.000
Keuntungan (xRp 1000) 6.000 10.000 15.000
Umur mesin 5 tahun 5 tahun 5 tahun
Jika pengusaha itu memiliki dana Rp 35 juta untuk pembelian mesin tersebut, dan dia juga mempunyai pilihan
untuk berinvestasi pada proyek lain yang memberi keuntungan 20%/tahun, mesin mana yangdipilih secara non-
mutually exclusive?
Jawab:
NPVA = -10 juta + 6 juta(P/A,20%,5) = -10 juta + 6 juta (2,991) = Rp 7,946 juta
NPVB = -25 juta + 10 juta(P/A,20%,5) = -25 juta + 10 juta (2,991) = Rp 4,910 juta
NPVC = -35 juta + 15 juta(P/a,20%,5) = -35 juta + 15 juta (2,991) = Rp 9,865 juta
Analisis:
Total harga mesin A + mesin B = Rp 35 juta, NPVA + NPVB= Rp 12,856 juta.
Harga mesin C = Rp 35 juta, NPVC = Rp 9,865 juta.
Kesimpulan: lebih menguntungkan membeli mesin A + mesin B.
PVRA = NPVA /│PW net CFA (-)│ = 7,946 /│-10│ = 0,795
PVRB = NPVB /│PW net CFB (-)│ = 4,910 /│-25│ = 0,196
Tujuan:
 Mampu memahami konsep pemilihanalternatifinvestasi non-mutually exclusive.
 Mampu melakukananalisis non-mutually exclusive berdasarkan indikator NPV, ROR, BCR, danPVR.
Yuli Kusumawati, Catatan Kuliah Ekonomi Mineral - 28
PVRC = NPVC /│PW net CFC (-)│ = 9,865 /│-35│ = 0,282
Berdasarkan PVR diperoleh rankingsebagai berikut:
Mesin A (PVRA = 0,795) palingmenguntungkan, disusul mesin C (PVRC = 0,282), dan terakhir mesin B (PVRB = 0,196).
BCRA = PVRA + 1 = 0,795 + 1 = 1,795
BCRB = PVRB + 1 = 0,196+ 1 = 1,196
BCRC = PVRC + 1 = 0,282 + 1 = 1,282
Berdasarkan BCR diperoleh rankingsebagai berikut:
Mesin A (BCRA= 1,795) palingmenguntungkan, disusul mesin C (BCRC = 1,282), dan terakhir mesin B (BCRB = 1,196).
Growth ROR mesin A:
Initial
Keuntungan yangdiperoleh diinvestasikanlagi ke proyek lain dengan i* = 20%:
Reinv est
Menghasilkan total pendapatan di akhir umur proyeksebesar:
FA = 6jt(F/A,20%,5) = 6jt(7,442) = Rp 44,65 juta
Initial +Reinv est
Growth ROR alternatifA (GRORA)dihitungdengancoba-coba:
NPVA @ 35% = - 10jt + 44,65jt (P/F,35%,5) = - 10jt + 44,65jt (0,269) = Rp 2,01 jt
NPVA @ 40% = - 10jt + 44,65jt (P/F,40%,5) = - 10jt + 44,65jt (0,186) = Rp -1,69 jt
Selanjutnyadicari nilai interpolasinya:
(GRORA-35%)/(2,01-0) = (40%-35%)/(2,01-(-1,69))
GRORA = 35% + (40%-35%)((2,01/(2,01+1,69))=37,7%
Growth ROR mesin B:
Initial
Keuntungan yangdiperoleh diinvestasikanlagi ke proyek lain dengan i* = 20%:
Reinv est
Menghasilkan total pendapatan di akhir umur proyeksebesar:
FB = 10jt(F/A,20%,5) = 10jt(7,442) = Rp 74,42 juta
Initial +Reinv est
Growth ROR alternatifB (GRORB)dihitungdengan coba-coba:
NPVB @ 20% = - 25jt + 74,42jt (P/F,20%,5) = - 25jt + 74,42jt (0,402) = Rp 4,92 jt
NPVB @ 25% = - 25jt + 74,42jt (P/F,25%,5) = - 25jt + 74,42jt (0,328) = Rp -0,59 jt
3 420 1 5
6jt6jt6jt6jt6jt-10jt
3 420 1 5
6jt6jt6jt6jt6jt-
FA = ?
3 420 1 5
------10jt
FA = 44,65jt
0
GROR =?35%
-1,69
2,01
40%
3 420 1 5
10jt10jt10jt10jt10jt-25jt
3 420 1 5
10jt10jt10jt10jt10jt-
FB = ?
3 420 1 5
------25jt
FB = 74,42jt
Yuli Kusumawati, Catatan Kuliah Ekonomi Mineral - 29
Selanjutnyadicari nilai interpolasinya:
(GRORB- 20%)/(4,92-0) = (25%-20%)/(4,92-(-0,59))
GRORB = 20% + (25%-20%)((4,92/(4,92+0,59)) = 24,46%
Growth ROR mesin C:
Initial
Keuntungan yangdiperoleh diinvestasikanlagi ke proyek lain dengan i* = 20%:
Reinv est
Menghasilkan total pendapatan di akhir umur proyeksebesar:
FC = 15jt(F/A,20%,5) = 15jt(7,442) = Rp 111,63 juta
Initial + Reinv est
Growth ROR alternatifC (GRORC)dihitungdengan coba-coba:
NPVC @ 30% = - 35jt + 111,63jt (P/F,20%,5) = - 35jt + 111,63jt (0,269) = Rp -4,97 jt
NPVC @ 25% = - 35jt + 111,63jt (P/F,25%,5) = - 35jt + 111,63jt (0,328) = Rp 1,61 jt
Selanjutnyadicari nilai interpolasinya:
(GRORC-25%)/(1,61-0) = (30%-25%)/(1,61-(-4,97))
GRORC = 25% + (30%-25%)((1,61/(1,61+4,97)) = 26,22%
Berdasarkan growth RORdiperoleh rankingsebagai berikut:
Mesin A (GRORA=37,7%) palingmenguntungkan, lalu mesin C (GRORC=26,22%), terakhir mesin B (GRORB=24,46%).
Hasil ini konsisten dengan rankingmetode PVR dan BCR.
6.2. Analisis non-mutually exclusive dengan umur alternatif berbeda
Contoh 6-2:
Seorang manager memiliki anggaran sebesar Rp 50 juta untuk dialokasikan pada alternatif investasi berikut ini
dengan pemilihan non-mutually exclusive. Tingkat pengembalian minimum yang ditetapkan adalah 10%.
Tentukan alternatifmanayangsebaiknya dipilih?
A)
B)
C)
Jawab:
NPVA = -50 jt + 20 jt(P/A,10%,2) + 50jt(P/F,10%,2) = -50 jt + 20 jt(1,736) + 50jt(0,826) = 26,033 jt
NPVB = -30 jt + 10 jt(P/A,10%,5) + 30jt(P/F,10%,5) = -30 jt + 10 jt(3,791) + 30jt(0,621) = 26,535 jt
NPVC = -20 jt + 5 jt(P/A,!0%,5) + 20jt(P/F,10%,7) = -20 jt + 5 jt(4,868) + 20jt(0,513) = 14,605 jt
0
GROR =?20%
-0,59
4,92
25%
3 420 1 5
15jt15jt15jt15jt15jt-35jt
3 420 1 5
15jt15jt15jt15jt15jt-
FC = ?
3 420 1 5
------35jt
FC = 111,63jt
0
GROR =?25%
-4,97
1,61
30%
L = 50jt
20jt20jt-50jt
20 1
L = 30jt
10jt10jt10jt10jt10jt-30jt
3 420 1 5
5jt5jt5jt5jt5jt-20jt
3 420 1 5
L = 20jt
5jt5jt
76
Yuli Kusumawati, Catatan Kuliah Ekonomi Mineral - 30
Analisis:
Total harga mesin B + mesin C = Rp 50 juta, NPVB + NPVC = Rp 41,140 juta.
Harga mesin A = Rp 50 juta, NPVA = Rp 26,033 juta.
Kesimpulan: lebih menguntungkan membeli mesin B + mesin C.
PVRA = NPVA /│PW net CFA (-)│ = 26,033 /│-50│ = 0,52
PVRB = NPVB /│PW net CFB (-)│ = 26,535 /│-30│ = 0,88
PVRC = NPVC /│PW net CFC (-)│ = 14,605 /│-20│ = 0,73
Berdasarkan PVR diperoleh rankingsebagai berikut:
Mesin B (PVRB = 0,88) palingmenguntungkan, disusul mesin C (PVRC = 0,73), dan terakhir mesin A (PVRA = 0,52).
BCRA = PVRA + 1 = 0,52 + 1 = 1,52
BCRB = PVRB + 1 = 0,88 + 1 = 1,88
BCRC = PVRC + 1 = 0,73 + 1 = 1,73
Berdasarkan BCR diperoleh rankingsebagai berikut:
Mesin B (BCRB = 1,88) palingmenguntungkan, disusul mesin C (BCRC = 1,73), dan terakhir mesin A (BCRA = 1,52).
Growth ROR mesin A:
Initial
Keuntungan yangdiperoleh diinvestasikanlagi ke proyek lain dengan i* = 10%:
Reinv est
Menghasilkan total pendapatan di akhir umur proyek sebesar:
FA = 20jt(F/P,10%,6) +70jt(F/P,10%,5) = 20jt(1,772) + 70jt(1,611) = Rp 148,21 juta
Initial +
Reinv est
Growth ROR alternatifA (GRORA)dihitungdengancoba-coba:
NPVA @ 15% = - 50jt + 148,21jt (P/F,15%,7) = - 50jt + 148,21jt (0,376) = Rp 5,73 jt
NPVA @ 20% = - 50jt + 148,21jt (P/F,20%,7) = - 50jt + 148,21jt (0,279) = Rp -8,65 jt
Selanjutnyadicari nilai interpolasinya:
(GRORA-15%)/(5,73-0) = (20%-15%)/(5,73-(-8,65))
GRORA = 15% + (20%-15%)((5,73/(5,73+8,65)) = 17%
Growth ROR mesin B:
Initial
Keuntungan yangdiperoleh diinvestasikanlagi ke proyek lain dengan i* = 10%:
Reinv est
0
GROR =?15%
-8,65
5,73
20%
L = 50jt
20jt20jt-50jt
20 1
L = 30jt
10jt10jt10jt10jt10jt-30jt
3 420 1 5
------50jt
3 420 1 5
F= 148,21jt
--
76
F = ?
--
76
---
50jt
20jt20jt
3 420 1 5
6
F = ?
--
7
30jt
10jt10jt10jt10jt10jt
3 420 1 5
Yuli Kusumawati, Catatan Kuliah Ekonomi Mineral - 31
Menghasilkan total pendapatan di akhir umur proyeksebesar:
FB = [10jt(F/A,10%,5) + 30jt](F/P,10%,2) = 10jt(6,105) +30jt(0,1,21) = Rp 110,17 juta
Initial +
Reinv est
Growth ROR alternatifB (GRORB)dihitungdengan coba-coba:
NPVB @ 20% = - 30jt + 110,17jt (P/F,20%,7) = - 30jt + 110,17jt (0,279) = Rp 0,74 jt
NPVB @ 25% = - 30jt + 110,17jt (P/F,25%,7) = - 30jt + 110,17jt (0,210) = Rp -6,86 jt
Selanjutnyadicari nilai interpolasinya:
(GRORB- 20%)/(0,74-0) = (25%-20%)/(0,74-(-6,86))
GRORB = 20% + (25%-20%)((0,74/(0,74+6,86)) = 20,4%
Growth ROR mesin C:
Initial
Keuntungan yangdiperoleh diinvestasikanlagi ke proyek lain dengan i* = 10%:
Reinv est
Menghasilkan total pendapatan di akhir umur proyeksebesar:
FC = 5jt(F/A,10%,7) + 20jt = 5jt(9,487) + 20jt = Rp 67,43 juta
Initial +
Reinv est
Growth ROR alternatifC (GRORC)dihitungdengan coba-coba:
NPVC @ 20% = - 20jt + 67,43jt (P/F,20%,7) = - 20jt + 67,43jt (0,279) = Rp -1,19 jt
NPVC @ 15% = - 20jt + 67,43jt (P/F,15%,7) = - 20jt + 67,43jt (0,376) = Rp 5,35 jt
Selanjutnyadicari nilai interpolasinya:
(GRORC-15%)/(5,35-0) = (20%-15%)/(5,35-(-1,19))
GRORC = 15% + (20%-15%)((5,35/(5,35+1,19)) = 19,1%
Berdasarkan growth RORdiperoleh rankingsebagai berikut:
Mesin B (GRORB=20,4%) palingmenguntungkan, disusul mesin C (GRORC=19,1%), terakhir mesin A (GRORA=17%).
Hasil ini konsisten dengan rankingmetode PVR dan BCR.
0
GROR =?20%
-6,86
0,74
25%
0
GROR =?15%
-1,19
5,35
20%
5jt5jt5jt5jt5jt-20jt
3 420 1 5
L = 20jt
5jt5jt
76
-20jt
3 420 1 5
F = 67,43jt
76
5jt5jt5jt5jt5jt
3 420 1 5
F = ?
20 jt
5jt5jt
76
------30jt
3 420 1 5
F=110,17jt
--
76
Yuli Kusumawati, Catatan Kuliah Ekonomi Mineral - 32
7. DEPRESIASI, DEPLESI, AMORTISASI
7.1. Depresiasi
Depresiasi adalah penurunan dalam nilaifisik properti seiring dengan waktu danpenggunaannya. Dalamkonsep
akuntansi, depresiasi adalahpemotongan tahunan terhadappendapatan sebelum pajak sehinggapengaruh
waktu dan penggunaan atasnilai aset dapat terwakili dalam laporan keuangan suatuperusahaan.
Faktor-faktor penyebabdepresiasi:
 Keausan atau penurunan performa mesin/alat akibat pemakaian (physical degradation).
 Perkembangan teknologi yang menghasilkan mesin/alat yang lebih efisian dan ekonomis (functional
depreciation).
Syarat-syarataset yangdapat didepresiasi:
 Digunakan dalam usahauntukmenghasilkanpendapatan.
 Mempunyai umur manfaattertentu.
 Umur aset harus lebih dari satu tahun.
 Nilai aset turun karena pemakaian.
Komponen dalam perhitungandepresiasi:
 Nilai awal, yaitu biaya awal untuk mendapatkan aset (harga beli ditambah pajak), termasuk biaya
transportasi danbiayalain sampai aset tersebutdapatdigunakansesuai fungsinya.
 Nilai sisa, adalah perkiraan nilai aset pada akhir umur manfaatnya, merupakan harga jual suatu aset jika
tidaklagi digunakan untukproses produksi.
 Nilai buku, adalah nilai aset sesuai dengan laporan akuntansi yang mewakili jumlah modal yang masih
diinvestasikan pada aset tersebut. Sama dengan nilai awal (termasuk segala penyesuaian) dikurangi
dengan pengurangankarena depresiasi.
 Tingkat/laju depresiasi, adalah prosentase depresiasi per tahun.
 Masa manfaat, adalah perkiraan periode waktu pemakaian aset (properti) dalam kegiatan
produktif/menghasilkanpendapatan.
Tabel 7.1. Masa manfaat dan tarifpenyusutanhartaberwujud menurut UU No. 36 Tahun 2008
Tujuan:
 Memahami konsep depresiasi, deplesi, danamortisasi.
 Mampu menghitungdepresiasi, deplesi, danamortisasi dengan metode yangsesuai.
 Memahami pengaruhdepresi, deplesi, danamortisasi terhadappajak.
Yuli Kusumawati, Catatan Kuliah Ekonomi Mineral - 33
Keterangan: Ayat (1) menggunakan metode depresiasi straigth line, ayat (2) menggunakan metode depresiasi
double decliningbalance.
Metode perhitungandepresiasiantara lain:
 Metode garis lurus (sraigth line)
 Metode keseimbanganmenurunganda (double decliningbalance)
 Metode jumlah digit tahun (sum ofyear digit)
 Metode sinkingfund
 Metode unit produksi (productionunit)
7.1.1. Straigth line (SL) depreciation
Metode ini mengalokasikan depresiasi aset secara konstan setiaptahunnya selama umur manfaatnya.Dengan
metode ini nilai buku (BV) pada akhir umur aset sama dengan nilai sisa (S).
Tingkat/laju depresiasi per tahun: d = 100% / N
Besarnya depresiasi dari tahun pertama sampai tahunke–t adalah tetap:
Sedangkan nilai buku(bookv alue) pada akhir tahun ke-t adalah:
Dimana:
Dt = depresiasi pada tahunke-t (t = 1,2,...,n)
BVt = book v alue (nilai buku) aset
d = tingkat/laju depresiasi tahunan
P = nilai awal aset
S = salv age value (nilai sisa) aset
N = umur aset (dalam satuan tahun)
t = akhir tahun depresiasi yangditinjau
Contoh 7-1:
Perusahaan ABC membeli satu unit mesin seharga $ 10,000. Diharapkanmasa pakai mesin tersebut selama 5
tahun.Pada akhir umur ekonomisnya mesin tersebut diperkirakan memiliki nilai sisa $ 1,000.
Hitung nilai depresiasi dengan metode straigth line dan nilai buku pada akhir tahun ke-1 sampai tahun ke-5 dari
mesin tersebut.
Jawab:
P = $ 10,000
S = $ 1,000
N = 5 tahun
t = akhir tahun depresiasi yangditinjau
Tingkat/laju depresiasi per tahun:
d = 100%/5 = 20%
Nilai depresiasi tiap tahun:
Dt = (P-S)/N
= ($ 10,000 – $ 1,000)/5= $ 1,800
Nilai bukuakhir tahunke-1 sampai ke-5:
BV1 = $ 10,000 – $ 1,800 = $ 8,200
Yuli Kusumawati, Catatan Kuliah Ekonomi Mineral - 34
BV2 = $ 8,200 – $ 1,800 = $ 6,400
BV3 = $ 6,400 – $ 1,800 = $ 4,600
BV4 = $ 4,600 – $ 1,800 = $ 2,800
BV5 = $ 2,800 – $ 1,800 = $ 1,000
Besarnya depresiasi dan nilai bukudisajikan padatabel berikut:
7.1.2. Double declining balance (DDB) depreciation
Metode ini memakai tingkat/laju depresiasi duakali (double) dari metode straigthline terhadapsisa nilai buku.
Tingkat/laju depresiasi tahunan adalah: d = 200%/N
Besarnya depresiasi tahunke–t adalah:
Sedangkan nilai buku(bookv alue) pada akhir tahun ke-t adalah:
Dimana:
Dt = depresiasi pada tahunke-t (t = 1,2,...,n)
BVt = book v alue (nilai buku) aset
d = tingkat/laju depresiasi tahunan
P = nilai awal aset
S = salv age value (nilai sisa) aset
N = umur aset (dalam satuan tahun)
t = akhir tahun depresiasi yangditinjau
Contoh 7-2:
Perusahaan ABC membeli satu unit mesin seharga $ 10,000. Diharapkan masa pakai mesin tersebut selama 5
tahun. Pada akhir umur ekonomisnya mesin tersebut diperkirakan memiliki nilai sisa $ 1,000.
Hitung nilai depresiasi dengan metode declining balance dan nilai buku pada akhir tahun ke-1 sampai tahun ke-
5 dari mesin tersebut.
Jawab:
P = $ 10,000
S = $ 1,000
N = 5 tahun
Tingkat/laju depresiasi per tahun:
d = 200%/5 = 40%
Nilai depresiasi tahunke-1 sampai ke-5:
Yuli Kusumawati, Catatan Kuliah Ekonomi Mineral - 35
D1 = 0,4 ($ 10,000 – $ 0) = $ 4.000
D2 = 0,4 ($ 10,000 – $ 4.000) = $ 2.400
D3 = 0,4 ($ 10,000 – $ 6.400) = $ 1,440
D4 = 0,4 ($ 10,000 – $ 7.840) = $ 864
D5 = 0,4 ($ 10,000 – $ 8.704) = $ 518,4
Nilai buku akhir tahun ke-1 sampai ke-5:
BV1 = $ 10,000 – $ 4.000 = $ 6.000
BV2 = $ 6.000 – $ 2.400 = $ 3.600
BV3 = $ 3.600 – $ 1.440 = $ 2.160
BV4 = $ 2.160 – $ 864 = $ 1.296
BV5 = $ 1.296 – $ 518,4 = $ 777,6
Besarnya depresiasi dan nilai bukudisajikan padatabel berikut:
7.1.3. Sum of years digits (SOYD) depreciation
Metode ini mengalokasikan beban depresiasi ast berdasarkan jumlah digit tahun pemakaiannya (SOYD),
sehingga alokasi biaya depresiasi tidak konstan. Dengan metode ini alokasi biaya depresiasi dibebankan lebih
besar pada awal-awal periode.
Jumlah digit tahun adalah: 𝐒𝐎𝐘𝐃 =
𝐧(𝐧+𝟏)
𝟐
Besarnya depresiasi tahunke–t adalah: 𝐃𝐭 =
𝐧−(𝐭−𝟏)
𝐒𝐎𝐘𝐃
(𝐏 − 𝐒)
Sedangkan nilai buku(bookv alue) pada akhir tahun ke-t adalah:
Dimana:
SOYD = jumlah digit tahun
Dt = depresiasi pada tahunke-t (t = 1,2,...,n)
BVt = book v alue (nilai buku) aset
P = nilai awal aset
S = salv age value (nilai sisa) aset
N = umur aset (dalam satuan tahun)
t = akhir tahun depresiasi yangditinjau
Contoh 7-3:
Yuli Kusumawati, Catatan Kuliah Ekonomi Mineral - 36
Perusahaan ABC membeli satu unit mesin seharga $ 10,000. Diharapkan masa pakai mesin tersebut selama 5
tahun. Pada akhir umur ekonomisnya mesin tersebut diperkirakan memiliki nilai sisa $ 1,000.
Hitung nilai depresiasi dengan metode sum of years digits (SOYD) dan nilai buku pada akhir tahun ke-1 sampai
tahun ke-5 dari mesin tersebut.
Jawab:
P = $ 10,000
S = $ 1,000
N = 5 tahun
Jumlah digit tahun:
SOYD =
n(n + 1)
2
=
5(5 + 1)
2
= 15
Nilai depresiasi tiap tahun:
Dt =
n − (t− 1)
SOYD
(P − S)
D1 =
5 − (1 − 1)
15
(10.000− 1.000) =
5
15
(9.000)= 3.000
D2 =
5 − (2 − 1)
15
(10.000− 1.000) =
4
15
(9.000)= 2.400
D3 =
5 − (3 − 1)
15
(10.000− 1.000) =
3
15
(9.000)= 1.800
D4 =
5 − (4 − 1)
15
(10.000− 1.000) =
2
15
(9.000)= 1.200
D5 =
5 − (5 − 1)
15
(10.000− 1.000) =
1
15
(9.000)= 600
Nilai buku padaakhir tahunke-1 sampai ke-5:
BV1 = $ 10,000 – $ 1,800 = $ 8,200
BV2 = $ 8,200 – $ 1,800 = $ 6,400
BV3 = $ 6,400 – $ 1,800 = $ 4,600
BV4 = $ 4,600 – $ 1,800 = $ 2,800
BV5 = $ 2,800 – $ 1,800 = $ 1,000
Besarnya depresiasi dan nilai buku disajikan padatabel berikut:
7.1.4. Sinking fund depreciation
Metode depresiasi sinking fund membebankan depresiasi tahunan secara seragam dengan cara
mempertimbangkan faktor suku bunga, sehingga nilai aset yang akan didepresiasi dikalikan dengan equal
payment series sinking fund factor.
Yuli Kusumawati, Catatan Kuliah Ekonomi Mineral - 37
Besarnya depresiasi tahunke–t adalah:
Sedangkan nilai buku(bookv alue) pada akhir tahun ke-t adalah:
Dimana:
Dt = depresiasi pada tahunke-t (t = 1,2,...,n)
BVt = book v alue (nilai buku) aset
P = nilai awal aset
S = salv age value (nilai sisa) aset
N = umur aset (dalam satuan tahun)
t = akhir tahun depresiasi yangditinjau
Contoh 7-4:
Perusahaan ABC membeli satu unit mesin seharga $ 10,000. Diharapkan masa pakai mesin tersebut selama 5
tahun. Pada akhir umur ekonomisnya mesin tersebut diperkirakan memiliki nilai sisa $ 1,000. Hitung nilai depresiasi
dengan metode sinkingfund dan nilai buku padaakhir tahun ke-1 sampai tahun ke-5 mesin tersebut, jika i =20%.
Jawab:
P = $ 10,000
S = $ 1,000
N = 5 tahun
i = 20%
Nilai depresiasi tiap tahun:
D1 = ($ 10,000 – $ 1.000)(A/F,20%,5) = $ 9.000 (0,1344) = $ 1.209,6
D2 = ($ 10,000 – $ 1.000)(A/F,20%,5) = $ 9.000 (0,1344) = $ 1.209,6
D3 = ($ 10,000 – $ 1.000)(A/F,20%,5) = $ 9.000 (0,1344) = $ 1.209,6
D4 = ($ 10,000 – $ 1.000)(A/F,20%,5) = $ 9.000 (0,1344) = $ 1.209,6
D5 = ($ 10,000 – $ 1.000)(A/F,20%,5) = $ 9.000 (0,1344) = $ 1.209,6
Nilai buku akhir tahun ke-1 sampai ke-5:
BV1 = $ 10,000 – $ 1.209,6 = $ 8,790,4
BV2 = $ 6.000 – $ 1.209,6 = $ 7.580,8
BV3 = $ 3.600 – $ 1.209,6 = $ 6.371,2
BV4 = $ 2.160 – $ 1.209,6 = $ 5.161,6
BV5 = $ 1.296 – $ 1.209,6 = $ 3.952,0
Besarnya depresiasi dan nilai bukudisajikan padatabel berikut:
Yuli Kusumawati, Catatan Kuliah Ekonomi Mineral - 38
Dt = (Produksi/Ʃ Produksi) (P-S)
BVt = P(t-1) - Dt
7.1.5. Production unit depreciation
Metode ini dipakai untuk peralatan yang menghasilkan layanan atau produksi berdasarkan kebutuhan per unit
produksi (misalnya, truk, excavator, mesin pompaair) dan tidaktergantungpadafungsi waktu.
Besarnya depresiasi tahunke–t adalah:
Sedangkan nilai buku(bookv alue) pada tahunke-t adalah:
Dimana:
Dt = depresiasi pada tahunke-t (t = 1,2,...,n)
BVt = book v alue (nilai buku) aset
P = nilai awal aset
S = salv age value (nilai sisa) aset
t = akhir tahun depresiasi yangditinjau
Contoh 7-5:
Perusahaan tambang pasir PT Galunggung membeli satu unit excavator seharga Rp 700 juta. Berdasarkan
spesifikasi teknis excavator tersebut mampu menambang pasir sebanyak 50.000 m3 dan nilai sisa pada akhir
umur ekonomisnya diperkirakan Rp150 juta.
Perencanaan produksi pasir perusahaanABC adalah sebagai berikut:
Tahun ke- Produksi (m3)
1 4.000
2 6.000
3 10.000
4 10.000
5 15.000
6 5.000
Ʃ 50.000
Hitungnilai depresiasi tahunanexcavator tersebutdenganmetode unit produksi.
Jawab:
P = Rp 700 juta
S = Rp 150 juta
Nilai depresiasi tahunke-1 sampai tahun ke-6:
Dt = (Produksi/Ʃproduksi)(P-S)
D1 = (4.000/50.000)(Rp 700 juta – Rp 150 juta) = Rp 44 juta
D2 = (6.000/50.000)(Rp 700 juta – Rp 150 juta) = Rp 66 juta
D3 = (10.000/50.000)(Rp 700 juta – Rp 150 juta) = Rp 110 juta
D4 = (10.000/50.000)(Rp 700 juta – Rp 150 juta) = Rp 110 juta
D5 = (15.000/50.000)(Rp 700 juta – Rp 150 juta) = Rp 165 juta
D6 = (5.000/50.000)(Rp 700 juta – Rp 150 juta) = Rp 55 juta
Nilai buku tahun ke-1 sampai ke-6:
BVt = P - Dt
BV1 = Rp 700 juta – Rp 44 juta = Rp 656 juta
BV2 = Rp 656 juta – Rp 66 juta = Rp 590 juta
Yuli Kusumawati, Catatan Kuliah Ekonomi Mineral - 39
BV3 = Rp 590 juta – Rp 110 juta = Rp 480 juta
BV4 = Rp 480 juta – Rp 110 juta = Rp 370 juta
BV5 = Rp 370 juta – Rp 165 juta = Rp 205 juta
BV6 = Rp 205 juta – Rp 55 juta = Rp 150 juta
Hasil hitungan depresiasi dan nilai bukudisajikan padatabel berikut:
Tahun ke- Produksi (m3) Depresiasi Nilai buku
1 4.000 Rp 44 juta Rp 656 juta
2 6.000 Rp 66 juta Rp 590 juta
3 10.000 Rp 110 juta Rp 480 juta
4 10.000 Rp 110 juta Rp 370 juta
5 15.000 Rp 165 juta Rp 205 juta
6 5.000 Rp 55 juta Rp 150 juta
Ʃ 50.000
7.2. Deplesi
Sumber daya alam juga termasuk aktiva tetap. Seperti halnya aktiva tetap lainnya, sumber daya alam setelah
dieksploitasi kemudian diolah terus-menerus maka lama kelamaan sumber daya alam akan habis. Oleh karena
itu perlu ada pembebananbiayayangmirip denganpersediaan.
Deplesi adalah suatu bentuk potongan pajak atas berkurangnya sumberdaya alam (cadangan mineral atau
kekayaan hutan) yang diberikan selama umur ekonomis cadangan tersebut. Perhitungan deplesi dilakukan
setiap tahun.
Perbedaandepresiasi dan deplesi adalahsebagai berikut:
 Deplesi merupakan penyusutan terhadap pengurangan kuantitatif yang terjadi dalam sumber daya alam,
sedangkan depresiasi merupakan penyusutan terhadap pengurangan service (manfaat ekonomi) yang
terjadi dalam aktivatetap.
 Deplesi digunakan untuk aktiva tetap yang tidak dapat diganti langsung dengan aktiva yang sama jika
sudah habis, sedangkan depresiasi digunakan untuk aktiva tetap yang pada umumnya dapat diganti jika
sudah habis.
 Deplesi adalah pengakuan terhadap perubahan langsung dari suatu sumber alam menjadi barang yang
dapat dijual, sedangkan depresiasi adalah alokasi harga perolehan ke pendapatan periode yang
bersangkutan untuk suatu serv ice yang dihasilkan (kecuali dalam perusahaan di mana depresiasi dihitung
berdasar hasil produksi).
Dalam menghitungbesarnyadeplesi perlu memperhatikan faktor-faktor:
 Harga perolehan aktiva,
 Taksiran nilai sisa jika sumber alam sudah selesai di eksploitasi
 Taksiran hasil yangsecara ekonomis dapat dieksploitasi.
Ada dua metode perhitungan deplesi untukmineral, minyak, dangas, yaitu:
a. Cost depletion
b. Percentage depletion
Sedangkan untukmenghitungdeplesi hasil hutanbiasanyamenggunakanmetode cost depletion.
Yuli Kusumawati, Catatan Kuliah Ekonomi Mineral - 40
Prosedur menentukan metodedeplesi:
7.2.1. Cost depletion
Basis perhitungancost depletion untukproyekminyakdan pertambanganadalah:
 Hakpenguasaan.
 Bonus penyewaan.
 Biaya surv ey geologi/geofisika.
 Biaya penaksiran danlegalitas.
 Biaya-biayasejenis yangdiketahui dengan pasti.
Biaya eksplorasi penambangan ataupun biaya pengeboran minyak yang tidak nyata biasanya jarang
digunakan sebagai basis perhitungan cost depletion.
𝐂𝐨𝐬𝐭 𝐝𝐞𝐩𝐥𝐞𝐭𝐢𝐨𝐧 = 𝐁𝐚𝐬𝐢𝐬 𝐩𝐞𝐧𝐲𝐞𝐬𝐮𝐚𝐢𝐚𝐧 𝐱
𝐌𝐢𝐧𝐞𝐫𝐚𝐥 𝐲𝐚𝐧𝐠 𝐝𝐢𝐚𝐦𝐛𝐢𝐥 𝐝𝐚𝐧 𝐝𝐢𝐣𝐮𝐚𝐥 𝐬𝐞𝐥𝐚𝐦𝐚 𝐭𝐚𝐡𝐮𝐧 𝐛𝐞𝐫𝐣𝐚𝐥𝐚𝐧
𝐌𝐢𝐧𝐞𝐫𝐚𝐥 𝐲𝐚𝐧𝐠 𝐝𝐢𝐭𝐞𝐦𝐮𝐤𝐚𝐧 𝐩𝐚𝐝𝐚 𝐚𝐰𝐚𝐥 𝐭𝐚𝐡𝐮𝐧
Dimana:
Basis penyesuaian = Basis biaya ± Penyesuaian – Deplesi kumulatif.
Contoh 7-6:
Suatu perusahaan mempunyai lahan pertambangan emas dengan cadangan diperkirakan 1.000.000ounces.
Biaya untuk mendapatkan IUP yang dikeluarkan pada tahun ke-0 adalah $150.000. Penambanganemas
diproyeksikan mulai tahun ke-1 dengan volume tetap sebesar 50.000 ounces/tahun.Hitunglah cost depletion
tahun ke-1 dan tahun ke-2.
Jawab:
Cost depletion tahun ke-1 = ($150.000)(50.000 ounces / 1.000.000 ounces) = $7.500
Cost depletion tahun ke-2 = ($150.000-$7.500)(50.000ounces / 950.000 ounces) = $7.500
7.2.2. Percentage depletion
Percentage depletion adalah persentase tertentu dari pendapatan bersih (pendapatan kotor setelah dikurangi
royalti) dari penjualanmineral ataumigas yangdiambil dari lahan pertambangan selama tahunpajak.
Persentase deplesi yang diijinkan biasanya tergantung pada jenis bahan tambang dan kapasitas produksi
perusahaan.
Contoh 7-7:
Suatu perusahaan mempunyai lahan pertambangan emas dengan cadangan diperkirakan 1.000.000 ounces.
Biaya untuk mendapatkan IUP yang dikeluarkan pada tahun ke-0 adalah $150.000. Penambanganemas
diproyeksikan mulai tahun ke-1 dengan volume tetap sebesar 50.000 ounces/tahun dengan harga jual
$29/ounce setelah royalti.Diasumsikan tingkat deplesi untuk emasadalah 15% dari pendapatan bersih.Hitunglah
percentage depletion tahun ke-1 dan tahunke-2.
% deplesi
Batas % deplesi:
50% untukmineral
100% untukminyakdan gas
Hasil yang lebih kecil
merupakan percentage
depletion yangdiijinkan
Cost depletion
Pilih metode deplesi yang
menghasilkan pengurangan
pajaklebih besar.
Yuli Kusumawati, Catatan Kuliah Ekonomi Mineral - 41
Jawab:
Percentage depletion tahun ke-1= 15% (50.000 ounce x $29) = $217.000
Percentage depletion tahun ke-2= 15% (50.000 ounce x $29) = $217.000
7.2.3. Memilih metode deplesi
Contoh 7-8:
PT Agung Linuwih mempunyai lahan emas dengan biaya hak penguasaan $150.000. Cadangan emas yang
ditemukan diperkirakan satu juta ounces. Produksi diproyeksikan dimulai tahun ke-1 sebesar 50.000 ounces/tahun
dan dijual dengan harga $29/ouces setelah royalti. Biaya operasi dan overhead tahun ini $180.000, dan
depresiasi aset adalah $120.000/tahun. Diasumsikan tingkat produksi, biaya operasi, dan harga jual akan sama
pada tahun depan. Tingkat deplesi untuk emas adalah 15% dengan batas persentase deplesi untukperusahaan
tersebut adalah 100%, pajakpertambangan $30.000/tahun, danpajakpenghasilan40%.
Jika perusahaan boleh memilih metode deplesi untuk mengurangi pajaknya, bandingkan besarnya percentage
depletion dan cost depletiontahun ke-1 dantahun ke-2.
Jawab:
Tahun ke-1
Pendapatanbersih (50.000 ounces x $29) $1.450.000
- Biaya operasi -180.000
- Pajakpertambangan -30.000
- Depresiasi -120.000
Penghasilan kena pajaksebelum deplesi $1.120.000
- Batas persentase deplesi (1,0)($1.200.000) 1.120.000
- Persentage depletion(0,15)($1.450.000) -217.500*)
- Cost depletion 7.500**)
Penghasilan kena pajak $902.500
- Pajak40% -361.000
Penghasilan bersih (keuntungan) 541.500
+ Depresiasi 120.000
+ Deplesi yangdiambil 217.500
Cash flow penjualantahun ke-1 $879.000
Keterangan:
*)Karena hasil percentage depletion 15% adalah $217.500 lebih kecil daripada batas 100% percetage depletion,
maka diperbolehkan menggunakan metode persentase deplesi. Pada tahun ke-2 jika pendapatan dan
pengurangan diasumsikan sama seperti tahun ke-1, maka batas percentage depletion juga sama, sehingga
diperbolehkan menggunakanmetode persentase deplesi.
**)Cost depletion tahun ke-1 = ($150.000)(50.000 ounces / 1.000.000 ounces) = $7.500, lebih kecil daripada
percentage depletion sehinggaperusahaanmemilih percentage depletion untukpengurangandeplesinya.
Basis biaya untuk perhitungan cost depletion tahun ke-2 harus disesuaikan untuk pengurangan aktual deplesi
yangdiambil.
Cost depletion tahun ke-2 = ($150.000-$217.500)(50.000/950.000) < 0.
Sehingga pada tahun ke-2 metode cost depletion juga tidak digunakan untuk pengurangan deplesi. Dengan
kata lain percentage depletion digunakanlagi untukmenghitungdeplesi padatahun ke-2.
7.3. Amortisasi
Yuli Kusumawati, Catatan Kuliah Ekonomi Mineral - 42
Amortisasi adalah proses penyusutan untuk aset tidak berwujud (intangible) dan digunakan untuk pengurangan
pajak. Menurut pajak, harga perolehan aset harus diamortisasi jika aset tersebut digunakan untuk mendapatkan,
menagih, dan memelihara penghasilan dan memiliki masa manfaat lebih dari satu tahun.
Harga aset atau biaya yang bisa diamortisasi misalnya biaya untuk pengembangan (development) tambang,
intangible cost padapengeboran migas, hakpatent,franchaise, dsb.
Cara mengitung amortisasi sama seperti perhitungan depresiasi garis lurus ataupun declining balance, hanya
saja amortisasi diberikan dengan tingkat (rate) tertentu dan selama periode tertentu, tergantung kebijakan
pajak masing-masing negara. Di Indonesia tarif amortisasi ditetapkan dalam pasal 11 A (2) UU PajakPenghasilan
(PPh) No.36 Th 2008, seperti yangdisajikan padaTabel 7.1.
Tabel 7.1. TarifAmortisasi berdasarkan UU No. 36 Tahun2008
Kelompokharta
Masa manfaat
(tahun)
Tarif amortisasi
Garis lurus Saldo menurun
Kelompok1 4 25% 50%
Kelompok2 8 12,5% 25%
Kelompok3 16 6,25% 12,5%
Kelompok4 20 5% 10%
Jika biaya pengembangan seluruhnya (100%) digunakan untuk pengurangan pendapatan sebelum pajak,
maka tidakada amortisasi.
Contoh 7-9:
Seorang investor akan mengembangkan suatu pertambangan. Biaya pengembangan sebesar $1 juta yang
diadakan pada bulan juli (bulan ke-7) tahun ini (tahun ke-0). Jika amortisasi yang diijinkan adalah 30% dari biaya
pengembangan dan periode amortisasi di atas 60 bulan, hitunglahpenguranganpajakyangdiijinkan.
Jawab:
Basis amortisasi = 30% x $1.000.000 = $300.000
Jangka waktu dari bulan ke-7 sampai bulan ke-12 tahun ke-0 adalah 6 bulan, sehingga amortisasi yang diambil
pada tahun ke-0 adalah6/60, tahun ke-1 sampai ke-4 adalah 12/60, dantahun ke-5 adalah6/60.
Tahun ke- Penguranganamortisasi
0 (6/60) ($300.000) = $30.000
1-4 (12/60) ($300.000) = $60.000
5 (6/60) ($300.000) = $30.000
Yuli Kusumawati, Catatan Kuliah Ekonomi Mineral - 43
8. PENGARUH PAJAK DALAM ANALISIS LIFE CYCLE COSTING
8.1. Pengertian pajak
Pajak(tax) adalahsalahsatu sumber keuanganpemerintah yangberasal dari rakyat.
Kegunaan pajakantaralain:
 Membiayai kegiatan mengelola negara (membayar gaji PNS, subsidi, pembangunan fasilitas umum, dll).
 Pemerataan pendapatan (orang yang memiliki pendapatan lebih besar akan dikenakan pajak lebih
banyakdibandingkanorangdenganpenghasilan lebih kecil).
 Mengurangi pemakaianbarangyangsecara umum tidakbermanfaat (misalnya: miras, rokok).
 Memproteksi industri nasional, industri kecil, industri ramah lingkungan.
Beberapa jenis pajakyangdikenakan padaperusahaan antaralain:
 Pajak pendapatan, yaitu pajak yang dipungut dari pendapatan usaha atau perorangan yang besarnya
dihitungberdasarkan persentase tertentudari pendapatan bersih.
 Pajakkekayaan, yaitupajakatas kepemilikan tanah, bangunan, kendaraan, mesin/peralatan, dsb.
 Pajak penjualan, yaitu pajak yang dipungut atas penjualan barang atau pemberian pelayanan dan tidak
ada kaitannya denganpendapatan bersih ataukeuntunganperusahaan.
Dalam industri pertambangan dikenal adanya pungutan yang bisa digunakan sebagai
pengurang/potongan(deduction) pendapatan sebelum pajak, antaralain:
 Royalti, yaitu suatu bentuk kompensasi atas diberikannya hak istimewa untuk mengembangkan atau
memproduksi mineral atau migas. Kompensasi berupa pembagian keuntungan dari nilai mineral yang
diekstraksi.
 Severance tax (mining tax), yaitu pajak negara atas pengambilan sumberdaya alam yang perhitungannya
berdasarkan nilai/jumlahproduksi aktual.
 Property tax (ad valorem tax), yaitu retribusi yang dipungut oleh otoritas perpajakan atas kekayaan baik
yangberupa barangbergerakmaupuntidakbergerak.
Di samping itu potongan pajak juga berasal dari penyusutan atau penurunan nilai sebuah aktiva yang memilki
umur manfaat. Penyusutan adalah biaya non-kas yang berpengaruh terhadap pajak pendapatan, karena
merupakan potongan tahunan terhadap pendapatan sebelum pajak sehingga pengaruh waktu dan
penggunaanatas nilai aset dapatterwakili dalam laporan keuangansuatuperusahaan.
Jenis-jenis penyusutandalam industri pertambanganyaitu:
a. Depresiasi digunakan untuk aktivatetap.
b. Amortisasi digunakan untuk aktiva tidakberwujud.
c. Deplesi digunakan untukaktiva sumber dayaAlam.
Dari sudut kepentingan pajak investor, metode tercepat dari pengurangan biaya adalah untuk mengeluarkan
atau mengurangi pajak secara penuh pada tahun dikeluarkan. Hal ini disebabkan semakin cepat diperoleh
pemotongan pajak, semakin cepat diperoleh keuntungan dari pemotongan tersebut sehingga akan
memperbaiki ekonomi proyek. Akan tetapi hukum pajakseringkali tidakmengijinkan hal yangdemikian.
Tujuan:
 Memahami konsep pajak.
 Mampu menyusun cashflow setelah pajak
 Mampu memahami pengaruhpajakdalam analisis life cycle costing
Yuli Kusumawati, Catatan Kuliah Ekonomi Mineral - 44
8.2. Cashflow setelah pajak
Contoh 8-1:
PT Agung Linuwih bermaksud menguasai dan mengembangkan lahan pertambangan yang diyakini
mengandung 500.000 ounces emas. Biaya hak penguasaan tambang tersebut $900.000, biaya pengembangan
$1.200.000, biaya pembelian peralatan tambang $1.000.000, dan biaya modal kerja $ 300.000, semuanya
dikeluarkan pada awal tahun ke-0. Umur pemakaian peralatan adalah 5 tahun, dan depresiasi peralatan
dihitungdengan metode straigth line dengannilai sisa padaakhir umur pemakaian peralatanadalahnol.
Produksi emas diproyeksikan dimulai tahun ke-1 sebesar 100.000 ounces/tahun selama 5 tahun.Harga jual emas
pada tahun ke-1 adalah $30/ouces, dan diperkirakan mengalami kenaikan 10%/tahun. Biaya operasi tahun ke-1
adalah $1.000.000, dan mengalami kenaikan 8%/tahun.Diasumsikan tingkat produksi, biaya operasi, dan harga
jual akan sama padatahun depan.
Royalti yang harus dibayarkan adalah 15% dari pendapatan (revenue). Pajak pertambangan $30.000/tahun,
dan pajak penghasilan 30%. Perusahaan boleh memilih metode cost deplesi ataupun persentase deplesi untuk
mengurangi pajaknyadengan tingkat deplesi adalah 15% dan batas persentase deplesi untuk perusahaan
tersebut adalah 50%.
Hitunglah:
a. Cashflow sebelum pajak
b. Cashflow setelah pajak jika 100% development cost digunakan untuk pengurang pajakdan deplesi dihitung
dengan metode cost depletion.
c. Cashflow setelah pajak jika 100% development cost digunakan untukpengurang pajakdan deplesi dihitung
dengan metode percentage depletion.
d. Cashlow setelah pajak jika tingkat amortisasi 30% selama 5 tahun dan deplesi dihitung dengan metode cost
depletion.
e. Cashlow setelah pajak jika tingkat amortisasi 30% selama 5 tahun dan deplesi dihitung dengan metode
percentage depletion.
Diasumsikan negatiftaxable income sebagai saving (expensed against other income).
Jawab:
Tahun ke- 0 1 2 3 4 5
Produksi (ounces/th) - 100.000 100.000 100.000 100.000 100.000
Harga jual ($/ounce) - 30 33 36,30 39,93 43,92
Biaya produksi ($/th) - 1.000.000 1.080.000 1.166.000 1.260.000 1.360.000
a. Cashflow sebelum pajak (x$1000):
Tahun ke- 0 1 2 3 4 5
Rev enue 0 3.000 3.300 3.630 3.993 4.392
-Royalti (15%) 0 -450 -495 -545 -599 -659
Net rev enue 0 2.550 2.805 3.086 3.394 3.733
-Operatingcost 0 -1.000 -1.080 -1.166 -1.260 -1.360
Net income 0 1.550 1.725 1.919 2.134 2.373
+Working capital return 0 0 0 0 0 300
-Capital cost1) -3.400 0 0 0 0 0
CASH FLOW -3.400 1.550 1.725 1.919 2.134 2.673
Keterangan:
1)Capital cost = working capital+acquisitionrigths +depreciable equipment+development.
= $300 + $900 + $1.000 + $1.200 = $3.400
Yuli Kusumawati, Catatan Kuliah Ekonomi Mineral - 45
b. Cashflow setelah pajak jika 100% development cost digunakan untukpengurang pajakdan deplesi dihitung
dengan metode cost depletion.
Tahun ke- 0 1 2 3 4 5
Rev enue 0 3.000 3.300 3.630 3.993 4.392
-Royalty (15%) 0 -450 -495 -545 -599 -659
Net rev enue 0 2.550 2.805 3.086 3.394 3.733
-Operatingcost 0 -1.000 -1.080 -1.166 -1.260 -1.360
-Miningtax 0 -30 -30 -30 -30 -30
-Dev elopment cost -1.200 0 0 0 0 0
-Depreciation1) 0 -200 -200 -200 -200 -200
-Cost depletion2) 0 -180 -180 -180 -180 -180
Taxable income -1.200 1.140 1.315 1.510 1.724 1.963
-Tax (30%)3) 360 -342 -394,5 -453 -517,2 -588,9
Net income -840 798 920,5 1.057 1.206,8 1.374,1
+Depreciation 0 200 200 200 200 200
+Depletion 0 180 180 180 180 180
+Working capital return 0 0 0 0 0 300
-Capital cost4) -2.200 0 0 0 0 0
CASH FLOW -3.040 1.178 1.300,5 1.437 1.586,8 2.054,1
Keterangan:
1)Deprecition tahunke-n = (P-S)/n
= (1.000.000 – 0)/5 = $200.000 (tetap selama 5 tahun)
2)Cost depletion = Basis penyesuaian x
Mineral yang diambil dan dijual selama tahun berjalan
Mineral yang ditemukan pada awal tahun
Cost depletion th-1 = ($900.000)(100.000 ounces / 500.000 ounces) = $180.000
Cost depletion th-2 = ($900.000-$180.000)(100.000 ounces / 400.000 ounces) = $180.000
Cost depletion th-3 = ($720.000-$180.000)(100.000 ounces / 300.000 ounces) = $180.000
Cost depletion th-4 = ($540.000-$180.000)(100.000 ounces / 200.000 ounces) = $180.000
Cost depletion th-5 = ($360.000-$180.000)(100.000 ounces / 100.000 ounces) = $180.000
4)Tax (30%), jika taxable income negatifmaka dianggapsebagai saving.
5)Capital cost = working capital+acquisitionrigths +depreciable equipment.
= $300.000 + $900.000 + $1.000.000 = $2.200.000
c. Cashflow setelah pajak jika 100% development cost digunakan untukpengurang pajakdan deplesi dihitung
dengan metode percentage depletion.
Tahun ke- 0 1 2 3 4 5
Rev enue 0 3.000 3.300 3.630 3.993 4.392
-Royalty (15%) 0 -450 -495 -545 -599 -659
Net rev enue 0 2.550 2.805 3.086 3.394 3.733
-Operatingcost 0 -1.000 -1.080 -1.166 -1.260 -1.360
-Miningtax 0 -30 -30 -30 -30 -30
-Dev elopment cost -1.200 0 0 0 0 0
-Depreciation1) 0 -200 -200 -200 -200 -200
-Percentage depletion2) 0 -382 -421 -463 -509 -560
Taxable income -1.200 938 1.074 1.227 1.395 1.583
-Tax (30%)3) 360 -281,4 -322,2 -368,1 -418,5 -474,9
Net income -840 656,6 751,8 858,9 976,5 1.108,1
+Depreciation 0 200 200 200 200 200
+Depletion 0 382 421 463 509 560
+Working capital return 0 0 0 0 0 300
-Capital cost4) -2.200 0 0 0 0 0
CASH FLOW -3.040 1.238,6 1.372,8 1.521,9 1.685,5 2.168,1
Yuli Kusumawati, Catatan Kuliah Ekonomi Mineral - 46
Keterangan:
1)Deprecition tahunke-n = (P-S)/n
= (1.000.000 – 0)/5 = $200.000 (tetap selama 5 tahun)
2)Percentage depletion diperbolehkan jika nilainyalebih kecil dari batas deplesi maksimal:
Batas deplesi maksimal = 50% x taxable income before depletion
Batas deplesi th-1 = 50%(2.550.000 - 1.000.000 – 30.000 – 0 – 200.000) = $660
Batas deplesi th-2 = 50%(2.805.000 - 1.080.000 – 30.000 – 0 – 200.000) = $747,5
Batas deplesi th-3 = 50%(3.086.000 - 1.166.000 – 30.000 – 0 – 200.000) = $845
Batas deplesi th-4 = 50%(3.394.000 - 1.260.000 – 30.000 – 0 – 200.000) = $952
Batas deplesi th-5 = 50%(3.733.000 - 1.360.000 – 30.000 – 0 – 200.000) = $1.071,5
Percentage depletion = 15% x net revenue
Percentage depletion th-1 = 15% x 2.550.000 = $382.000
Percentage depletion th-2 = 15% x 2.805.000 = $421.000
Percentage depletion th-3 = 15% x 3.086.000 = $463.000
Percentage depletion th-4 = 15% x 3.394.000 = $509.000
Percentage depletion th-5 = 15% x 3.733.000 = $560.000
3)Tax (30%), jika taxable income negatifmaka dianggapsebagai saving.
4)Capital cost = working capital+acquisitionrigths +depreciable equipment.
= $300.000 + $900.000 + $1.000.000 = $2.200.000
d. Cashflow setelah pajak jika amortisasi 30% selama 5 tahun dan deplesi dihitung dengan metode cost
depletion.
Tahun ke- 0 1 2 3 4 5
Rev enue 0 3.000 3.300 3.630 3.993 4.392
-Royalty (15%) 0 -450 -495 -545 -599 -659
Net rev enue 0 2.550 2.805 3.086 3.394 3.733
-Operating cost 0 -1.000 -1.080 -1.166 -1.260 -1.360
-Miningtax 0 -30 -30 -30 -30 -30
-Dev elopment cost1) -840 0 0 0 0 0
-Amortization2) -72 -72 -72 -72 -72 0
-Depreciation3) 0 -200 -200 -200 -200 -200
-Cost depletion4) 0 -180 -180 -180 -180 -180
Taxable income -912 1.068 1.243 1.438 1.652 1.963
-Tax (30%)5) 273,6 -320,4 -372,9 -431,4 -495,6 -588,9
Net income -638,4 747,6 870,1 1.006,6 1.156,4 1.374,1
+Amortization 72 72 72 72 72 0
+Depreciation 0 200 200 200 200 200
+Depletion 0 180 180 180 180 180
+Working capital return 0 0 0 0 0 300
-Capital cost6) -2.560 0 0 0 0 0
CASH FLOW -3.126,4 1.199,6 1.322,1 1.458,6 1.608,4 2.054,1
Keterangan:
1)Dev elopmentcost = 70% x $1.200 = $840
2)Amortisasi tahun ke-n = 30% (dev . cost)/periode amortisasi
= 30% (1.200.000)/5 = $72.000 (tetap sampai tahunke-5)
3)Deprecition tahunke-n = (P-S)/n
= (1.000.000 – 0)/5 = $200.000 (tetap selama 5 tahun)
Ekonomi Mineral by Yuli Kusumawati
Ekonomi Mineral by Yuli Kusumawati
Ekonomi Mineral by Yuli Kusumawati
Ekonomi Mineral by Yuli Kusumawati
Ekonomi Mineral by Yuli Kusumawati
Ekonomi Mineral by Yuli Kusumawati
Ekonomi Mineral by Yuli Kusumawati
Ekonomi Mineral by Yuli Kusumawati
Ekonomi Mineral by Yuli Kusumawati
Ekonomi Mineral by Yuli Kusumawati
Ekonomi Mineral by Yuli Kusumawati
Ekonomi Mineral by Yuli Kusumawati
Ekonomi Mineral by Yuli Kusumawati
Ekonomi Mineral by Yuli Kusumawati
Ekonomi Mineral by Yuli Kusumawati
Ekonomi Mineral by Yuli Kusumawati
Ekonomi Mineral by Yuli Kusumawati
Ekonomi Mineral by Yuli Kusumawati
Ekonomi Mineral by Yuli Kusumawati

More Related Content

What's hot

Perencanaan tambang
Perencanaan tambangPerencanaan tambang
Perencanaan tambangramaldini
 
PENGARUH AIR ASAM TAMBANG TERHADAP LINGKUNGAN AIR DAN PENANGANNYA
PENGARUH AIR ASAM TAMBANG TERHADAP LINGKUNGAN AIR DAN PENANGANNYAPENGARUH AIR ASAM TAMBANG TERHADAP LINGKUNGAN AIR DAN PENANGANNYA
PENGARUH AIR ASAM TAMBANG TERHADAP LINGKUNGAN AIR DAN PENANGANNYA
Fathur Rozaq
 
mineral-dan-batuan
mineral-dan-batuanmineral-dan-batuan
mineral-dan-batuan
ALAM SEKITAR
 
Proposal kegiatan perencanaan pemboran
Proposal kegiatan perencanaan pemboranProposal kegiatan perencanaan pemboran
Proposal kegiatan perencanaan pemboran
LeonardoSitorus
 
Genesa Bahan Galian
Genesa Bahan GalianGenesa Bahan Galian
Genesa Bahan Galian
permukaan bumi
 
GeoTek Kestabilan Lereng
GeoTek Kestabilan LerengGeoTek Kestabilan Lereng
GeoTek Kestabilan LerengAyu Kuleh Putri
 
Istilah-istilah Pertambangan
Istilah-istilah  PertambanganIstilah-istilah  Pertambangan
Istilah-istilah Pertambangan
fridolin bin stefanus
 
Humprey spiral
Humprey spiralHumprey spiral
Humprey spiral
Sylvester Saragih
 
Pola peledakan
Pola peledakanPola peledakan
Pola peledakan
Romie Hendrawan
 
40084299 analisis-investasi-tambang
40084299 analisis-investasi-tambang40084299 analisis-investasi-tambang
40084299 analisis-investasi-tambangNurfirman Halwan
 
Sistem Penambangan
Sistem PenambanganSistem Penambangan
Sistem Penambangan
fridolin bin stefanus
 
Teknik eksplorasi_chapter 1_eksplorasi langsung
Teknik eksplorasi_chapter 1_eksplorasi langsungTeknik eksplorasi_chapter 1_eksplorasi langsung
Teknik eksplorasi_chapter 1_eksplorasi langsung
heny novi
 
Acara 1
Acara 1Acara 1
Acara 1
Mega Ayu
 
Teknik Eksplorasi Tambang
Teknik Eksplorasi TambangTeknik Eksplorasi Tambang
Teknik Eksplorasi Tambang
nyongker29
 
PASIR KUARSA - BAHAN GALIAN INDUSTRI - BONITA
PASIR KUARSA - BAHAN GALIAN INDUSTRI - BONITAPASIR KUARSA - BAHAN GALIAN INDUSTRI - BONITA
PASIR KUARSA - BAHAN GALIAN INDUSTRI - BONITA
Bonita Susimah
 
Eksplorasi sumber daya bahan galian
Eksplorasi sumber daya bahan galianEksplorasi sumber daya bahan galian
Eksplorasi sumber daya bahan galianIpung Noor
 

What's hot (20)

Definisi tereka, terukur dll
Definisi tereka, terukur dllDefinisi tereka, terukur dll
Definisi tereka, terukur dll
 
Perencanaan tambang
Perencanaan tambangPerencanaan tambang
Perencanaan tambang
 
1.geoteknik tambang
1.geoteknik tambang1.geoteknik tambang
1.geoteknik tambang
 
PENGARUH AIR ASAM TAMBANG TERHADAP LINGKUNGAN AIR DAN PENANGANNYA
PENGARUH AIR ASAM TAMBANG TERHADAP LINGKUNGAN AIR DAN PENANGANNYAPENGARUH AIR ASAM TAMBANG TERHADAP LINGKUNGAN AIR DAN PENANGANNYA
PENGARUH AIR ASAM TAMBANG TERHADAP LINGKUNGAN AIR DAN PENANGANNYA
 
mineral-dan-batuan
mineral-dan-batuanmineral-dan-batuan
mineral-dan-batuan
 
Proposal kegiatan perencanaan pemboran
Proposal kegiatan perencanaan pemboranProposal kegiatan perencanaan pemboran
Proposal kegiatan perencanaan pemboran
 
Genesa Bahan Galian
Genesa Bahan GalianGenesa Bahan Galian
Genesa Bahan Galian
 
GeoTek Kestabilan Lereng
GeoTek Kestabilan LerengGeoTek Kestabilan Lereng
GeoTek Kestabilan Lereng
 
Istilah-istilah Pertambangan
Istilah-istilah  PertambanganIstilah-istilah  Pertambangan
Istilah-istilah Pertambangan
 
Humprey spiral
Humprey spiralHumprey spiral
Humprey spiral
 
Pola peledakan
Pola peledakanPola peledakan
Pola peledakan
 
Pengolahan Bahan Galian
Pengolahan Bahan GalianPengolahan Bahan Galian
Pengolahan Bahan Galian
 
40084299 analisis-investasi-tambang
40084299 analisis-investasi-tambang40084299 analisis-investasi-tambang
40084299 analisis-investasi-tambang
 
Sistem Penambangan
Sistem PenambanganSistem Penambangan
Sistem Penambangan
 
Teknik eksplorasi_chapter 1_eksplorasi langsung
Teknik eksplorasi_chapter 1_eksplorasi langsungTeknik eksplorasi_chapter 1_eksplorasi langsung
Teknik eksplorasi_chapter 1_eksplorasi langsung
 
Acara 1
Acara 1Acara 1
Acara 1
 
Pemboran tambang
Pemboran tambangPemboran tambang
Pemboran tambang
 
Teknik Eksplorasi Tambang
Teknik Eksplorasi TambangTeknik Eksplorasi Tambang
Teknik Eksplorasi Tambang
 
PASIR KUARSA - BAHAN GALIAN INDUSTRI - BONITA
PASIR KUARSA - BAHAN GALIAN INDUSTRI - BONITAPASIR KUARSA - BAHAN GALIAN INDUSTRI - BONITA
PASIR KUARSA - BAHAN GALIAN INDUSTRI - BONITA
 
Eksplorasi sumber daya bahan galian
Eksplorasi sumber daya bahan galianEksplorasi sumber daya bahan galian
Eksplorasi sumber daya bahan galian
 

Viewers also liked

Hal-Hal Yang Penting dalam Survey Tambang
Hal-Hal Yang Penting dalam Survey TambangHal-Hal Yang Penting dalam Survey Tambang
Hal-Hal Yang Penting dalam Survey Tambang
Edhis Triyono Hermawan
 
Tambang STTNAS _ Mata Kuliah Batubara_Semester IV_Coal sttnas supandi_2014_09...
Tambang STTNAS _ Mata Kuliah Batubara_Semester IV_Coal sttnas supandi_2014_09...Tambang STTNAS _ Mata Kuliah Batubara_Semester IV_Coal sttnas supandi_2014_09...
Tambang STTNAS _ Mata Kuliah Batubara_Semester IV_Coal sttnas supandi_2014_09...
Mario Yuven
 
Materi Kuliah Komputasi tambang Materi 1 Kuliah Teknik Pertambangan STTNAS Yo...
Materi Kuliah Komputasi tambang Materi 1 Kuliah Teknik Pertambangan STTNAS Yo...Materi Kuliah Komputasi tambang Materi 1 Kuliah Teknik Pertambangan STTNAS Yo...
Materi Kuliah Komputasi tambang Materi 1 Kuliah Teknik Pertambangan STTNAS Yo...
Mario Yuven
 
Studi Kelayakan Tambang: Pengantar
Studi Kelayakan Tambang: PengantarStudi Kelayakan Tambang: Pengantar
Studi Kelayakan Tambang: Pengantar
Universitas Sriwijaya
 
Barisan dan deret
Barisan dan deretBarisan dan deret
Barisan dan deret
yulika usman
 
Persamaan dan fungsi kuadrat
Persamaan dan fungsi kuadratPersamaan dan fungsi kuadrat
Persamaan dan fungsi kuadrat
yulika usman
 
ilmu ukur tambang
ilmu ukur tambangilmu ukur tambang
ilmu ukur tambang
Agnes Evelina
 
Lutfi koto : model & teknik pengambilan keputusan
Lutfi koto : model & teknik pengambilan keputusanLutfi koto : model & teknik pengambilan keputusan
Lutfi koto : model & teknik pengambilan keputusan
Lutfi Koto
 
Cara Terbentuknya Berbagai Macam Sumber Daya Mineral
Cara Terbentuknya Berbagai Macam Sumber Daya MineralCara Terbentuknya Berbagai Macam Sumber Daya Mineral
Cara Terbentuknya Berbagai Macam Sumber Daya Mineral
Mira Micako
 
Teknik Pengambilan Keputusan
Teknik Pengambilan KeputusanTeknik Pengambilan Keputusan
Teknik Pengambilan Keputusan
Eko Mardianto
 
Latihan soal ilmu ukur tambang
Latihan soal ilmu ukur tambangLatihan soal ilmu ukur tambang
Latihan soal ilmu ukur tambang
yulika usman
 
Teknik Pengambilan Keputusan
Teknik Pengambilan KeputusanTeknik Pengambilan Keputusan
Teknik Pengambilan Keputusan
hanatamara
 
Materi Kuliah Komputasi tambang Materi 1 Kuliah Teknik Pertambangan STTNAS Yo...
Materi Kuliah Komputasi tambang Materi 1 Kuliah Teknik Pertambangan STTNAS Yo...Materi Kuliah Komputasi tambang Materi 1 Kuliah Teknik Pertambangan STTNAS Yo...
Materi Kuliah Komputasi tambang Materi 1 Kuliah Teknik Pertambangan STTNAS Yo...
Mario Yuven
 
materi-kuliah-komputasi-tambang-kuliah-teknik-pertambangan-sttnas-yogyakarta...
 materi-kuliah-komputasi-tambang-kuliah-teknik-pertambangan-sttnas-yogyakarta... materi-kuliah-komputasi-tambang-kuliah-teknik-pertambangan-sttnas-yogyakarta...
materi-kuliah-komputasi-tambang-kuliah-teknik-pertambangan-sttnas-yogyakarta...
Mario Yuven
 
materi-kuliah-komputasi-tambang kuliah-teknik-pertambangan-sttnas-yogyakarta-...
materi-kuliah-komputasi-tambang kuliah-teknik-pertambangan-sttnas-yogyakarta-...materi-kuliah-komputasi-tambang kuliah-teknik-pertambangan-sttnas-yogyakarta-...
materi-kuliah-komputasi-tambang kuliah-teknik-pertambangan-sttnas-yogyakarta-...
Mario Yuven
 
[Cv] tedy satria
[Cv]   tedy satria[Cv]   tedy satria
[Cv] tedy satria
Tedy Satria
 
Ilmuwan minescape
Ilmuwan minescapeIlmuwan minescape
Ilmuwan minescape
airfanza
 
kalimatan timur, samarinda
kalimatan timur, samarindakalimatan timur, samarinda
kalimatan timur, samarinda
ALAM SEKITAR
 
All , minescape
All , minescapeAll , minescape
All , minescape
ALAM SEKITAR
 

Viewers also liked (20)

Hal-Hal Yang Penting dalam Survey Tambang
Hal-Hal Yang Penting dalam Survey TambangHal-Hal Yang Penting dalam Survey Tambang
Hal-Hal Yang Penting dalam Survey Tambang
 
Tambang STTNAS _ Mata Kuliah Batubara_Semester IV_Coal sttnas supandi_2014_09...
Tambang STTNAS _ Mata Kuliah Batubara_Semester IV_Coal sttnas supandi_2014_09...Tambang STTNAS _ Mata Kuliah Batubara_Semester IV_Coal sttnas supandi_2014_09...
Tambang STTNAS _ Mata Kuliah Batubara_Semester IV_Coal sttnas supandi_2014_09...
 
Materi Kuliah Komputasi tambang Materi 1 Kuliah Teknik Pertambangan STTNAS Yo...
Materi Kuliah Komputasi tambang Materi 1 Kuliah Teknik Pertambangan STTNAS Yo...Materi Kuliah Komputasi tambang Materi 1 Kuliah Teknik Pertambangan STTNAS Yo...
Materi Kuliah Komputasi tambang Materi 1 Kuliah Teknik Pertambangan STTNAS Yo...
 
Studi Kelayakan Tambang: Pengantar
Studi Kelayakan Tambang: PengantarStudi Kelayakan Tambang: Pengantar
Studi Kelayakan Tambang: Pengantar
 
Pohon analisis valid
Pohon analisis validPohon analisis valid
Pohon analisis valid
 
Barisan dan deret
Barisan dan deretBarisan dan deret
Barisan dan deret
 
Persamaan dan fungsi kuadrat
Persamaan dan fungsi kuadratPersamaan dan fungsi kuadrat
Persamaan dan fungsi kuadrat
 
ilmu ukur tambang
ilmu ukur tambangilmu ukur tambang
ilmu ukur tambang
 
Lutfi koto : model & teknik pengambilan keputusan
Lutfi koto : model & teknik pengambilan keputusanLutfi koto : model & teknik pengambilan keputusan
Lutfi koto : model & teknik pengambilan keputusan
 
Cara Terbentuknya Berbagai Macam Sumber Daya Mineral
Cara Terbentuknya Berbagai Macam Sumber Daya MineralCara Terbentuknya Berbagai Macam Sumber Daya Mineral
Cara Terbentuknya Berbagai Macam Sumber Daya Mineral
 
Teknik Pengambilan Keputusan
Teknik Pengambilan KeputusanTeknik Pengambilan Keputusan
Teknik Pengambilan Keputusan
 
Latihan soal ilmu ukur tambang
Latihan soal ilmu ukur tambangLatihan soal ilmu ukur tambang
Latihan soal ilmu ukur tambang
 
Teknik Pengambilan Keputusan
Teknik Pengambilan KeputusanTeknik Pengambilan Keputusan
Teknik Pengambilan Keputusan
 
Materi Kuliah Komputasi tambang Materi 1 Kuliah Teknik Pertambangan STTNAS Yo...
Materi Kuliah Komputasi tambang Materi 1 Kuliah Teknik Pertambangan STTNAS Yo...Materi Kuliah Komputasi tambang Materi 1 Kuliah Teknik Pertambangan STTNAS Yo...
Materi Kuliah Komputasi tambang Materi 1 Kuliah Teknik Pertambangan STTNAS Yo...
 
materi-kuliah-komputasi-tambang-kuliah-teknik-pertambangan-sttnas-yogyakarta...
 materi-kuliah-komputasi-tambang-kuliah-teknik-pertambangan-sttnas-yogyakarta... materi-kuliah-komputasi-tambang-kuliah-teknik-pertambangan-sttnas-yogyakarta...
materi-kuliah-komputasi-tambang-kuliah-teknik-pertambangan-sttnas-yogyakarta...
 
materi-kuliah-komputasi-tambang kuliah-teknik-pertambangan-sttnas-yogyakarta-...
materi-kuliah-komputasi-tambang kuliah-teknik-pertambangan-sttnas-yogyakarta-...materi-kuliah-komputasi-tambang kuliah-teknik-pertambangan-sttnas-yogyakarta-...
materi-kuliah-komputasi-tambang kuliah-teknik-pertambangan-sttnas-yogyakarta-...
 
[Cv] tedy satria
[Cv]   tedy satria[Cv]   tedy satria
[Cv] tedy satria
 
Ilmuwan minescape
Ilmuwan minescapeIlmuwan minescape
Ilmuwan minescape
 
kalimatan timur, samarinda
kalimatan timur, samarindakalimatan timur, samarinda
kalimatan timur, samarinda
 
All , minescape
All , minescapeAll , minescape
All , minescape
 

Similar to Ekonomi Mineral by Yuli Kusumawati

Tugas Ekonomi Teknik ke 3
Tugas Ekonomi Teknik ke 3Tugas Ekonomi Teknik ke 3
Tugas Ekonomi Teknik ke 3Sri Sediaz
 
Design Studi Kelayakan_dalam "PROJECT FEASIBILITY STUDY"
Design Studi Kelayakan_dalam "PROJECT FEASIBILITY STUDY"Design Studi Kelayakan_dalam "PROJECT FEASIBILITY STUDY"
Design Studi Kelayakan_dalam "PROJECT FEASIBILITY STUDY"
Kanaidi ken
 
BISNIS INTERNASIONAL
BISNIS INTERNASIONALBISNIS INTERNASIONAL
BISNIS INTERNASIONAL
tera paradisani
 
BMP EKMA4311 Studi Kelayakan Bisnis
BMP EKMA4311 Studi Kelayakan BisnisBMP EKMA4311 Studi Kelayakan Bisnis
BMP EKMA4311 Studi Kelayakan Bisnis
Mang Engkus
 
KAJIAN METODA PERANCANGAN II “TOKO BAJU”
KAJIAN METODA PERANCANGAN II “TOKO BAJU”KAJIAN METODA PERANCANGAN II “TOKO BAJU”
KAJIAN METODA PERANCANGAN II “TOKO BAJU”
Rabiyatul Adawiyah
 
Studi kelayakan
Studi kelayakanStudi kelayakan
Studi kelayakan
Lukman Hakim
 
ANALIS PROYEK PEMBANGUNAN KEHUTANAN
ANALIS PROYEK PEMBANGUNAN KEHUTANANANALIS PROYEK PEMBANGUNAN KEHUTANAN
ANALIS PROYEK PEMBANGUNAN KEHUTANAN
EDIS BLOG
 
Analisis proyek pembangunan kehutanan
Analisis proyek pembangunan kehutananAnalisis proyek pembangunan kehutanan
Analisis proyek pembangunan kehutanan
EDIS BLOG
 
External Macro Environment Analysis. Universitas Mercu Buana. 2019
External Macro Environment Analysis. Universitas Mercu Buana. 2019External Macro Environment Analysis. Universitas Mercu Buana. 2019
External Macro Environment Analysis. Universitas Mercu Buana. 2019
Donna Wibiananda Suryaman
 
External macro environment analysis. universitas mercu buana. 2019
External macro environment analysis. universitas mercu buana. 2019External macro environment analysis. universitas mercu buana. 2019
External macro environment analysis. universitas mercu buana. 2019
Donna Wibiananda Suryaman
 
mata Kuliah Analisa Kelayakan Usaha semester 6 agribisnis
mata Kuliah Analisa Kelayakan Usaha semester 6 agribisnismata Kuliah Analisa Kelayakan Usaha semester 6 agribisnis
mata Kuliah Analisa Kelayakan Usaha semester 6 agribisnis
budiresno
 
Kuliah Materi Analisa Kelayakan Usaha.pptx
Kuliah Materi Analisa Kelayakan Usaha.pptxKuliah Materi Analisa Kelayakan Usaha.pptx
Kuliah Materi Analisa Kelayakan Usaha.pptx
budiresno
 
Pengertian Study Kelayakan Proyek (PROJECT FEASIBILITY STUDY)
Pengertian Study Kelayakan Proyek (PROJECT FEASIBILITY STUDY)Pengertian Study Kelayakan Proyek (PROJECT FEASIBILITY STUDY)
Pengertian Study Kelayakan Proyek (PROJECT FEASIBILITY STUDY)
Kanaidi ken
 
Ekonomi teknik 2
Ekonomi teknik 2Ekonomi teknik 2
Ekonomi teknik 2
Yanu Priandana
 
Peran Pelaku Usaha dalam Perbaikan Tata Kelola Batubara di Indonesia
Peran Pelaku Usaha dalam Perbaikan Tata Kelola Batubara di IndonesiaPeran Pelaku Usaha dalam Perbaikan Tata Kelola Batubara di Indonesia
Peran Pelaku Usaha dalam Perbaikan Tata Kelola Batubara di Indonesia
Publish What You Pay (PWYP) Indonesia
 
Ekonomi teknik
Ekonomi teknikEkonomi teknik
Ekonomi teknik
Rifan Bukhori
 
Pendahuluan studi kelayakan
Pendahuluan studi kelayakanPendahuluan studi kelayakan
Pendahuluan studi kelayakan72gar
 
Capital budgeting untuk lembaga nonprofit dan sektor publik bagian 1
Capital budgeting untuk lembaga nonprofit dan sektor publik bagian 1Capital budgeting untuk lembaga nonprofit dan sektor publik bagian 1
Capital budgeting untuk lembaga nonprofit dan sektor publik bagian 1
Futurum2
 
External macro environment analysis
External macro environment analysisExternal macro environment analysis
External macro environment analysis
DavidNehemia1
 
Makalah ekonomi teknik asbal
Makalah ekonomi teknik asbalMakalah ekonomi teknik asbal
Makalah ekonomi teknik asbal
asbalkhairi
 

Similar to Ekonomi Mineral by Yuli Kusumawati (20)

Tugas Ekonomi Teknik ke 3
Tugas Ekonomi Teknik ke 3Tugas Ekonomi Teknik ke 3
Tugas Ekonomi Teknik ke 3
 
Design Studi Kelayakan_dalam "PROJECT FEASIBILITY STUDY"
Design Studi Kelayakan_dalam "PROJECT FEASIBILITY STUDY"Design Studi Kelayakan_dalam "PROJECT FEASIBILITY STUDY"
Design Studi Kelayakan_dalam "PROJECT FEASIBILITY STUDY"
 
BISNIS INTERNASIONAL
BISNIS INTERNASIONALBISNIS INTERNASIONAL
BISNIS INTERNASIONAL
 
BMP EKMA4311 Studi Kelayakan Bisnis
BMP EKMA4311 Studi Kelayakan BisnisBMP EKMA4311 Studi Kelayakan Bisnis
BMP EKMA4311 Studi Kelayakan Bisnis
 
KAJIAN METODA PERANCANGAN II “TOKO BAJU”
KAJIAN METODA PERANCANGAN II “TOKO BAJU”KAJIAN METODA PERANCANGAN II “TOKO BAJU”
KAJIAN METODA PERANCANGAN II “TOKO BAJU”
 
Studi kelayakan
Studi kelayakanStudi kelayakan
Studi kelayakan
 
ANALIS PROYEK PEMBANGUNAN KEHUTANAN
ANALIS PROYEK PEMBANGUNAN KEHUTANANANALIS PROYEK PEMBANGUNAN KEHUTANAN
ANALIS PROYEK PEMBANGUNAN KEHUTANAN
 
Analisis proyek pembangunan kehutanan
Analisis proyek pembangunan kehutananAnalisis proyek pembangunan kehutanan
Analisis proyek pembangunan kehutanan
 
External Macro Environment Analysis. Universitas Mercu Buana. 2019
External Macro Environment Analysis. Universitas Mercu Buana. 2019External Macro Environment Analysis. Universitas Mercu Buana. 2019
External Macro Environment Analysis. Universitas Mercu Buana. 2019
 
External macro environment analysis. universitas mercu buana. 2019
External macro environment analysis. universitas mercu buana. 2019External macro environment analysis. universitas mercu buana. 2019
External macro environment analysis. universitas mercu buana. 2019
 
mata Kuliah Analisa Kelayakan Usaha semester 6 agribisnis
mata Kuliah Analisa Kelayakan Usaha semester 6 agribisnismata Kuliah Analisa Kelayakan Usaha semester 6 agribisnis
mata Kuliah Analisa Kelayakan Usaha semester 6 agribisnis
 
Kuliah Materi Analisa Kelayakan Usaha.pptx
Kuliah Materi Analisa Kelayakan Usaha.pptxKuliah Materi Analisa Kelayakan Usaha.pptx
Kuliah Materi Analisa Kelayakan Usaha.pptx
 
Pengertian Study Kelayakan Proyek (PROJECT FEASIBILITY STUDY)
Pengertian Study Kelayakan Proyek (PROJECT FEASIBILITY STUDY)Pengertian Study Kelayakan Proyek (PROJECT FEASIBILITY STUDY)
Pengertian Study Kelayakan Proyek (PROJECT FEASIBILITY STUDY)
 
Ekonomi teknik 2
Ekonomi teknik 2Ekonomi teknik 2
Ekonomi teknik 2
 
Peran Pelaku Usaha dalam Perbaikan Tata Kelola Batubara di Indonesia
Peran Pelaku Usaha dalam Perbaikan Tata Kelola Batubara di IndonesiaPeran Pelaku Usaha dalam Perbaikan Tata Kelola Batubara di Indonesia
Peran Pelaku Usaha dalam Perbaikan Tata Kelola Batubara di Indonesia
 
Ekonomi teknik
Ekonomi teknikEkonomi teknik
Ekonomi teknik
 
Pendahuluan studi kelayakan
Pendahuluan studi kelayakanPendahuluan studi kelayakan
Pendahuluan studi kelayakan
 
Capital budgeting untuk lembaga nonprofit dan sektor publik bagian 1
Capital budgeting untuk lembaga nonprofit dan sektor publik bagian 1Capital budgeting untuk lembaga nonprofit dan sektor publik bagian 1
Capital budgeting untuk lembaga nonprofit dan sektor publik bagian 1
 
External macro environment analysis
External macro environment analysisExternal macro environment analysis
External macro environment analysis
 
Makalah ekonomi teknik asbal
Makalah ekonomi teknik asbalMakalah ekonomi teknik asbal
Makalah ekonomi teknik asbal
 

More from yulika usman

Eksponen dan logaritma
Eksponen dan logaritmaEksponen dan logaritma
Eksponen dan logaritma
yulika usman
 
Persamaan linear dan matriks
Persamaan linear dan matriksPersamaan linear dan matriks
Persamaan linear dan matriks
yulika usman
 
Hitungan Ilmu Ukur Tanah
Hitungan Ilmu Ukur TanahHitungan Ilmu Ukur Tanah
Hitungan Ilmu Ukur Tanah
yulika usman
 
Ilmu Ukur Tanah by Yuli Kusumawati
Ilmu Ukur Tanah by Yuli KusumawatiIlmu Ukur Tanah by Yuli Kusumawati
Ilmu Ukur Tanah by Yuli Kusumawati
yulika usman
 
SAP Ilmu Ukur Tambang
SAP Ilmu Ukur TambangSAP Ilmu Ukur Tambang
SAP Ilmu Ukur Tambang
yulika usman
 
Sap ekonomi mineral
Sap ekonomi mineralSap ekonomi mineral
Sap ekonomi mineral
yulika usman
 

More from yulika usman (6)

Eksponen dan logaritma
Eksponen dan logaritmaEksponen dan logaritma
Eksponen dan logaritma
 
Persamaan linear dan matriks
Persamaan linear dan matriksPersamaan linear dan matriks
Persamaan linear dan matriks
 
Hitungan Ilmu Ukur Tanah
Hitungan Ilmu Ukur TanahHitungan Ilmu Ukur Tanah
Hitungan Ilmu Ukur Tanah
 
Ilmu Ukur Tanah by Yuli Kusumawati
Ilmu Ukur Tanah by Yuli KusumawatiIlmu Ukur Tanah by Yuli Kusumawati
Ilmu Ukur Tanah by Yuli Kusumawati
 
SAP Ilmu Ukur Tambang
SAP Ilmu Ukur TambangSAP Ilmu Ukur Tambang
SAP Ilmu Ukur Tambang
 
Sap ekonomi mineral
Sap ekonomi mineralSap ekonomi mineral
Sap ekonomi mineral
 

Recently uploaded

EKONOMI INDUSTRI ilmu tentang industri dan disiplin
EKONOMI INDUSTRI ilmu tentang industri dan disiplinEKONOMI INDUSTRI ilmu tentang industri dan disiplin
EKONOMI INDUSTRI ilmu tentang industri dan disiplin
anthoniusaldolemauk
 
Konsep Perbankan Syariah di Indonesia.ppt
Konsep Perbankan Syariah di Indonesia.pptKonsep Perbankan Syariah di Indonesia.ppt
Konsep Perbankan Syariah di Indonesia.ppt
AchmadHasanHafidzi
 
Pengertian Surplus Konsumen dan Produsen.pdf
Pengertian Surplus Konsumen dan Produsen.pdfPengertian Surplus Konsumen dan Produsen.pdf
Pengertian Surplus Konsumen dan Produsen.pdf
fadilahsaleh427
 
PPT PAJAK DAERAH PERPAJAKAN MANAJEMEN S1
PPT PAJAK DAERAH PERPAJAKAN MANAJEMEN S1PPT PAJAK DAERAH PERPAJAKAN MANAJEMEN S1
PPT PAJAK DAERAH PERPAJAKAN MANAJEMEN S1
IndahMeilani2
 
METODE MODI (MODIFIED DISTRIBUTION METHODE).pptx
METODE MODI (MODIFIED DISTRIBUTION METHODE).pptxMETODE MODI (MODIFIED DISTRIBUTION METHODE).pptx
METODE MODI (MODIFIED DISTRIBUTION METHODE).pptx
UNIVERSITAS MUHAMMADIYAH BERAU
 
Modul Ajar Kurikulum Merdeka Tahun 2024.pptx
Modul Ajar Kurikulum Merdeka Tahun 2024.pptxModul Ajar Kurikulum Merdeka Tahun 2024.pptx
Modul Ajar Kurikulum Merdeka Tahun 2024.pptx
MarkusPiyusmanZebua
 
Sesi 4_Kelompok 3 Kode Etik Profesi Akuntan Publik.pptx
Sesi 4_Kelompok 3 Kode Etik Profesi Akuntan Publik.pptxSesi 4_Kelompok 3 Kode Etik Profesi Akuntan Publik.pptx
Sesi 4_Kelompok 3 Kode Etik Profesi Akuntan Publik.pptx
bidakara2016
 
reksadana syariah lutfi nihayatul khusniah
reksadana syariah lutfi nihayatul khusniahreksadana syariah lutfi nihayatul khusniah
reksadana syariah lutfi nihayatul khusniah
AhmadVikriKhoirulAna
 
Ppt_perdagangan_luar_negeri_proteksi_dan.ppt
Ppt_perdagangan_luar_negeri_proteksi_dan.pptPpt_perdagangan_luar_negeri_proteksi_dan.ppt
Ppt_perdagangan_luar_negeri_proteksi_dan.ppt
mariapasaribu13
 
DJP - RUU KUP.pdf RUU Perubahan Kelima UU KUP
DJP - RUU KUP.pdf RUU Perubahan Kelima UU KUPDJP - RUU KUP.pdf RUU Perubahan Kelima UU KUP
DJP - RUU KUP.pdf RUU Perubahan Kelima UU KUP
adjhe17ks1
 
PPT Reksadana (Reksadana ekonomi syariah).pptx
PPT Reksadana (Reksadana ekonomi syariah).pptxPPT Reksadana (Reksadana ekonomi syariah).pptx
PPT Reksadana (Reksadana ekonomi syariah).pptx
f4hmizakaria123
 
Cost Benefit Analysisss perhitunngan.ppt
Cost Benefit Analysisss perhitunngan.pptCost Benefit Analysisss perhitunngan.ppt
Cost Benefit Analysisss perhitunngan.ppt
meincha1152
 
PPT SEMPRO PENGARUH JIWA KEWIRAUSAHAAN MOTIVASI DAN MODAL USAHA TERHADAP PERK...
PPT SEMPRO PENGARUH JIWA KEWIRAUSAHAAN MOTIVASI DAN MODAL USAHA TERHADAP PERK...PPT SEMPRO PENGARUH JIWA KEWIRAUSAHAAN MOTIVASI DAN MODAL USAHA TERHADAP PERK...
PPT SEMPRO PENGARUH JIWA KEWIRAUSAHAAN MOTIVASI DAN MODAL USAHA TERHADAP PERK...
hoiriyono
 
Prosedur Ekspor : Studi Kasus Ekspor Briket ke Yaman dan Proses Produksi Brik...
Prosedur Ekspor : Studi Kasus Ekspor Briket ke Yaman dan Proses Produksi Brik...Prosedur Ekspor : Studi Kasus Ekspor Briket ke Yaman dan Proses Produksi Brik...
Prosedur Ekspor : Studi Kasus Ekspor Briket ke Yaman dan Proses Produksi Brik...
Anisa Rizki Rahmawati
 
Dapat SP2DK, Harus Apa? Bagimana cara merespon surat cinta DJP?
Dapat SP2DK, Harus Apa? Bagimana cara merespon surat cinta DJP?Dapat SP2DK, Harus Apa? Bagimana cara merespon surat cinta DJP?
Dapat SP2DK, Harus Apa? Bagimana cara merespon surat cinta DJP?
EnforceA Real Solution
 
Pendapatan dan beban dalam Akuntansi.pptx
Pendapatan dan beban dalam Akuntansi.pptxPendapatan dan beban dalam Akuntansi.pptx
Pendapatan dan beban dalam Akuntansi.pptx
LidyaManuelia1
 
METODE STEPPING STONE (BATU LONCATANA) REVISI.pptx
METODE STEPPING STONE (BATU LONCATANA) REVISI.pptxMETODE STEPPING STONE (BATU LONCATANA) REVISI.pptx
METODE STEPPING STONE (BATU LONCATANA) REVISI.pptx
UNIVERSITAS MUHAMMADIYAH BERAU
 
460012937-Rpp-kelas-rangkap-model-221-docx.docx
460012937-Rpp-kelas-rangkap-model-221-docx.docx460012937-Rpp-kelas-rangkap-model-221-docx.docx
460012937-Rpp-kelas-rangkap-model-221-docx.docx
JefryColter
 

Recently uploaded (18)

EKONOMI INDUSTRI ilmu tentang industri dan disiplin
EKONOMI INDUSTRI ilmu tentang industri dan disiplinEKONOMI INDUSTRI ilmu tentang industri dan disiplin
EKONOMI INDUSTRI ilmu tentang industri dan disiplin
 
Konsep Perbankan Syariah di Indonesia.ppt
Konsep Perbankan Syariah di Indonesia.pptKonsep Perbankan Syariah di Indonesia.ppt
Konsep Perbankan Syariah di Indonesia.ppt
 
Pengertian Surplus Konsumen dan Produsen.pdf
Pengertian Surplus Konsumen dan Produsen.pdfPengertian Surplus Konsumen dan Produsen.pdf
Pengertian Surplus Konsumen dan Produsen.pdf
 
PPT PAJAK DAERAH PERPAJAKAN MANAJEMEN S1
PPT PAJAK DAERAH PERPAJAKAN MANAJEMEN S1PPT PAJAK DAERAH PERPAJAKAN MANAJEMEN S1
PPT PAJAK DAERAH PERPAJAKAN MANAJEMEN S1
 
METODE MODI (MODIFIED DISTRIBUTION METHODE).pptx
METODE MODI (MODIFIED DISTRIBUTION METHODE).pptxMETODE MODI (MODIFIED DISTRIBUTION METHODE).pptx
METODE MODI (MODIFIED DISTRIBUTION METHODE).pptx
 
Modul Ajar Kurikulum Merdeka Tahun 2024.pptx
Modul Ajar Kurikulum Merdeka Tahun 2024.pptxModul Ajar Kurikulum Merdeka Tahun 2024.pptx
Modul Ajar Kurikulum Merdeka Tahun 2024.pptx
 
Sesi 4_Kelompok 3 Kode Etik Profesi Akuntan Publik.pptx
Sesi 4_Kelompok 3 Kode Etik Profesi Akuntan Publik.pptxSesi 4_Kelompok 3 Kode Etik Profesi Akuntan Publik.pptx
Sesi 4_Kelompok 3 Kode Etik Profesi Akuntan Publik.pptx
 
reksadana syariah lutfi nihayatul khusniah
reksadana syariah lutfi nihayatul khusniahreksadana syariah lutfi nihayatul khusniah
reksadana syariah lutfi nihayatul khusniah
 
Ppt_perdagangan_luar_negeri_proteksi_dan.ppt
Ppt_perdagangan_luar_negeri_proteksi_dan.pptPpt_perdagangan_luar_negeri_proteksi_dan.ppt
Ppt_perdagangan_luar_negeri_proteksi_dan.ppt
 
DJP - RUU KUP.pdf RUU Perubahan Kelima UU KUP
DJP - RUU KUP.pdf RUU Perubahan Kelima UU KUPDJP - RUU KUP.pdf RUU Perubahan Kelima UU KUP
DJP - RUU KUP.pdf RUU Perubahan Kelima UU KUP
 
PPT Reksadana (Reksadana ekonomi syariah).pptx
PPT Reksadana (Reksadana ekonomi syariah).pptxPPT Reksadana (Reksadana ekonomi syariah).pptx
PPT Reksadana (Reksadana ekonomi syariah).pptx
 
Cost Benefit Analysisss perhitunngan.ppt
Cost Benefit Analysisss perhitunngan.pptCost Benefit Analysisss perhitunngan.ppt
Cost Benefit Analysisss perhitunngan.ppt
 
PPT SEMPRO PENGARUH JIWA KEWIRAUSAHAAN MOTIVASI DAN MODAL USAHA TERHADAP PERK...
PPT SEMPRO PENGARUH JIWA KEWIRAUSAHAAN MOTIVASI DAN MODAL USAHA TERHADAP PERK...PPT SEMPRO PENGARUH JIWA KEWIRAUSAHAAN MOTIVASI DAN MODAL USAHA TERHADAP PERK...
PPT SEMPRO PENGARUH JIWA KEWIRAUSAHAAN MOTIVASI DAN MODAL USAHA TERHADAP PERK...
 
Prosedur Ekspor : Studi Kasus Ekspor Briket ke Yaman dan Proses Produksi Brik...
Prosedur Ekspor : Studi Kasus Ekspor Briket ke Yaman dan Proses Produksi Brik...Prosedur Ekspor : Studi Kasus Ekspor Briket ke Yaman dan Proses Produksi Brik...
Prosedur Ekspor : Studi Kasus Ekspor Briket ke Yaman dan Proses Produksi Brik...
 
Dapat SP2DK, Harus Apa? Bagimana cara merespon surat cinta DJP?
Dapat SP2DK, Harus Apa? Bagimana cara merespon surat cinta DJP?Dapat SP2DK, Harus Apa? Bagimana cara merespon surat cinta DJP?
Dapat SP2DK, Harus Apa? Bagimana cara merespon surat cinta DJP?
 
Pendapatan dan beban dalam Akuntansi.pptx
Pendapatan dan beban dalam Akuntansi.pptxPendapatan dan beban dalam Akuntansi.pptx
Pendapatan dan beban dalam Akuntansi.pptx
 
METODE STEPPING STONE (BATU LONCATANA) REVISI.pptx
METODE STEPPING STONE (BATU LONCATANA) REVISI.pptxMETODE STEPPING STONE (BATU LONCATANA) REVISI.pptx
METODE STEPPING STONE (BATU LONCATANA) REVISI.pptx
 
460012937-Rpp-kelas-rangkap-model-221-docx.docx
460012937-Rpp-kelas-rangkap-model-221-docx.docx460012937-Rpp-kelas-rangkap-model-221-docx.docx
460012937-Rpp-kelas-rangkap-model-221-docx.docx
 

Ekonomi Mineral by Yuli Kusumawati

  • 1. Yuli Kusumawati, S.T., M.T. CATATAN KULIAH EKONOMI MINERAL
  • 2. CATATAN KULIAH EKONOMI MINERAL YULI KUSUMAWATI, S.T., M.T.
  • 3. Allah, tidakada Tuhan selain Dia. YangMaha Hidup, yangterus-menerus mengurus (makhluk-Nya), tidakmengantukdan tidaktidur. Milik-Nya apa yangada di langit dan apa yangada di bumi. Tidakada yangmemberi syafaat di sisi-Nya tanpa izin-Nya. Dia mengetahui apa yangada di hadapan mereka dan apa yangada di belakangmereka dan mereka tidakmengetahui sesuatu apapun tentangilmu-Nya melainkan apa yangDia kehendaki. Kursi-Nya meliputi langit dan bumi. Dan Dia tidakmerasa berat memelihara keduanya, dan Dia Maha Tinggi, Maha Besar. (QS. Al Baqarah : 255)
  • 4. iii DAFTAR ISI 1. DEFINISI DAN KONSEP DASAR 1 1.1. Ekonomi teknik dan pengambilankeputusaninvestasi 1 1.2. Karakteristikindustri pertambangan 2 1.3. Konsep nilai waktu uang 3 1.4. Konsep kesetaraan 4 1.5. Aliran kas 5 2. KONSEP BIAYA 6 2.1. Pengertian biaya 6 2.2. Klasifikasi biaya 6 2.2.1. Biaya berdasarkanwaktu 6 2.2.2. Biaya berdasarkansifat penggunaannya 6 2.2.3. Biaya berdasarkanproduk 7 2.2.4. Biaya berdasarkanvolume produksi 8 2.3. Biaya dalam usahapertambangan 9 2.3.1. Biaya kapital 9 2.3.2. Biaya operasi 10 3. KONSEP DAN RUMUSAN BUNGA 12 3.1. Bunga sederhana dan bungamajemuk 12 3.2. Suku bunga nominal dan suku bungaefektif 13 3.3. Minimum Attractive RateofReturn (MARR) 13 3.4. Waktu lipat dua 14 3.5. Rumus bunga majemuk 14 3.5.1. Single payment compound-amount 14 3.5.2. Single payment present-worth 15 3.5.3. Uniform series compound-amount 15 3.5.4. Sinking-fund deposit 15 3.5.5. Capital-recovery 16 3.5.6. Uniform series present-worth 16 3.5.7. Uniform gradient-series 16 4. INDIKATOR KELAYAKAN INVESTASI 18 4.1. Net Present Value (NPV) 18 4.2. Internal Rate ofReturn(ROR) 19 4.3. Benefit-Cost Ratio (BCR) 19 4.4. Present Value Ratio (PVR) 20 4.5. PaybackPeriod (PBP) 20 5. ANALISIS MUTUALLY EXCLUSIVE 22 5.1. Analisis mutually exclusive dengan umur alternatifsama 22 5.2. Analisis mutually exclusive dengan umur alternatifberbeda 24 6. ANALISIS NON MUTUALLY EXCLUSIVE 27 6.1. Analisis non-mutually exclusivedenganumur alternatifsama 27 6.2. Analisis non-mutually exclusivedenganumur alternatifberbeda 29 7. DEPRESIASI,DEPLESI, AMORTISASI 32 7.1. Depresiasi 32 7.1.1. Straigth line depreciation (SLD) 33
  • 5. iv 7.1.2. Double decliningbalance depreciation (DDBD) 34 7.1.3. Sum of years digits depreciation (SOYD) 35 7.1.4. Sinkingfund depreciation (SFD) 36 7.1.5. Unit ofproduction depreciation (UPD) 38 7.2. Deplesi 39 7.2.1. Cost depletion 40 7.2.2. Percentage depletion 40 7.2.3. Pemilihan metode deplesi 41 7.3. Amortisasi 41 8. PENGARUH PAJAK DALAM ANALISIS LIFE CYCLE COSTING 43 8.1. Pengertian pajak 43 8.2. Cashflow setelah pajak 44 9. ANALISIS PENGGANTIAN 53 9.1. Konsep penggantian 53 9.2. Umur ekonomis 53 9.3. Membandingkansewaatau beli 54 10. ANALISIS RESIKO DAN KETIDAKPASTIAN 59 10.1. Analisis sensitiv itas 59 10.1.1. Analisis sensitiv itas variabel tunggal 60 10.1.2. Analisis sensitiv itas dengan pendekatan rentang 62 10.2. Analisis probabilitas 62 10.2.1. Teori probabilitas 62 10.2.2. Pohon keputusan 63
  • 6. v PENGANTAR Alhamdulillahirabbil’alamiin, berkat kuasa dan kasih sayang ALLAH SWT akhirnya Catatan Kuliah Ekonomi Mineral ini bisa diselesaikan. Catatan Kuliah Ekonomi Mineral ini diperuntukkan bagi mahasiswa teknik tingkat diploma maupun sarjana yang ingin mendapatkan pemahaman mengenai prinsip-prinsip dasar dan metodologi ekonomi teknik dalam proses pengambilan keputusan investasi pada kegiatan perencanaan dan pengembangan proyek-proyek keteknikan, khususnya pada industri pertambangan dan pengelolaan sumberdayamineral. Materi dalam catatan kuliah ini merupakan rangkuman dari berbagai referensi yang disajikan secara ringkas dan mudah dipahami namun tetap mencakup esensi dari setiap teori yang berkaitan dengan ekonomi teknik. Disampingitu disajikan pula contoh soal sederhanauntukmembantu pemahaman materi. Mengingat keterbatasan yang ada, maka catatan kuliah ini masih banyak kekurangan dan perlu perbaikan. Oleh karena itu masukan dan saran sangat diharapkan untuk penyempurnannya. Tak lupa penyusun menghaturkan terima kasih kepada semua pihak yang telah membantu dalam menyelesaikan catatan kuliah ini. Selain itu penghargaan yang besar penyusun berikan kepada suami dan anak-anak yang tidak surut atas memberikan doa, semangat, dan dukungan. Mudah-mudahan catatan kuliah ini menjadi amal kebaikan penyusun dan dapat memberikan manfaat yang sebesar-besarnya. Bandung, Juli 2014 Yuli Kusumawati S.T., M.T.
  • 7. Yuli Kusumawati, Catatan Kuliah Ekonomi Mineral - 1 1. DEFINISI DAN KONSEP DASAR 1.1. Ekonomi teknik dan pengambilan keputusan investasi Ekonomi teknik adalah ilmu yang berkaitan dengan teknik analisis kuantitatif yang berguna untuk memilih satu alternatifyanglebih baikdari beberapaalternatifyangsecarateknis layak. Prosedur dalam analisis ekonomi teknikadalah: 1. Pengenalan masalah, perumusan, dan evaluasi. 2. Merancangalternatifyanglayak. 3. Menyusun net cashflow dari masing-masingalternatif. 4. Pemilihan kriteria. 5. Menganalisis danmembandingkan semuaalternatif. 6. Memilih alternatifyangtepat. 7. Memonitor kinerja dan mengevaluasi hasilnya. Aplikasi prinsip-prinsip ekonomi teknik tidak hanya berguna dalam analisis kelayakan dari aspek finansial terhadap proyek-proyek keteknikan tetapi juga membantu dalam pengambilan keputusan investasi pada umumnya. Investasi merupakan penukaran sejumlah dana dengan kemungkinan perolehan 100% (karena telah dikuasai) dengan jumlah dana yang lebih besar tetapi kemungkinan perolehannya kurang dari 100%. Investasi diperlukan untuk memulai suatu usaha atau mempertahankan dan meningkatkan kapasitas produksi suatu usaha yang sedangberjalan. Inv estasi memiliki dua faktor, yaitu: 1. Waktu 2. Resiko Tujuan dilakukannya investasi adalah untuk memperoleh nilai lebih/keuntungan di masa depan dari modal yang diinvestasikan. Modal yang dimaksud dapat berupa uang, barang modal, tanah, bangunan, teknologi, ataupun sesuatu yang tidak riil, misalnya hak paten atau kemampuan manajerial. Dalam bidang pertambangan, kapital umumnyaberupadeposit bahan tambangdan modal. Keputusan investasi modal akan mempunyai dampak jangka pendek dan jangka panjang bagi kelangsungan perusahaan untuk dapat berkompetisi ataupun untuk tetap berproduksi. Keputusan investasi yang salah tidak saja dapat mengurangi keuntungan perusahaan tetapi juga dapat menghentikan kegiatan perusahaan sama sekali. Keputusaninvestasi modal mempunyai duabentuktindakan utama, yaitu : 1. Mengalokasikan sejumlah modal untukinvestasi proyektertentu atauuntukmenyediakan asset produksi. Tujuan:  Memahami konsep ekonomi teknikdan aplikasinyadalam pengambilan keputusan investasi.  Memahami karakteristikindustri pertambangan.  Memahami konsep nilai waktu uangdankesetaraan.  Memahami konsep aliran kas.
  • 8. Yuli Kusumawati, Catatan Kuliah Ekonomi Mineral - 2 2. Memperoleh sejumlah modal untukmeningkatkannilai perusahaan. Ada lima langkah pentingdalam pengambilan keputusan investasi, yaitu : 1. Mendefinisikan masalah 2. Menganalisis masalah 3. Mengembangkan alternatifsolusi 4. Memutuskan solusi yangterbaik 5. Mengubah keputusanmenjadi tindakanyangefektif Analisis inv estasi diklasifikasikan menjadi tiga, yaitu : 1. Analisis ekonomi Evaluasi terhadap kemakmuran relatifdari situasi-situasi investasi dari sudut pandang laba dan ongkos, yaitu studi ev aluasi terhadapkeuntungandari alternatifproyek. 2. Analisis finansial Analisis finansial adalah evaluasi terhadap cara pendanaan terhadap investasi yang diusulkan. Terdapat beberapa alternatif metode untuk pendanaan, di antaranya adalah dengan dana pribadi atau perusahaan, meminjam dari bank, atau menawarkansaham padapublik. Analisis finansial terbagi lagi menjadi tiga pengambilan keputusan penting yang harus dilakukan oleh sebuah perusahaan,yaitu : a. Keputusandeviden b. Keputusankeuangan c. Keputusaninvestasi Ketiga keputusan ini salingterkait satu sama lain dan mempengaruhi analisis inv estasi secara keseluruhan. 3. Analisis intangible Evaluasi terhadap faktor-faktor yang mempengaruhi investasi tetapi sukar diukur secara kuantitatif. Contoh faktor intangible antara lain perijinan, pertimbangan keamanan, opini publik, pertimbangan politik, faktor ekologi dan lingkungan, danketidakpastiankondisi peraturan pajak. Ketiga jenis analisis investasi ini mutlak harus dilakukan, karena sering terjadi suatu alternatif yang hasil analisis ekonominya baik, ternyata tidak cukup baik jika dilihat dari analisis finansial dan intangible-nya. Sebagai contoh, suatu proyek ditolak dari analisis finansial bila dana internalnya tidak mencukupi untuk membiayai proyek dan juga tidak dapat memperoleh pinjaman dana dari luar dengan tingkat bunga yang sesuai. Faktor intangible yang dapat mengakibatkan suatu proyek ditolak secara analisis ekonomi pada umumnya adalah karena opini publik dan masalah lingkungan (polusi udara, tanah, dan air). Karenanya pengaruh analisis finansial dan intangible terhadap analisis ekonomi harus diperhatikan secara seksama karena analisis-analisis tersebut mempunyai pengaruh yangsangat besar terhadap pemilihan alternatifinvestasi. 1.2. Karakteristik industri pertambangan Industri pertambangan merupakan bagian dari dunia industri umum namun industri pertambangan mempunyai karakteristik khusus yang berbeda dengan industri lainnya. Pemahaman tentang karakteristik khusus ini penting untukmelakukan analisis kelayakansuatuproyek/investasi tambang. KarakteristikIndustri pertambangan antara lain: 1. Modal Besar Besarnya modal yang dibutuhkan untuk industri tambang bervariasi, tergantung dari jenis bahan tambang, metode penambangan, skala penambangan, lokasi dan parameter lainnya.
  • 9. Yuli Kusumawati, Catatan Kuliah Ekonomi Mineral - 3 2. Periode Pra Produksi yangPanjang Lama periode pra produksi tergantungdari metode penambangan, metode pengolahan, ukuran dan letak deposit, kompleksitas operasi, dan kendala lingkungan. Periode pra produksi ini berkisar antara 3 – 12 tahun. Periode pra produksi yang panjang akan berdampak terhadap besar modal yang dibutuhkan dan terhadaptingkat pengembalian modal. 3. Beresiko Tinggi Disamping resiko yang berhubungan dengan kebutuhan modal yang besar serta masa pra produksi yang lama, terdapat resiko lain yang mempengaruhi keputusan investasi pada industri tambang, yaitu: resiko geologi, resiko engineeringdan konstruksi, reiko ekonomi, resiko politik, dan resiko pasar mineral. 4. Sumber daya alam yangtakdapat diperbaharui (nonrenewable resources) Implikasi dari sumber daya alam yang tidak dapat diperbaharui ini adalahterhadap pendapatan utama perusahaan yang diperoleh dari penjualan bahan tambang, yang mengakibatkan umur tambang tergantung dari jumlah cadangan dan tingkat produksi sehingga dibutuhkan eksplorasi kontinyu untuk menemukan deposit baru. 5. Mendorongpertumbuhan ekonomi Dikarenakan letak aktivitas penambangan banyak terdapat di daerah terpencil, hal ini akan dapat memberikan dampak positif terhadap aktivitas ekonomi masyarakat setempat sehingga dapat mendorong pertumbuhan ekonomi di daerahtersebut. 6. Dampakterhadap lingkungan Kegiatan eksploitasi bahan tambangakan mengubah bentangalam sehingga berdampak buruk terhadap keadaan lingkungan. Oleh karena itu tingkat kepedulian industri tambang terhadap lingkungan harus tinggi. Reklamasi merupakan salah satu upaya untuk mengurangi dampak lingkungan dari kegiatan penambangandanpengolahan. 7. Sifat indestructibility ofproduct Konsekuensi dari sifat ini adalah munculnya pasar sekunder dan dapat mengurangi prosentase kebutuhan akan bijih/bahan tambang. Daur ulang logam sering dipertimbangkan lebih menguntungkan dibandingkan menambangbijih untukdijadikan logam. 1.3. Konsep nilai waktu uang Konsep nilai waktu uang (time value of money) mengacu pada biaya kesempatan (opportunity cost) dalam menghasilkan pendapatan pada saat sekarang. Bila sejumlah dana tidak digunakan untuk membiayai suatu proyek, maka dana tersebut akan dapat digunakan untuk membiayai proyek lain. Dengan kata lain jika suatu dana ditanamkan seluruhnya pada suatu proyek maka akan menghilangkan kesempatan untuk mendapatkan penghasilan (return) dari proyeklain yangtidakdipilih. Nilai sesuatu (termasuk uang) berubah seiring dengan berjalannya waktu. Perjalanan waktu pada umumnya mengurangi nilai kecuali pada benda antik dan benda yang mempunyai umur layanan (service life) tak terbatas, seperti tanah. Beberapa hal yangmenyebabkan nilai uangdipengaruhi oleh waktu adalah: 1. Uang memiliki purchasingpower (dayabeli). Secara umum uang digunakan untuk melakukan transaksi, sehingga semua komoditi harus dinilai berdasarkan nilai tukarnyaterhadapuang. Daya beli uangmenurun disebabkan oleh:  Inflasi  Perubahan polasupply(pasokan) dandemand (permintaan)
  • 10. Yuli Kusumawati, Catatan Kuliah Ekonomi Mineral - 4  Perubahan struktur ekonomi Sebagai contoh harga premium sekarang Rp 6.500/liter, artinya 1 liter premium nilainya Rp 6.500.Beberapa tahun yang lalu harga premium masih Rp 4.500/liter, atau dikatakan nilai uang mengalami penurunan terhadappremium. 2. Uangmemiliki earningpower (kemampuan menghasilkan). Sejumlah uang yang disimpan di bank akan bertambah dengan adanya bunga. Sebagaimana meminjam barang, bunga(interest) dianggapsebagai sewaatas pemakaianuangtersebut. 1.4. Konsep kesetaraan Konsep kesetaraan (equivalence) menyatakan bahwa sejumlah uang pada satu waktu tertentu setara nilainya dengan uang dalam jumlah yangberbeda pada waktu yang berbeda apabila diberikan discount rate tertentu. Konsep kesetaraan dapat digunakan untuk mengevaluasi rencana investasi. Untuk keperluan tersebut maka seluruh pendapatan dan pengeluaran proyek di masa yang akan datang dihitung nilainya pada saat sekarang (present worth). Sebagai contoh uang sejumlah Rp 10 juta yang disimpan di bank dengan bunga 20%/tahun dapat diambil dalam beberapacara, antaralain: a. Disimpan hari ini dan baru diambil tahun depan dengan jumlah Rp 12 juta, dengan rincian pokok simpanan Rp 10 juta ditambahbungaRp 2 juta. b. Diambil bunganya saja Rp 2 juta tiap tahun selama lima tahun, dan pada akhir tahun kelima pokok simpanannya diambil. Jumlah uang yang diperoleh selama lima tahun tersebut adalah Rp 20 juta, dengan rincian bunga Rp 2 juta x 5 = Rp 10 juta dan pokoksimpanan Rp 10 juta. c. Diambil dalam jumlah yang sama setiap tahunnya selama sepuluh tahun sebesar Rp 2,385 juta. Jumlah uangyangdiperoleh selama sepuluhtahun tersebut adalah Rp23,85 juta, yaituRp 2,385 juta x 10. Ketiga jumlah nominal uang tersebut yaitu Rp 12 juta, Rp 20 juta, dan Rp 23,85 juta pada dasarnya setara dengan Rp 10 juta padasaatsekarang. Kesetaraantersebut terjadi dengan penggunaanbunga 20% per tahun. 1.5. Aliran kas Aliran kas (cashflow) adalah aliran uang yang terdiri atas cash-in (uang masuk) dan cash-out (uang keluar) yang terjadi selama periode operasi tertentu, misalnya bulan atau tahun. Cashflow digunakan untuk menggambarkan pergerakanuangsuatu perusahaanpada satuan periode tertentu. Aliran kas terdiri dari:  Uangmasuk(cash in), umumnya berasal dari penjualan produkatau manfaat(benefit) yangterukur.  Uangkeluar (cash out), merupakankumulatifdari biaya-biaya(cost) yangdikeluarkan. Net cashflow adalah jumlah uang masuk dikurangi jumlah uang keluar. Net cashflow yang negatif artinya defisit, yaitu cash-out lebih besar daripada cash-in. Discounted cashflow adalah nilai cashflow yang dibawa ke nilai sekarang (present worth) sesuai dengan discount rate yang diberikan. Istilah discount mengacu pada pengurangan nilai, yaitu jika sejumlah tertentu uangdimasa datangdihitungnilainyasekarangmaka nilai uangtersebut akanlebih kecil (berkurang). Cashflow dapat digambarkan dalam bentuk: 1. Tabel yangterdiri palingsedikit tiga kolom, yaitu kolom waktu, kolom pengeluaran, dankolom pendapatan.
  • 11. Yuli Kusumawati, Catatan Kuliah Ekonomi Mineral - 5 2. Diagram yang ditunjukkan dengan anak panah yang mengarah ke atas untuk cash-in dan anak panah yangmengarah ke bawahuntukmenggambarkancash-out. Contoh 1-1: Pada tahun awal (tahun ke-0) biaya yang dikeluarkan adalah Rp 10 juta. Tahun ke-1 dikeluarkan lagi biaya Rp 2 juta selain diterima pendapatan (revenue) Rp 3 juta. Pada tahun ke-2, 3, 4, dan 5 diperoleh pendapatan masing-masing Rp 5 juta. Gambarkan undiscounted cashflow perusahaan tersebut dalam bentuk tabel dan diagram. Jawab: Tabel undiscountedcashflow: Tahun Pendapatan(cash-inflow) Pengeluaran (cash-outflow) Net cashflow CummulativeCashflow 0 0 10.000.000 -10.000.000 -10.000.000 1 3.000.000 2.000.000 1.000.000 -9.000.000 2 5.000.000 0 5.000.000 -4.000.000 3 5.000.000 0 5.000.000 1.000.000 4 5.000.000 0 5.000.000 6.000.000 5 5.000.000 0 5.000.000 11.000.000 Jumlah 23.000.000 12.000.000 11.000.000 Diagram cashflow: Dari tabel di atas terlihat pada akhir tahun ke-5, proyek tersebut menghasilkan keuntungan bersih sebesar Rp 11.000.000. Secara umum cash flow dikelompokkan menjadi: 1. Cashflow uniform, adalah cashflow dimana besar cash-in dan cash-out tiap periode adalah seragam (tetap). 2. Cashflow gradient uniform, adalah cashflow dimana besar cash-in atau cash-out bertambah atau berkurangdengan gradien tetap. 3. Cashflow seri geometrik, adalah cashflow dimana besar cash-in atau cash-out bertambah atau berkurang dengan presentase tetap. 10 2 3 4 5 10.000.0000 2.000.000 3.000.000 5.000.000 5.000.000 5.000.000 5.000.000
  • 12. Yuli Kusumawati, Catatan Kuliah Ekonomi Mineral - 6 2. KONSEP BIAYA 2.1. Pengertian biaya Biaya adalah jumlah uang yang harus dikeluarkan untuk memproduksi sesuatu (cost of production) atau harga yangharus dikeluarkan untukmendapatkansesuatu(supply price). Konsep biaya dalam analisis ekonomi teknik perlu dipahami karena semua analisis yang dilakukan didasarkan atas perkiraan arus kas yang dikembangkan berdasarkan perkiraan biaya (cost estimate) dan proyeksi pendapatan(revenue projection). 2.2. Klasifikasi biaya Biaya dapatkelompokkan berdasarkanklasifikasi berikut (Giatman, Ekonomi Teknik): a. Berdasarkan waktunya, dibedakanmenjadi:  Biaya masa lalu  Biaya perkiraan  Biaya sebenarnya b. Berdasarkan sifat penggunaannya, dibedakanmenjadi:  Biaya inv estasi  Biaya operasi  Biaya perawatan c. Berdasarkan produknya, dibedakanmenjadi:  Biaya produksi/pabrikasi  Biaya komersial d. Berdasarkan v olume produk, dibedakanmenjadi:  Biaya tetap  Biaya tidaktetap  Biaya semi v ariable 2.2.1. Biaya berdasarkan waktu Biaya berdasarkanwaktu dibedakanmenjadi: 1) Biaya masa lalu (hystorical cost), yaitu biaya yang secara riil telah dikeluarkan yang dibuktikan dengan catatan historis pengeluarankegiatan. Biaya historis digunakan sebagai dasar untuk:  Penyusunan/perkiraan biayamasa datang.  Pertanggungjawabanatau audit terhadapbiaya-biayayangtelah dikeluarkan. 2) Biaya perkiraan (predictive cost), yaitu perkiraan biaya yang akan dikeluarkan bila suatu kegiatan dilaksanakan. Biaya perkiraan digunakan untuk:  Memperkirakan pemakaianbiayadalam merealisasikan suatukegiatan.  Menganalisis efektivitas dan efisiensi biaya terhadapkegiatanyangakandilaksanakan. Tujuan:  Memahami konsep biayadan klasifikasinya secara umum.  Mampu menghitungbiaya produksi denganmetode yangsesuai.  Mampu menganalisis biayaproduksi sehingga bisa memperbaiki efisiensinya.  Mengetahui komponen biaya yangadadalam usaha pertambangan.
  • 13. Yuli Kusumawati, Catatan Kuliah Ekonomi Mineral - 7 3) Biaya aktual (actual cost), yaitu biaya yang sebenarnya dikeluarkan. Biaya ini perlu diperhitungkan jika rentang waktu antara pembelian dengan proses produksi atau penjualan cukup lama sehingga terpengaruh dengan perubahan harga pasar. Biaya aktual dibedakanmenjadi:  Biaya riil (expense), yaitu biaya yangdikeluarkansecara riil.  Biaya semu (sunk cost), yaitu biaya yang ditanggung tetapi tidak pernah dikeluarkan secara riil. Misalnya, selisih harga pembukuanaset yangakandilikuidasi denganharga pasar.  Biaya kesempatan (opportunity cost), yaitu biaya yang ditanggung akibat kelalaian dalam memanfaatkanpeluanguntukmendapatkan keuntungan yanglebih besar. Metode perhitunganbiayaaktual antaralain:  First in first out (FIFO)  Last in first out (LIFO)  Metode rata-rata(average method)  Metode harga standar (standard price method) 2.2.2. Biaya berdasarkan sifat penggunaannya Biaya berdasarkansifat penggunaannyadikelompokkan menjadi tiga, yaitu: 1) Biaya investasi (investment cost), yaitu biaya yang ditanamkan untuk membangun dan mengembangkan suatu usaha sehingga siap beroperasi. Biaya ini bisanya dikeluarkan pada awal kegiatan dengan jumlah yangrelatifbesar dan mempunyai dampakjangka panjanguntukkeberlangsunganusahatersebut. Contoh:  Pembelian tanah atauhakpenggunaan lahan.  Pembangunanpabrik, gedung, dan infrastuktur pendukung.  Pengadaan mesin danperalatan produksi.  Pengadaan alat transportasi  Pengadaan perabotan kantor dan sarana pendukungadministrasi.  Pendidikan dan pelatihan sumberdayamanusia. 2) Biaya operasi (operational cost), yaitu biaya yang dikeluarkan untuk menjalankan kegiatan usaha. Biaya ini biasanya dikeluarkan secara rutin atau berkala dalam jumlah yang relatif sama sesuai dengan jadwal kegiatan/produksi. Biaya operasi digunakan antara lain untuk:  Pembelian bahanbaku  Pembelian bahanpendukung  Pembayaran upahkaryawan  Pengeluaran untukkeperluanorganisasi danadministrasi. 3) Biaya perawatan (maintenance cost), yaitu biaya yang digunakan untuk menjamin kinerja fasilitas atau peralatan agar selalu prima dan siapuntukdigunakan. Sifat pengeluaranini umumnya dibedakan menjadi dua, yaitu: a. Biaya perawatan rutin (preventive maintenance) b. Biaya perawatan mendadak(curative maintenance) 2.2.3. Biaya berdasarkan produk Biaya berdasarkanproduk dikelompokkan menjadi dua, yaitu: 1) Biaya produksi (production cost) atau biaya pabrikasi (factory cost), yaitu biaya yang langsung berkaitan dengan proses produksi. Biaya produksi terdiri dari komponen berikut:
  • 14. Yuli Kusumawati, Catatan Kuliah Ekonomi Mineral - 8 a. Biaya utama (prime cost), yaitu biaya untuk bahan langsung dan tenaga kerja langsung yang berhubungan denganproses produksi. b. Biaya overhead, yaitu biaya untuk bahan tak langsung, tenaga kerja tak langsung, dan biaya tak langsunglainnya. c. Biaya konversi (conversion cost), yaitu biaya pengubahan bahan bahan baku menjadi barang jadi. Biaya bn biaya overheaddapat digabungke dalam kelompokbiaya konversi. 2) Biaya komersial (commercial cost) Biaya komersial adalah biayadi luar biayaproduksi yangdigunakan untukkeperluanpenjualan produk. Biaya komersial meliputi: a. Biaya umum dan administrasi (general and administration cost), contoh gaji karyawan dan pimpinan, biaya untukkeperluanadministrasi, dantelekomunikasi. b. Biaya pemasaran(marketingcost), meliputi biayadistribusi, danpromosi. c. Pajak usaha dan perusahaan (company tax), meliputi semua pajak dan retribusi yang dikeluarkan perusahaan. Tujuan perhitunganbiaya berdasarkan produkantaralain:  Memproyeksikan biayaproduksi danharga jual produk.  Mengetahui komposisi komponen biaya produksi dan biayakeseluruhanproduk.  Untuk menganalisis struktur biaya produk yang ideal guna memperbaiki struktur pembiayaan melalui konsep pusat-pusatpembiayaan. 2.2.4. Biaya berdasarkan volume produksi Biaya berdasarkanvolume produksi dibedakanmenjadi: 1) Biaya tetap (fixed cost), yaitu biaya yang dikeluarkan dengan jumlah yang relatif sama walaupun volume produksi berubah dalam batas tertentu. Contoh:gaji karyawan tetap, biaya penyusutan, pajak bumi dan bangunan, biaya sewa, biayalistrikuntukpenerangan, telepon,dan air. 2) Biaya tidak tetap (variable cost), yaitu biaya yang besarnya berubah secara proporsional dengan jumlah produk yang dibuat. Contoh: biaya bahan baku, tenaga kerja langsung yang digaji berdasarkan v olume kerja, biaya penyusutan yangtergantungdenganvolume produksi. 3) Biaya semi variable (semi variable cost), yaitu biaya yang berubah tidak proporsional terhadap volume produksi. Contoh: perubahan volume produksi yang melampaui kapasitas mesin, sehingga diperlukan biaya penambahan kapasitas mesin, atau biayaperbaikan mesin. Contoh 2-1: PT Insan Kreatif mendapat pesanan bangku untuk suatu sekolah. Di pasaran harga bangku (S) sesuai dengan spesifikasi teknis yang diajukan sekolah itu adalah Rp 450.000/unit. Jika bangku itu dibuat sendiri maka diperlukan biaya-biayasebagai berikut: Harga kayu = Rp 950.000/m3 Paku = Rp 20.000/kg Bahan finishing = Rp 100.000/lt Upah tukang = Rp 80.000/orang/hari Upah pembantutukang = Rp 50.000/orang/hari Satu unit bangku membutuhkan 0,20 m3 kayu, 0,01kg paku, bahan finishing 0,1lt, dan diselesaikan selama 0,5 hari kerja oleh tukang ditambah1 hari kerja oleh pembantunya.Selain itu perusahaan memerlukan bengkel kerja dengan biayasekitar Rp 1,5 juta dan pengadaan peralatankerja Rp 2,2 juta.
  • 15. Yuli Kusumawati, Catatan Kuliah Ekonomi Mineral - 9 Kelompokkan biaya berdasarkan biaya tetap dan biaya variable. Hitunglah berapa jumlah minimal pesanan bangku yangharus diterima perusahaan untukmencapai titikimpas (breakev enpoint/BEP)? Jawab: Biaya tetap: Membangun bengkel kerja = Rp 1.500.000 Pengadaan alat kerja = Rp 2.200.000 Jumlah fixed cost (FC) = Rp 3.700.000 Biaya v ariable per unit bangku: Bahan kayu = 0,2 m3 x Rp 950.000/m3 = Rp 190.000 Paku = 0,01kg x Rp 20.000/kg = Rp200 Bahan finishing = 0,1 lt x Rp 100.000/lt = Rp 10.000 Upah tukang = 0,5 hari x Rp 80.000/orang/hari = Rp 40.000 Upah pembantutukang = 1 hari x Rp 50.000/orang/hari = Rp 50.000 Jumlah v ariable cost (VC) = Rp 290.200 Jumlah produksi minimal untukmencapai titikimpas (BEP) adalah: 𝐁𝐄𝐏 = 𝐅𝐂 𝐒 − 𝐕𝐂 BEP = Rp 3.700.000 Rp 450.000− Rp 290.200 = 23,15 ≈ 24 unit 2.3. Biaya dalam usaha pertambangan Secara umum biaya dalam usahapertambangan dibedakan menjadi dua, yaitu: 1. Biaya kapital (biayainvestasi), terdiri dari duakomponen yaitu: 1) Biaya kapital tetap 2) Modal kerja 2. Biaya operasi, terdiri dari tiga komonen yaitu: 1) Biaya operasi langsung 2) Biaya operasi taklangsung 3) Biaya ov erhead 2.3.1. Biaya kapital Biaya kapital (capital cost) adalah jumlah biaya yang dibutuhkan untuk membuat suatu endapan bahan galian yangberada di dalam bumi menjadi produk tambang yangdapat dijual. Biaya kapital terdiri dari dua komponen penting, yaitu: 1) Biaya kapital tetap (initial investment) Jika tambang yang akandikerjakan merupakan tambang baru, maka biaya tetap biasanya terdiri dari komponen-komponen berikut:  Land Acquisition (pembebasanlahan), biayanyatergantungkepada luas dan lokasi lahan.  Konstruksi pra-penambangan(pengupasantanahpenutup, dan sebagainya).  Pembangunantambang/masa konstruksi  Analisa DampakLingkungan.  Peralatan tambang, bangunan, saranalain.  Peralatan pabrik, bangunan, sarana lain.  Sarana penunjang(jalan, listrik, perumahan, sarana olahraga, instalasi air, dansebagainya).  Jasa perancangan dan konsultasi.
  • 16. Yuli Kusumawati, Catatan Kuliah Ekonomi Mineral - 10  Contingency. 2) Modal kerja (workingcapital). Modal kerja adalah biaya yang digunakan untuk memulai produksi sebelum perusahaan mendapatkan uang dan hasil penjualan produknya. Besarnya modal kerja umumnya adalah 25% dari biaya operasi atau mencukupi kebutuhan operasi selama 3-6 bulan. Modal kerja umumnya terdiri dari komponen-komponen biaya sebagai berikut: a. Persediaan  Bahan baku, berupa dimana cadangan endapan mineral/bijih yang ekonomis yang belum dilaksanakan proses penambangan.  Suku cadang, yangberguna apabilaterjadi ker usakan padaalat-alat penambangan.  Supplies, merupakan stock dari suatu perusahan tambang yang berupa perlengkapan habis sekali pakai seperti perlengkapankantor, bahan bakar, bahanpelumas, dll.  Bahan dalam proses (materialsin process), berupa endapan mineral/bijih yang sedang atau dalam proses penambangan ataudalam proses pengolahan (mineral dressing).  Bahan jadi/produk tambang, merupakan bahan galian/bijih yang telah melalui proses pengolahan yangsiap dijual. b. Piutangdagang Piutang dagang merupakan suatu modal kerja yang dapat ditarik sewaktu-waktu dari pihak kedua sesuai dengan perjanjian dagang (seperti pembayaran diakhir transaksi penjualan komoditas dagang). c. Hutangdagang Merupakan modal kerja yang diperoleh dari or ang lain dalam bentuk pinjaman yang bernilai ekonomis, yang harus dibayar oleh kita apabila telah jatuh tempo sesuai dengan perjanjian daga ng kedua belah pihak d. Kas, dan lain-lain. Merupakan cadangan uang yang disimpan yang berguna untuk membiayai kehidupan tambang sehari-hari, dengan periode waktu yangrelatifsingkat. 2.3.2. Biaya Operasi Biaya operasi(operating cost) adalah segala macam biaya yang harus dikeluarkanagar proyek penambangan dapat beroperasi/berjalan dengan normal. Dalam suatu operasi penambangan, keseluruhan biaya penambangan terdiri dari banyak komponenbiaya yang merupakan akibat dari masing-masing tahap kegiatan. Besar kecilnya biaya penambanganakantergantung pada perancangan teknis sistem penambangan, jenis dan jumlahalat yangdigunakan. Aspek teknis dan aspek ekonomis tidak dapat berjalan sendiri-sendiri, keduanyaakanselalu saling mempengaruhi. Perkiraan biaya investasi alatakan tergantung pada jumlah alat yang dipergunakan dan kapasitas alat yang dipilih. Demikian pula biaya produksi merupakan fungsi dari kapasitas alat yang dipakai. Biaya penambangan yang rendah dapat dicapai jika rancangan teknis dapat dioptimalkan dengan memperhatikan pemilihan dan jumlah alat yang akan digunakan, yaitu yang dapat memberikan biaya produksi per ton yangrendah. Secara umum biaya operasi dibagi menjadi tiga komponen biaya, yaitu: 1) Biaya operasi langsung
  • 17. Yuli Kusumawati, Catatan Kuliah Ekonomi Mineral - 11 Biayaoperasi langsung merupakan biaya utama dan berkaitan langsungdenganproduk yang dihasilkan. Walaupun komponen biaya operasi langsung dari satu tambang ke tambang yang lain bervariasi, akan tetapi padaumumnyaterdiri dari:  Upah pekerja (pekerja lapangan, pengawas lapangan, dan sebagainya)  Bahan bakar (bahan bakar, oli, dan sebagainya)  Royalti.  Persiapan daerah produksi/permukaan kerja 2) Biaya operasi taklangsung Biaya operasi tak langsung adalah pengeluaran-pengeluaran yang tak terpengaruholeh produksi yang dihasilkan. Biaya operasi langsung terdiri dari:  Gaji pekerja (administrasi, keamanan, teknisi, jurubayar, petugas kantor, bengkel dan sebagainya).  Asuransi.  Penyusutanalat.  Pajak.  Reklamasi daerah bekas tambang.  Perjalanan bisnis, rapat, sumbangan-sumbangan.  Keperluan kantor.  Humas dan sebagainya. 3) Biaya ov erhead Biaya overhead dapat/tidak dapat dimasukkan sebagai ko mponen biaya operasi tetapi biaya-biaya ini berpengaruh terhadap total biaya produksi walaupun umumnya mencerminkan biaya-biaya diluar tambang/biaya-biayaperusahaan. Overheadbiasanya dikelompokkanmenjadi: a. Penjualan. b. Administrasi kantor pusat.
  • 18. Yuli Kusumawati, Catatan Kuliah Ekonomi Mineral - 12 3. KONSEP DAN RUMUSAN BUNGA 3.1. Bunga sederhana dan bunga majemuk Bunga (interest) adalah uang yang dibayarkkan atas penggunaan sejumlah pinjaman. Bunga juga bisa berarti uang yang diperoleh dari investasi sejumlah modal tertentu sebagai bentuk kompensasi resiko yang timbul dari inv estasi tersebut. Tingkat/suku bunga (interest rate) adalah rasio antara bunga yang dibayarkan terhadap pokok dalam suatu periode waktu. Periode bunga (interest period) adalahinterval waktuyangdijadikandasar dalam perhitunganbunga. Secara umum perhitungan bungadikelompokkan menjadi dua, yaitu: 1. Bunga sederhana (simple interest), yaitu bunga yang dihitung dari pokok uang/pinjaman selama periode pinjaman. I = P i n Dimana: I = bunga (interest) P = pokok (principal) i = tingkat/suku bunga (interest rate) n = periode pinjaman Sehingga jumlah uang/pinjaman padaakhir periode (F) adalah: F = Pokok + Bunga = P + P i n = P (1 + i n) Contoh 3-1: Seorang petani meminjam uang di KUD sebesar Rp 10.000.000 dengan bunga 10%/tahun dengan perhitungan bungasederhana. Berapahutangyangharus dibayar pada akhir tahunkeempat? Jawab: F = Rp 10.000.000 x (1 +(10% x 4)) = Rp 14.000.000 2. Bunga majemuk (compound interest), yaitu bunga yang dihasilkan dari pokok pinjaman dibungakan lagi bersama-samadenganpokokpinjaman. I = P(1 + i)n-1 . i Sehingga jumlah uang/pinjaman padaakhir periode (F) adalah: F = Pokok + Bunga = P + P(1 + i)n-1 . i = P (1 + i)n Tujuan:  Memahami konsep bunga sederhanadan bunga majemuk.  Mampu menghitungsuku bunganominal, periode, danefektif.  Mampu menerapkan rumus bunga majemuk yang sesuai untuk menghitung kesetaraan nilai uangpada berbagai titikwaktu.
  • 19. Yuli Kusumawati, Catatan Kuliah Ekonomi Mineral - 13 Contoh 3-2: Seorang karyawan meminjam uang di Bank sebesar Rp 10.000.000 dengan bunga 10%/tahun dengan perhitungan bungamajemuk. Berapa hutangyangharus dibayar pada akhir tahun keempat? Jawab: F = Rp 10.000.000 (1 + 10%)4 = Rp 14.641.000 Dari kedua contoh di atas ternyata bunga majemuk nilainya lebih tinggi dari bunga sederhana. Dalam pembahasan pemilihanalternatifatauevaluasi rencanainvestasi digunakan bunga majemuk. 3.2. Suku bunga nominal dan suku bunga efektif Suku bunga nominal (nominal rate) per tahun (r) adalah perkalian suku bunga per periode (i) dengan jumlah pemajemukan dalam setahun (m). r = i. m Suku bunga efektif (effectiv e rate) disimbolkan E adalah suku bunga sesungguhnya dalam satu tahun yang tepat dibayarkanterhadapsejumlah uangyangdisimpan atau dipinjam. E = (1 + i)m -1 atau E = (1+ (r/m))m – 1 atau E = (F/P)1/n– 1 Contoh 3-3: Bank ABC membebankan suku bunga sebesar 1,25 % per bulan pada semua saldo rekening yang belum terbayar. Berapakah suku bunga nominal dansuku bunga efekti per tahunnya? Jawab: Suku bunga periode (i) = 1,25% per bulan Jumlah pemajemukan/th (m) = 12 Suku bunga nominal (r) = 1,25% x 12 = 15% per tahun Suku bunga efektif(E) = (1 + (0,15/12))12 – 1 = 16,18% per tahun Tabel 3.1. Tingkat suku bunga efektifper tahun dari berbagai suku bunga nominal. 3.3. Minimum Attractive Rate of Return (MARR) MARR adalah laju pengembalian minimum yang dikehendaki oleh investor.MARR tergantung pada lingkungan, jenis kegiatan, tujuan dan kebijakan organisasi, dan tingkat resiko masing-masingproyek. Pertimbangandalam menentukanMARR antaralain: a. Jika perusahaan beroperasi dengan modal pinjaman, maka besarnya MARR harus lebih besar daripada bunga yangdibayarkan padapinjamanitu.
  • 20. Yuli Kusumawati, Catatan Kuliah Ekonomi Mineral - 14 b. Jika modal datang dari berbagai sumber (dana-dana yangberasal dari perusahaan sendiri, hutang jangka panjang maupun jangka pendek, dan sumber-sumber keuangan lainnya), maka penentuan biaya modal rata-ratabisa dipakai sebagai basis untukmenentukan besarnyaMARR. c. Besarnya MARR harus setara dengan kecepatan pertumbuhan dari kekayaan perusahaan yang telah ditetapkanoleh pimpinan perusahaan. d. MARR untukproyekyangresikonya tinggi harus lebih besar daripada proyekyangresikonya rendah. e. Perusahaan yang lebih bonafide (lebih banyak kesempatan mendapatkan proyek) biasanya menetapkan MARR lebih tinggi daripadaperusahaansejenis yangbelum punyanama. 3.4. Waktu lipat dua Waktu lipat dua (doubling time) adalah waktu yang diperlukan agar uang yang ditanamkan hari ini menjadi dua kali lipat. Untukmemperkirakan waktuini dengan cepat digunakan angka ajaib(magic number) 72. Waktu lipat dua = 72 / (suku bunga majemuk per periode x 100) Akan tetapi penggunaan magic number ini akan salah jika periode pemajemukan kurangdari tiga (n≤3) atau suku bunga (i) di atas 30% Contoh 3-4: Berapa tahun uang yang disimpan di bank menjadi dua kali lipat jika suku bunga yang berlaku adalah 8% per tahun? Jawab: n = 72/(8% x 100) = 9 tahun 3.5. Rumus bunga majemuk Notasi dalam rumus bunga: i = suku bunga tiap periode bunga n = jumlah periode bunga P = nilai uangsekarang F = nilai uangmendatang A = pembayaranseri tahunan yangseragam G = pembayaranseri gradient 3.5.1. Single payment compound-amount Digunakan untuk menentukan nilai mendatang (F) dari sejumlah uang pada saat sekarang (P) selama periode (n) tertentu dan padatingkat suku bunga(i) tertentu. Contoh 3-5: Seorang ayah mendepositokan uang Rp 100 juta sekarang di bank dengan tingkat bunga 12%/ tahun. Uang tersebut akan diambil pada akhir tahun kelima, saat anaknya akan masuk universitas. Berapa jumlah uang yang akan diperoleh sangayah?
  • 21. Yuli Kusumawati, Catatan Kuliah Ekonomi Mineral - 15 Jawab: F = Rp 100 juta (1 + 0,12)5 = Rp 100 juta (F/P 12%, 5) = Rp 100 juta (1,7623)= Rp 176.230.000 3.5.2. Single payment present-worth Digunakan untuk menentukan nilai sekarang dari sejumlah uang yang diterima pada waktu mendatang selama periode tertentu dan tingkatbungatertentu. Contoh 3-6: Biaya masukSMA fav orit lima tahun mendatang diperkirakan Rp 30 juta. Untuk keperluan tersebut, seorang ayah berniat membuka tabungan pendidikan yang memberikan bunga 18% per tahun. Berapa uang yang harus disetorkan sekarang? Jawab: P = Rp 30 juta (1/(1 + 0,18)5) = Rp 30 juta (P/F 18%, 5) = Rp 30 juta (0,4371)= Rp 13.113.000 3.5.3. Uniform series compound-amount Digunakan untuk menentukan nilai mendatang (F) dari suatu rangkaian pembayaran yang seragam (A) yang terjadi pada setiapakhir periode tertentudantingkat bunga tertentu. Contoh 3-7: Seorang PNS menabung Rp 10 juta per tahun untuk keperluan investasi saat dia pensiun lima tahun yang akan datang. Jika suku bunga yang diberikan bank 6% per tahun, berapa total uang yang dia kumpulkan sampai akhir tahun ke-5? Jawab: F = Rp 10 juta ((1 + 0,06)5 – 1)/ 0,06 = Rp 10 juta (F/A6%, 5) = Rp 10 juta (5,6371)= Rp 56.371.000 3.5.4. Sinking-fund deposit Digunakan untuk menghitung besarnya pembayaran seri yang seragam pada tiap akhir tahun dari sejumlah uangpada masa mendatangpadaakhir periode tertentudantingkat suku bunga tertentu. Contoh 3-8: Jika kita ingin memiliki uang $ 1000 pada akhir tahun keenam, berapa uang yang harus kita depositokan setiap tahunnya, jika suku bunga deposito itu 10%/tahun? A A = Rp 10 jt/th A A AA 0 21 F= ? 3 4 5
  • 22. Yuli Kusumawati, Catatan Kuliah Ekonomi Mineral - 16 Jawab: A = $1000 (0,1/((1 + 0,1)6 -1)) = $ 1000 (A/F10%,6) = $ 1000 (0,12961)= $ 129,61 3.5.5. Capital-recovery Digunakan untuk menentukan nilai aliran kas yang seragam setiap akhir tahun selama periode tertentu dengan suku bunga tertentu dari nilai sekarang. Contoh 3-9: Untuk ekspansi bisnisnya seorang pengusaha meminjam uang di bank sebesar Rp 100 juta dengan suku bunga 5% per tahun untuk jangka waktu 10 tahun. Berapa angsuran yang dibayar pengusaha tersebut setiap tahunnya? Jawab: A = Rp 100 juta {(0,05(1+0,05)10) / ((1+0,05)10 – 1)} = Rp 100 juta (A/P5%,10) = Rp 100 juta (0,1295)= Rp 12.950.000/tahun 3.5.6. Uniform series present-worth Digunakan untuk menentukan nilai sekarang dari sejumlah pembayaran yang seragam setiap akhir tahun selama periode tertentu dan suku bunga tertentu. Contoh 3-10: Sebuah dealer menawarkan mobil dengan uang muka Rp 10 juta dan sisanya dapat diangsur Rp 2 juta/bulan selama 100 bulan. Bila bunga yang diberlakukan dealer tersebut adalah 1%/bulan, berapa harga mobil itu jika dibayar tunai saat ini? Jawab: Nilai sekarangdari total angsuran: P = Rp 2 juta (P/A1%,100) = Rp 2 juta (63,029) = Rp 126.038.000 Harga mobil tersebut = uangmuka +total angsuran H = Rp 10 juta + Rp 126.038.000 = Rp 136.038.000 3.5.7. Uniform gradient-series Digunakan untukmenghitungpembayaran yangnaikatauturun secarakonstan padasetiapperiode. A = B + g {(1/i)-(n/(1+i)n – 1))} A = B + g (A/G i,n) A A = ? A A AA 0 ..1 F= $1000 .. .. 6 A A = ? A A AA 0 ..1 P=Rp 100 jt .. .. 10 A A= 2jt/bln A A AA 0 ..1 P=? .. .. 100
  • 23. Yuli Kusumawati, Catatan Kuliah Ekonomi Mineral - 17 Contoh 3-11: Biaya operasi dan pemeliharaan suatu mesin produksi dari tahun ke-1 sampai tahun ke-5 berturut-turut adalah Rp 5 juta, Rp 7,5 juta, Rp 10 juta, Rp 12,5 juta, dan Rp 15 juta. Berapa biaya tersebut per tahunnya jika diketahui tingkat suku bunga adalah 20% per tahun. Jawab: A = B + g (A/G i,n) = Rp 5 juta + Rp 2,5 juta (A/G 20%,5) = Rp 5 juta + Rp 2,5 juta (1,6045) = Rp 9.011.250 Sama dengan diagram aliran kas: Tabel 3.2. Rumus aljabar dankonversi rumus bungamajemuk
  • 24. Yuli Kusumawati, Catatan Kuliah Ekonomi Mineral - 18 4. INDIKATOR-INDIKATOR KELAYAKAN INVESTASI Kelayakan suatu investasi (proyek) dari aspek ekonomi dapat dianalisis berdasarkan manfaat (benefit) dan biaya (cost). Suatu proyek dikatakan layak untuk dikerjakan jika manfaat yang diberikan lebih besar daripada biaya yang dikeluarkan. Untuk menilai keekonomian suatu proyek perlu dilihat seluruh pengeluaran dan pendapatan sepanjang umur proyek tersebut (life cycle analysis), bukan hanya dengan melihat biaya per satuan produksi di tahun tertentu. Indikator keuntungan yangdigunakandalam pengambilan keputusan seyogyanyamempunyai ciri-ciri: 1. Dapat dengan tepat dalam membandingkan dan mengelompokkan kesanggupan memberikan keuntungan (profitability) dari kesempatan-kesempatan penanamanmodal. 2. Mencerminkan nilai waktu dari modal perusahaan dan secara realistis merupakan masukan bagi kebijaksanaan fiskal dari perusahaan, termasukinvestasi kembali di masa yangakan datang. 3. Dapat menunjukkan keuntungan meskipun sekecil-kecilnya. 4. Mencakup pernyataan-pernyataankuantitatifdari resiko. 5. Menggambarkan faktor-faktor lain, seperti hasil-hasil gabungan, resiko, dan kekayaan perusahaan bila mungkin. Beberapa indikator yangdigunakan untukmenilai kelayakan proyek/investasi antara lain: a. Net present v alue (NPV) b. Rate of return (ROR) c. Present v alue ratio (PVR) atauBenefit-cost ratio (BCR) d. Paybackperiod (PBP) Pada dasarnya untukanalisis inv estasi tunggal, semua indikator tersebut akan memberikan hasil keputusan yang konsisten satu sama lain, tetapi informasi spesifik yang dihasilkan tentu akan berbeda. Sehingga dalam prakteknya beberapa indikator sering digunakan secara bersamaan untuk mendapatkan gambaran yang lebih komprehensif. 4.1. Net present value (NPV) NPV adalah perbedaan antara total present worth dari total penerimaan (revenue) atau penghematan (saving) dan present worth dari pengeluaransepanjangumur proyekpadadiscountrate yangdiberikan. NPV = ∑ PW revenue - ∑ PW cost NPV menunjukkan keuntunganproyeksecara absolut.Suatuproyekdikatakan layak(feasible) apabilaNPV>0. Cara menghitung NPV bukan trial and error, memperhitungkan nilai waktu uang, menggunakan discount rate sama dengan MARR, danbisa mempertimbangkanresiko. Contoh 4-1: Suatu proyekmemiliki aliran kas sebagai berikut: Tahun ke- 0 1 2 3 4 5 Aliran kas (x Rp 1.000.000) -100 20 30 20 40 40 Tujuan:  Mampu memahami konsep analisis kelayakan investasi tunggal.  Mampu menganalisis kelayakaninvestasi tunggal berdasarkan indikator NPV, ROR, PVR, BCR, dan PBP
  • 25. Yuli Kusumawati, Catatan Kuliah Ekonomi Mineral - 19 Analisis kelayakan proyek tersebut berdasarkan NPVjika diketahui MARR = 10%. Jawab: NPV = ∑ PW rev enue - ∑ PW cost = 20(P/F,10%,1)+30(P/F,10%,2)+20(P/F,10%,3)+40(P/F,10%,4)+40(P/F,10%,5)-100 = 20(0,9091)+30(0,8264)+20(0,7513)+40(0,6830)+40(0,6209)-100 = 10,156 Karena NPV>0, maka proyektersebut layak. 4.2. Rate of return (ROR) ROR adalah discount rate (i) yang menyebabkan NPV = 0.ROR merupakan laju perolehan per tahun yang dihasilkan oleh inv estasi suatuproyek(menunjukkankeuntungan secara relatifterhadap skala investasi proyek). Cara menghitung ROR adalah coba-coba (trial and error), memperhitungkan nilai waktu uang, lebih dipengaruhi cashflow awal, tidak tergantung nilai absolut cashflow, tidak bisa dihitung jika semua cashflow negatifatau positifatau belum balikmodal, dan bisadiperoleh nilai ganda. Suatu proyekdikatakan layakjika ROR>MARR. Contoh 4-2: Suatu proyekmemiliki aliran kas sebagai berikut: Tahun ke- 0 1 2 3 4 5 Aliran kas (x Rp 1.000.000) -100 20 30 20 40 40 Analisis kelayakan proyektersebut berdasarkan ROR jika diketahui MARR = 10%. Jawab: ROR adalahnilai i yangmenghasilkan NPV=0, diperoleh dengan cara coba-coba: Untuki = 10% diperoleh NPV = 10,156 (perhitungannya seperti contoh4.1) Untuki = 15%, nilai NPVadalah: NPV = 20(P/F,15%,1)+30(P/F,15%,2)+20(P/F,15%,3)+40(P/F,15%,4)+40(P/F,15%,5)-100 = 20(0,8696)+30(0,7561)+20(0,6575)+40(0,5718)+40(0,4972)-100 = -4,015 Selanjutnyadicari nilai RORdengan interpolasi: ROR−10% 10 ,156 −0 = 15%−10% 10,156−(−4,015) ROR = 10% + (15%-10%)((10,156/(10,156+4,02)) =13,58% Karena ROR>MARR, maka proyektersebutlayak. 4.3. Present value ratio (PVR) PVR adalah perbandingan antara NPV dengan nilai absolut investasi yang dikeluarkan (PW net cashflow negatif). 𝐏𝐕𝐑 = 𝐍𝐏𝐕 │𝐏𝐖 𝐧𝐞𝐭 𝐂𝐅 (−)│ Proyekdikatakan layakjika PVR>0. ROR=?10% 0 -4,015 10,156 15%
  • 26. Yuli Kusumawati, Catatan Kuliah Ekonomi Mineral - 20 Contoh 4-3: Suatu proyekmemiliki aliran kas sebagai berikut: Tahun ke- 0 1 2 3 4 5 Aliran kas (x Rp 1.000.000) -100 20 30 20 40 40 Analisis kelayakan proyektersebut berdasarkan PVR jika diketahui MARR = 10%. Jawab: PVR = NPV/│PW net CF (-)│ = 10,156/│-100│= 0,10 Karena PVR>0, maka proyektersebut layak. 4.4. Benefit-cost ratio (BCR) BCR adalah perbandinganantara discountedbenefit (savingataurevenue) dengan investasi. 𝐁𝐂𝐑 = 𝐏𝐖 𝐧𝐞𝐭 𝐂𝐅(+) │𝐏𝐖 𝐧𝐞𝐭 𝐂𝐅 (−)│ Proyekdikatakan layakjika BCR>1. HubunganPVR danBCR adalah: BCR = PVR + 1 atau PVR = BCR -1 Contoh 4-4: Suatu proyekmemiliki aliran kas sebagai berikut: Tahun ke- 0 1 2 3 4 5 Aliran kas (x Rp 1.000.000) -100 20 30 20 40 40 Analisis kelayakan proyektersebut berdasarkan BCR jika diketahui MARR = 10%. Jawab: BCR = PW net CF(+)/│PW net CF (-)│ = 20(P/F,10%,1)+30(P/F,10%,2)+20(P/F,10%,3)+40(P/F,10%,4)+40(P/F,10%,5) / │-100│ = 110,156/100 = 1,10 Atau BCR = PVR + 1 = 0,10 + 1 = 1,10 Karena BCR>1, maka proyektersebut layak. 4.5. Payback period (PBP) PBP adalah jangka waktu yang diperlukan untuk mengembalikan modal investasi yang ditanam (investasi mencapai titik impas). Simple PBP adalah PBP dengan discount rate sama dengan nol (undiscounted). Kelemahan PBP adalah tidak mempersoalkan keuntungan investasi.PBP dihitung dengan cara interpolasi berdasarkan aliran kas kumulatif.Diterima atau tidaknya suatu proyek pada periode pengembalian tertentu tergantung pada investor. Proyek akan diterima jika PBP lebih singkat dari periode terpendek yang diinginkan inv estor.
  • 27. Yuli Kusumawati, Catatan Kuliah Ekonomi Mineral - 21 Contoh 4-5: Suatu proyekmemiliki aliran kas sebagai berikut: Tahun ke- 0 1 2 3 4 5 Aliran kas (x Rp 1.000.000) -100 20 30 20 40 40 Analisis kelayakan proyektersebut berdasarkan undisconteddandiscounted PBP jika diketahui MARR = 10%. Jawab: a. Simple PBP Terlebih dulu menghitungaliran kas kumulatifnya: Tahun ke- 0 1 2 3 4 5 Aliran kas (x Rp 1 juta) -100 20 30 20 40 40 Aliran kas kumulatif(x Rp 1 juta) -100 -80 -50 -30 10 50 Terlihat bahwa periode pengembalian investasi proyek tersebut antara tahun ke-3 dan ke-4, sehingga PBP dapat dihitungdenganinterpolasi: 𝐏𝐁𝐏 = 𝐭𝐚𝐡𝐮𝐧 𝐂𝐮𝐦. 𝐂𝐅(−)𝐭𝐞𝐫𝐚𝐤𝐡𝐢𝐫 + │ 𝐂𝐮𝐦. 𝐂𝐅(−)𝐭𝐞𝐫𝐚𝐤𝐡𝐢𝐫 𝐂𝐅(+) 𝐭𝐡 𝐛𝐞𝐫𝐢𝐤𝐮𝐭𝐧𝐲𝐚 │ PBP = 3 + │ −30 40 │ = 3 + 0,75= 3,75 tahun. b. Discounted PBP denganMARR 10% Terlebih dulu menghitungaliran kas diskonto dankumulatifnya: DCF th-1 = 20(P/F,10%,1) = 20(0,9091) = 18,182 DCF th-2 = 30(P/F,10%,2) = 30(0,8264) = 24,792 DCF th-3 = 20(P/F,10%,3) = 20(0,7513) = 15,026 DCF th-4 = 40(P/F,10%,4) = 40(0,6830) = 27,320 DCF th-5 = 40(P/F,10%,5) = 40(0,6209) = 24,836 Tahun ke- 0 1 2 3 4 5 Discounted CF (x Rp 1 juta) -100 18,182 24,792 15,026 27,320 24,836 CummulatifDCF (x Rp 1 juta) -100 -81,818 -57,026 -42,000 -14,680 10,156 Terlihat bahwa periode pengembalian investasi proyek tersebut antara tahun ke-4 dan ke-5, sehingga PBP dapat dihitungdenganinterpolasi: 𝐏𝐁𝐏 = 𝐭𝐚𝐡𝐮𝐧 𝐂𝐮𝐦. 𝐂𝐅(−)𝐭𝐞𝐫𝐚𝐤𝐡𝐢𝐫 + │ 𝐂𝐮𝐦. 𝐂𝐅(−)𝐭𝐞𝐫𝐚𝐤𝐡𝐢𝐫 𝐂𝐅(+) 𝐭𝐡 𝐛𝐞𝐫𝐢𝐤𝐮𝐭𝐧𝐲𝐚 │ PBP = 4 + │ −14,680 24,836 │= 4 + 0,59 = 4,59 tahun.
  • 28. Yuli Kusumawati, Catatan Kuliah Ekonomi Mineral - 22 5. ANALISIS MUTUALLY EXCLUSIVE Proyek dapat didefinisikan sebagai suatu kegiatan ekonomi dengan tujuan tertentu. Untuk mencapai tujuan tersebut terdapat berbagai alternatif yang dapat dipilih. Pertimbangan untuk memilih alternatif yang terbaik adalah yang membutuhkan biaya minimum atau yang menghasilkan keuntungan maksimum atau yangwaktu pengembalianinvestasinya palingsingkat. Secara umum investasi dibedakanmenjadi dua, yaitu: a. Investasi yang menghasilkan pendapatan (revenue) atau penghematan (saving). Pertimbangan yang digunakan untukmenilai kelayakaninvestasi ini adalah keuntungan yangmaksimum. b. Investasi yang tidak menghasilkan revenue atau saving. Pertimbangan yang digunakan untuk menilai kelayakan investasi ini adalahbiayayangminimum. Analisis mutually exclusive adalah menganalisis beberapa alternatif dimana hanya ada satu alternatif yang akan dipilih, yaitu alternatif yang akan menghasilkan pelayanan (service) terbaik, atau yang akan memperbaiki, memperluas, mengembangkan operasi yang ada, atau yang akan menghasilkan pemasukan (income) terbesar. Alternatif yang dipilih adalah yang mempunyaiROR, NPV, BCR, atau PVR yang paling tinggi dibandingkan alternatif yang lain. Akan tetapi terkadang suatu alternatif mempunyai ROR yang lebih tinggi dibanding alternatif yang lain tetapi NPV nya lebih rendah. Untuk kondisi tersebut maka indikator kelayakan tersebut harus dihitungnilai inkrementalnya. 5.1. Analisis mutually exclusive alternatif dengan umur sama Jika terdapat beberapa alternatif proyek/investasi yang mempunyai umur investasi yang sama, maka prosedur analisis mutually exclusive alternatiftersebut adalahsebagai berikut: 1. Urutkan alternatifdari yangmempunyai investasi awal terendah ke tertinggi. 2. Buat cashflow inkremental (∆CF), yaitu alternatif yang investasi awalnya lebih tinggi (penantang/challenger) dikurangi alternatifyanginvestasi awalnyalebih rendah (petahana/defender). 3. Tentukan nilai NPVinkremental, ROR incremental, BCR inkremental, PVR inkremental. 4. Analisis dengan kriteria sebagai berikut: a. Jika ∆NPV> 0, maka pilih alternatifdengan investasi yanglebih besar (penantang) dansebaliknya. b. Jika ∆ROR> MARR, maka pilih alternatifdenganinvestasi yanglebih besar (penantang) dan sebaliknya. c. Jika ∆BCR> 1, maka pilih alternatifdengan investasi yanglebih besar (penantang) dan sebaliknya. d. Jika ∆PVR> 0, maka pilih alternatifdengan investasi yanglebih besar (penantang) dan sebaliknya. Contoh 5-1: Untuk memenuhi kebutuhan listrik di area penambangan, investor bermaksud membeli generator listrik. Ada tiga alternatifgenerator listrik yangditawarkanoleh penjual, denganbiaya dan manfaat sebagai berikut: Merekgenerator X Y Z Harga (x Rp 1 juta) 17 25 28 Biaya operasi dan perawatan (x Rp 1 juta) 8 6 5 Nilai manfaat/tahun (x Rp 1 juta) 13 13 13 Nilai sisa (x Rp 1 juta) 5 8 14 Tujuan:  Mampu memahami konsep pemilihanalternatifinvestasi secara mutually exclusive.  Mampu melakukananalisis mutually exclusiveberdasarkanindikator NPV, ROR,BCR, dan PVR.
  • 29. Yuli Kusumawati, Catatan Kuliah Ekonomi Mineral - 23 Ketiga generator tersebut dibeli pada tahun ini (tahun ke-0) dan akan digunakan pada tahun depan (tahun ke- 1) serta memiliki umur pemakaian yang sama yaitu 5 tahun.Jika hanya satu generator yang akan dibeli, tentukan generator manayangpalingmenguntungkan dengan asumsi MARR 14%/tahun. Jawab: Diagram cash flow masing-masingalternatifadalahsebagai berikut: Alternatif X: Alternatif Y: Alternatif Z: Kemudian urutkan alternatif dari yang mempunyai investasi terkecil sampai yang mempunyai investasi terbesar, dengan cara membuat tabel cashflow masing-masingalternatifdaninkrementalnya: Tahun ke- X Y Z Y-X Z-Y 0 -17 -25 -28 -8 -3 1 5 7 8 2 1 2 5 7 8 2 1 3 5 7 8 2 1 4 5 7 8 2 1 5 10 15 22 5 7 Selajutnya dianalisis berdasarkannilai inkrementalnya: Bandingkan generator X dengangenerator Y: ∆NPVY-X @ 14% =- 8 + 2 (P/A,14%,4) + 5 (P/F,14%,5) = - 8 + 2(2,914) + 5(0,519) = 0,423 juta Karena NPVY-X> 0, maka pilih Y (alternatifdengan investasi awal lebih besar). Lalu bandingkangenerator Y dengan generator Z: ∆NPVZ-Y @ 14% = - 3 + 1 (P/A,14%,4) + 7 (P/F,14%,5) = - 3 + 1(2,914) + 7(0,519) = 3,547 juta Karena ∆NPVZ-Y > 0, maka pilih Z (alternatifdenganinvestasi awal lebih besar). ∆RORY-X (dihitungdengancara coba-coba)= 15,9% Karena ∆RORY-X> MARR, makapilih Y (alternatifdengan investasi awal lebih besar). Lalu bandingkangenerator Y dengan generator Z: ∆RORZ-Y (dihitungdengan caracoba-coba) = 42,1% Karena ∆RORZ-Y> MARR, maka pilih Z (alternatifdenganinvestasi awal lebih besar). CX = 17 juta 1 OCX = 8 juta IX = 13 juta OCX = 8 juta IX = 13 juta LX = 5 juta ...........................0 4 OCX = 8 juta IX = 13 juta 5 CFX0 = -17 juta CFX1 = 5 juta CFX4 = 5 juta CFX5 = 10 juta Cy = 25 juta 1 OCy = 6 juta Iy = 13 juta OCy = 6 juta Iy = 13 juta Ly = 8 juta ...........................0 4 OCy = 6 juta Iy = 13 juta 5 CFY0 = -25 juta CFY1 = 7 juta CFY4 = 7 juta CFY5 = 15 juta CZ = 28 juta 1 OCZ = 5 juta IZ = 13 juta OCZ = 5 juta IZ = 13 juta LZ = 14 juta ...........................0 4 OCZ = 5 juta IZ = 13 juta 5 CFZ0 = -28 juta CFZ1 = 8 juta CFZ4 = 8 juta CFZ5 = 22 juta
  • 30. Yuli Kusumawati, Catatan Kuliah Ekonomi Mineral - 24 ∆PVRY-X @ 14% = (∆NPVY-X @ 14%)/│PW net ∆CFY-X (-)│ = 0,423 / 8 = 0,052 Karena ∆PVRY-X> 0, maka pilih Y (alternatifdengan investasi awal lebih besar). Lalu bandingkangenerator Y dengan generator Z: ∆PVRZ-Y @ 14% = (∆NPVZ-Y @ 14%)/│PW net ∆CFZ-Y (-)│ =3,547 / 3 = 1,182 Karena ∆PVRZ-Y> 0, maka pilih Z (alternatifdengan investasi lebih besar). ∆BCRY-X @ 14% = ∆PVRY-X @ 14% + 1 = 0,052 + 1 = 1,052 Karena BCRY-X>1, maka pilih Y (alternatifdenganinvestasi awal lebih besar). Lalu bandingkangenerator Y dengan generator Z: ∆BCRZ-Y @ 14% = ∆PVRZ-Y @ 14% + 1 = 1,182 + 1 = 2,182 Karena ∆BCRZ-Y>1, maka pilih Z (alternatifdengan investasi awal lebih besar). Hasil perhitungan dirangkum dalam tabel berikut ini: Tahun ke- CF X CF Y CF Z CF (Y-X) CF (Z-Y) 0 -17 -25 -28 -8 -3 1 5 7 8 2 1 2 5 7 8 2 1 3 5 7 8 2 1 4 5 7 8 2 1 5 10 15 22 5 7 NPV 2,76 3,19 6,74 0,4 3,5 ROR 19,9% 18,6% 22,1% 15,9% 42,1% PVR 0,16 0,13 0,24 0,05 1,18 BCR 1,16 1,13 1,24 1,05 2,18 Terbukti dengan menganalisis secara inkremental maka semua indikator tersebut memberikan hasil yang sama/konsisten (generator Z palinglayakuntukdipilih). 5.2. Analisis mutually exclusive alternatif dengan umur berbeda Seringkali alternatif-alternatif yangada memiliki umur yang berbeda, sehingga diperlukan basis umur yang sama untukbisa menganalisisnya. Perhatikan diagram cashflow dari dua alternatifberikut ini: Alternatif A: Alternatif B: Ada tiga metode untukmenyamakan basis umur alternatif, yaitu: a. Berdasarkan kelipatanpersekutuanterkecil dari masing-masingumur alternatif. Kelemahan metode ini adalah jika diperoleh hasil kelipatan persekutuan terkecil yang besar maka akan terlalu banyakdummy reinvestment(investasi ulang semu). Alternatif A: Alternatif B: CA 2 OCA2 0 1 OCA1 CB 2 OCB2 0 31 OCB1 OCB3 LA LB LA CA 2 OCA2 0 1 OCA1 LB CB 2 OCB2 0 31 OCB1 OCB3 LA CA 4 OCA2 2 3 OCA1 LB CB 5 OCB2 3 64 OCB1 OCB3 LA CA 6 OCA2 4 5 OCA1
  • 31. Yuli Kusumawati, Catatan Kuliah Ekonomi Mineral - 25 b. Berdasarkan umur alternatifterpanjang Alternatif dengan umur lebih pendek akan ditambahkan investasi ulang semu sampai periodenya sama dengan alternatifdenganumur terpanjang. Jika menggunakan alat yang sama maka nilai sisa pada investasi ulang semu adalah nilai buku pada periode yangterpotong. Jika menggunakan alatbaru (CA ≠ C’A) padainvestasi ulangsemu, maka nilai LA ≠ L’A. Alternatif A: Alternatif B: c. Berdasarkan umur alternatifterpendek. Pada alternatif yang dikurangi umurnya, maka nilai sisa yang baru sama dengan nilai buku yang terpotong (L’B> LB). Alternatif A: Alternatif B: Contoh 5-1: Seorang investor dihadapkan pada pilihan untuk menjual bukit karst miliknya seharga Rp 150 juta atau mengembangkan lahan tersebutdengan investasi Rp 300 juta pada tahun ke-0 dan Rp 400 juta pada tahun ke-1 akan menghasilkan keuntungan tetap dari tahun ke-2 sampai tahun ke-10 sebesar Rp 200 juta. Jika MARR yang ditetapkanadalah15%, keputusanmanayangpalingmenguntungkan secara ekonomi? Jawab: Diagram cash flow masing-masing alternatifadalah: Alternatif A (mengembangkan lahan): Alternatif B (lahan dijual): Diagram inkremental cashflow (alternatifA – alternatifB) adalah: ∆NPVA-B = - 450 - 400(P/F,15%,1) +200(P/A,15%,9)(P/F,15%,1) = - 450 – 400(0,870) + 200(4,772)(0,870) = Rp 32,3 juta Keputusanpilih alternatifA(∆NPVA-B> 0, pilih alternatifdenganinvestasi awal lebih besar) LA CA 2 OCA2 0 1 OCA1 L’A C’A 2 3 OCA3 LB CB 2 OCB2 0 31 OCB1 OCB3 LA CA 2 OCA2 0 1 OCA1 L’B CA 2 OCB2 0 1 OCB1 20 31 64 75 108 9 200-300 200-400 200200 200200 200200 200 20 31 64 75 108 9 -150 -- -- -- -- - 20 31 64 75 108 9 200-450 200-400 200200 200200 200200 200
  • 32. Yuli Kusumawati, Catatan Kuliah Ekonomi Mineral - 26 ∆RORA-Bdihitungdengancara coba-coba: ∆NPVA-B @15% = Rp 32,3 juta ∆NPVA-B @ 20% = - 450 - 400(P/F,20%,1) +200(P/A,20%,9)(P/F,20%,1) = - 450 - 400(0,833) + 200(4,031)(0,833) = Rp -111,6 juta Selanjutnyadicari nilai ∆RORA-B denganinterpolasi: (∆RORA-B-15%)/(32,3-0) = (20%-15%)/(32,3-(-111,6)) ∆RORA-B = 15% + (20%-15%)((32,3/(32,3+111,6)) =16% Karena ∆RORA-B>MARR, maka pilih alternatifA. ∆PVRA-B = ∆NPVA-B /│PW net ∆CF (-)│ = 32,3 /│(-450 + -400(P/F,15%,1)│ = 0,04 Karena ∆PVRA-B> 0, maka pilih alternatifA. ∆BCRA-B = ∆PVRA-B + 1 = 0,04 + 1 = 1,04 Karena ∆BCRA-B>1, maka pilih alternatifA. ∆RORA-B =? 15% 0 -111,6 32,3 20%
  • 33. Yuli Kusumawati, Catatan Kuliah Ekonomi Mineral - 27 6. ANALISIS NON-MUTUALLY EXCLUSIVE Analisis non-mutually exclusive adalah analisis terhadap beberapa alternatif dimana dari beberapa alternatif tersebut dapat dipilih lebih dari satu alternatif, sesuai dengan tersedianya modal atau anggaran untuk investasi tersebut. Tujuan dari analisis non-mutually exclusive adalah untuk memaksimalkan keuntungan kumulatif yang dapat dihasilkan dari investasi (cummulative net value atau cummulative future worth profit) berdasarkan kombinasi beberapa alternatif. Aplikasi analisis non-mutually exclusive antara lain pemilihan alternatif riset dan pengembangan, pemilihan metode eksplorasi, serta meranking prospekpengeboran padaindustri minyak. Ada dua teknikyangdigunakan untukmeranking proyeknon-muatually exclusive, yaitu: a. Growth rate ofreturn (growth ROR), yaitusuatutingkat suku bungamajemukdimana investasi tumbuh. b. Analisis rasio, baikmenggunakan PVR atauBCR. Individual ROR dan individual NPV tidak bisa digunakan untuk meranking proyek non-mutually exclusive, karena individual NPV maupun RORyang besar belum tentu mencerminkan alternatif terbaik.Analisis menggunakan net value analysis (NPV,NAV,NFV) pada proyek non-mutually exclusive digunakan untuk memilih kombinasi proyek yangmenghasilkan nilai bersih kumulatifterbesar. 6.1. Analisis non-mutually exclusive dengan umur alternatif sama Contoh 6-1: Seorangpengusahadihadapkanpadaempatalternatifpembelian mesin seperti berikut: Mesin A Mesin B Mesin C Biaya inv estasi (xRp1000) 10.000 25.000 35.000 Keuntungan (xRp 1000) 6.000 10.000 15.000 Umur mesin 5 tahun 5 tahun 5 tahun Jika pengusaha itu memiliki dana Rp 35 juta untuk pembelian mesin tersebut, dan dia juga mempunyai pilihan untuk berinvestasi pada proyek lain yang memberi keuntungan 20%/tahun, mesin mana yangdipilih secara non- mutually exclusive? Jawab: NPVA = -10 juta + 6 juta(P/A,20%,5) = -10 juta + 6 juta (2,991) = Rp 7,946 juta NPVB = -25 juta + 10 juta(P/A,20%,5) = -25 juta + 10 juta (2,991) = Rp 4,910 juta NPVC = -35 juta + 15 juta(P/a,20%,5) = -35 juta + 15 juta (2,991) = Rp 9,865 juta Analisis: Total harga mesin A + mesin B = Rp 35 juta, NPVA + NPVB= Rp 12,856 juta. Harga mesin C = Rp 35 juta, NPVC = Rp 9,865 juta. Kesimpulan: lebih menguntungkan membeli mesin A + mesin B. PVRA = NPVA /│PW net CFA (-)│ = 7,946 /│-10│ = 0,795 PVRB = NPVB /│PW net CFB (-)│ = 4,910 /│-25│ = 0,196 Tujuan:  Mampu memahami konsep pemilihanalternatifinvestasi non-mutually exclusive.  Mampu melakukananalisis non-mutually exclusive berdasarkan indikator NPV, ROR, BCR, danPVR.
  • 34. Yuli Kusumawati, Catatan Kuliah Ekonomi Mineral - 28 PVRC = NPVC /│PW net CFC (-)│ = 9,865 /│-35│ = 0,282 Berdasarkan PVR diperoleh rankingsebagai berikut: Mesin A (PVRA = 0,795) palingmenguntungkan, disusul mesin C (PVRC = 0,282), dan terakhir mesin B (PVRB = 0,196). BCRA = PVRA + 1 = 0,795 + 1 = 1,795 BCRB = PVRB + 1 = 0,196+ 1 = 1,196 BCRC = PVRC + 1 = 0,282 + 1 = 1,282 Berdasarkan BCR diperoleh rankingsebagai berikut: Mesin A (BCRA= 1,795) palingmenguntungkan, disusul mesin C (BCRC = 1,282), dan terakhir mesin B (BCRB = 1,196). Growth ROR mesin A: Initial Keuntungan yangdiperoleh diinvestasikanlagi ke proyek lain dengan i* = 20%: Reinv est Menghasilkan total pendapatan di akhir umur proyeksebesar: FA = 6jt(F/A,20%,5) = 6jt(7,442) = Rp 44,65 juta Initial +Reinv est Growth ROR alternatifA (GRORA)dihitungdengancoba-coba: NPVA @ 35% = - 10jt + 44,65jt (P/F,35%,5) = - 10jt + 44,65jt (0,269) = Rp 2,01 jt NPVA @ 40% = - 10jt + 44,65jt (P/F,40%,5) = - 10jt + 44,65jt (0,186) = Rp -1,69 jt Selanjutnyadicari nilai interpolasinya: (GRORA-35%)/(2,01-0) = (40%-35%)/(2,01-(-1,69)) GRORA = 35% + (40%-35%)((2,01/(2,01+1,69))=37,7% Growth ROR mesin B: Initial Keuntungan yangdiperoleh diinvestasikanlagi ke proyek lain dengan i* = 20%: Reinv est Menghasilkan total pendapatan di akhir umur proyeksebesar: FB = 10jt(F/A,20%,5) = 10jt(7,442) = Rp 74,42 juta Initial +Reinv est Growth ROR alternatifB (GRORB)dihitungdengan coba-coba: NPVB @ 20% = - 25jt + 74,42jt (P/F,20%,5) = - 25jt + 74,42jt (0,402) = Rp 4,92 jt NPVB @ 25% = - 25jt + 74,42jt (P/F,25%,5) = - 25jt + 74,42jt (0,328) = Rp -0,59 jt 3 420 1 5 6jt6jt6jt6jt6jt-10jt 3 420 1 5 6jt6jt6jt6jt6jt- FA = ? 3 420 1 5 ------10jt FA = 44,65jt 0 GROR =?35% -1,69 2,01 40% 3 420 1 5 10jt10jt10jt10jt10jt-25jt 3 420 1 5 10jt10jt10jt10jt10jt- FB = ? 3 420 1 5 ------25jt FB = 74,42jt
  • 35. Yuli Kusumawati, Catatan Kuliah Ekonomi Mineral - 29 Selanjutnyadicari nilai interpolasinya: (GRORB- 20%)/(4,92-0) = (25%-20%)/(4,92-(-0,59)) GRORB = 20% + (25%-20%)((4,92/(4,92+0,59)) = 24,46% Growth ROR mesin C: Initial Keuntungan yangdiperoleh diinvestasikanlagi ke proyek lain dengan i* = 20%: Reinv est Menghasilkan total pendapatan di akhir umur proyeksebesar: FC = 15jt(F/A,20%,5) = 15jt(7,442) = Rp 111,63 juta Initial + Reinv est Growth ROR alternatifC (GRORC)dihitungdengan coba-coba: NPVC @ 30% = - 35jt + 111,63jt (P/F,20%,5) = - 35jt + 111,63jt (0,269) = Rp -4,97 jt NPVC @ 25% = - 35jt + 111,63jt (P/F,25%,5) = - 35jt + 111,63jt (0,328) = Rp 1,61 jt Selanjutnyadicari nilai interpolasinya: (GRORC-25%)/(1,61-0) = (30%-25%)/(1,61-(-4,97)) GRORC = 25% + (30%-25%)((1,61/(1,61+4,97)) = 26,22% Berdasarkan growth RORdiperoleh rankingsebagai berikut: Mesin A (GRORA=37,7%) palingmenguntungkan, lalu mesin C (GRORC=26,22%), terakhir mesin B (GRORB=24,46%). Hasil ini konsisten dengan rankingmetode PVR dan BCR. 6.2. Analisis non-mutually exclusive dengan umur alternatif berbeda Contoh 6-2: Seorang manager memiliki anggaran sebesar Rp 50 juta untuk dialokasikan pada alternatif investasi berikut ini dengan pemilihan non-mutually exclusive. Tingkat pengembalian minimum yang ditetapkan adalah 10%. Tentukan alternatifmanayangsebaiknya dipilih? A) B) C) Jawab: NPVA = -50 jt + 20 jt(P/A,10%,2) + 50jt(P/F,10%,2) = -50 jt + 20 jt(1,736) + 50jt(0,826) = 26,033 jt NPVB = -30 jt + 10 jt(P/A,10%,5) + 30jt(P/F,10%,5) = -30 jt + 10 jt(3,791) + 30jt(0,621) = 26,535 jt NPVC = -20 jt + 5 jt(P/A,!0%,5) + 20jt(P/F,10%,7) = -20 jt + 5 jt(4,868) + 20jt(0,513) = 14,605 jt 0 GROR =?20% -0,59 4,92 25% 3 420 1 5 15jt15jt15jt15jt15jt-35jt 3 420 1 5 15jt15jt15jt15jt15jt- FC = ? 3 420 1 5 ------35jt FC = 111,63jt 0 GROR =?25% -4,97 1,61 30% L = 50jt 20jt20jt-50jt 20 1 L = 30jt 10jt10jt10jt10jt10jt-30jt 3 420 1 5 5jt5jt5jt5jt5jt-20jt 3 420 1 5 L = 20jt 5jt5jt 76
  • 36. Yuli Kusumawati, Catatan Kuliah Ekonomi Mineral - 30 Analisis: Total harga mesin B + mesin C = Rp 50 juta, NPVB + NPVC = Rp 41,140 juta. Harga mesin A = Rp 50 juta, NPVA = Rp 26,033 juta. Kesimpulan: lebih menguntungkan membeli mesin B + mesin C. PVRA = NPVA /│PW net CFA (-)│ = 26,033 /│-50│ = 0,52 PVRB = NPVB /│PW net CFB (-)│ = 26,535 /│-30│ = 0,88 PVRC = NPVC /│PW net CFC (-)│ = 14,605 /│-20│ = 0,73 Berdasarkan PVR diperoleh rankingsebagai berikut: Mesin B (PVRB = 0,88) palingmenguntungkan, disusul mesin C (PVRC = 0,73), dan terakhir mesin A (PVRA = 0,52). BCRA = PVRA + 1 = 0,52 + 1 = 1,52 BCRB = PVRB + 1 = 0,88 + 1 = 1,88 BCRC = PVRC + 1 = 0,73 + 1 = 1,73 Berdasarkan BCR diperoleh rankingsebagai berikut: Mesin B (BCRB = 1,88) palingmenguntungkan, disusul mesin C (BCRC = 1,73), dan terakhir mesin A (BCRA = 1,52). Growth ROR mesin A: Initial Keuntungan yangdiperoleh diinvestasikanlagi ke proyek lain dengan i* = 10%: Reinv est Menghasilkan total pendapatan di akhir umur proyek sebesar: FA = 20jt(F/P,10%,6) +70jt(F/P,10%,5) = 20jt(1,772) + 70jt(1,611) = Rp 148,21 juta Initial + Reinv est Growth ROR alternatifA (GRORA)dihitungdengancoba-coba: NPVA @ 15% = - 50jt + 148,21jt (P/F,15%,7) = - 50jt + 148,21jt (0,376) = Rp 5,73 jt NPVA @ 20% = - 50jt + 148,21jt (P/F,20%,7) = - 50jt + 148,21jt (0,279) = Rp -8,65 jt Selanjutnyadicari nilai interpolasinya: (GRORA-15%)/(5,73-0) = (20%-15%)/(5,73-(-8,65)) GRORA = 15% + (20%-15%)((5,73/(5,73+8,65)) = 17% Growth ROR mesin B: Initial Keuntungan yangdiperoleh diinvestasikanlagi ke proyek lain dengan i* = 10%: Reinv est 0 GROR =?15% -8,65 5,73 20% L = 50jt 20jt20jt-50jt 20 1 L = 30jt 10jt10jt10jt10jt10jt-30jt 3 420 1 5 ------50jt 3 420 1 5 F= 148,21jt -- 76 F = ? -- 76 --- 50jt 20jt20jt 3 420 1 5 6 F = ? -- 7 30jt 10jt10jt10jt10jt10jt 3 420 1 5
  • 37. Yuli Kusumawati, Catatan Kuliah Ekonomi Mineral - 31 Menghasilkan total pendapatan di akhir umur proyeksebesar: FB = [10jt(F/A,10%,5) + 30jt](F/P,10%,2) = 10jt(6,105) +30jt(0,1,21) = Rp 110,17 juta Initial + Reinv est Growth ROR alternatifB (GRORB)dihitungdengan coba-coba: NPVB @ 20% = - 30jt + 110,17jt (P/F,20%,7) = - 30jt + 110,17jt (0,279) = Rp 0,74 jt NPVB @ 25% = - 30jt + 110,17jt (P/F,25%,7) = - 30jt + 110,17jt (0,210) = Rp -6,86 jt Selanjutnyadicari nilai interpolasinya: (GRORB- 20%)/(0,74-0) = (25%-20%)/(0,74-(-6,86)) GRORB = 20% + (25%-20%)((0,74/(0,74+6,86)) = 20,4% Growth ROR mesin C: Initial Keuntungan yangdiperoleh diinvestasikanlagi ke proyek lain dengan i* = 10%: Reinv est Menghasilkan total pendapatan di akhir umur proyeksebesar: FC = 5jt(F/A,10%,7) + 20jt = 5jt(9,487) + 20jt = Rp 67,43 juta Initial + Reinv est Growth ROR alternatifC (GRORC)dihitungdengan coba-coba: NPVC @ 20% = - 20jt + 67,43jt (P/F,20%,7) = - 20jt + 67,43jt (0,279) = Rp -1,19 jt NPVC @ 15% = - 20jt + 67,43jt (P/F,15%,7) = - 20jt + 67,43jt (0,376) = Rp 5,35 jt Selanjutnyadicari nilai interpolasinya: (GRORC-15%)/(5,35-0) = (20%-15%)/(5,35-(-1,19)) GRORC = 15% + (20%-15%)((5,35/(5,35+1,19)) = 19,1% Berdasarkan growth RORdiperoleh rankingsebagai berikut: Mesin B (GRORB=20,4%) palingmenguntungkan, disusul mesin C (GRORC=19,1%), terakhir mesin A (GRORA=17%). Hasil ini konsisten dengan rankingmetode PVR dan BCR. 0 GROR =?20% -6,86 0,74 25% 0 GROR =?15% -1,19 5,35 20% 5jt5jt5jt5jt5jt-20jt 3 420 1 5 L = 20jt 5jt5jt 76 -20jt 3 420 1 5 F = 67,43jt 76 5jt5jt5jt5jt5jt 3 420 1 5 F = ? 20 jt 5jt5jt 76 ------30jt 3 420 1 5 F=110,17jt -- 76
  • 38. Yuli Kusumawati, Catatan Kuliah Ekonomi Mineral - 32 7. DEPRESIASI, DEPLESI, AMORTISASI 7.1. Depresiasi Depresiasi adalah penurunan dalam nilaifisik properti seiring dengan waktu danpenggunaannya. Dalamkonsep akuntansi, depresiasi adalahpemotongan tahunan terhadappendapatan sebelum pajak sehinggapengaruh waktu dan penggunaan atasnilai aset dapat terwakili dalam laporan keuangan suatuperusahaan. Faktor-faktor penyebabdepresiasi:  Keausan atau penurunan performa mesin/alat akibat pemakaian (physical degradation).  Perkembangan teknologi yang menghasilkan mesin/alat yang lebih efisian dan ekonomis (functional depreciation). Syarat-syarataset yangdapat didepresiasi:  Digunakan dalam usahauntukmenghasilkanpendapatan.  Mempunyai umur manfaattertentu.  Umur aset harus lebih dari satu tahun.  Nilai aset turun karena pemakaian. Komponen dalam perhitungandepresiasi:  Nilai awal, yaitu biaya awal untuk mendapatkan aset (harga beli ditambah pajak), termasuk biaya transportasi danbiayalain sampai aset tersebutdapatdigunakansesuai fungsinya.  Nilai sisa, adalah perkiraan nilai aset pada akhir umur manfaatnya, merupakan harga jual suatu aset jika tidaklagi digunakan untukproses produksi.  Nilai buku, adalah nilai aset sesuai dengan laporan akuntansi yang mewakili jumlah modal yang masih diinvestasikan pada aset tersebut. Sama dengan nilai awal (termasuk segala penyesuaian) dikurangi dengan pengurangankarena depresiasi.  Tingkat/laju depresiasi, adalah prosentase depresiasi per tahun.  Masa manfaat, adalah perkiraan periode waktu pemakaian aset (properti) dalam kegiatan produktif/menghasilkanpendapatan. Tabel 7.1. Masa manfaat dan tarifpenyusutanhartaberwujud menurut UU No. 36 Tahun 2008 Tujuan:  Memahami konsep depresiasi, deplesi, danamortisasi.  Mampu menghitungdepresiasi, deplesi, danamortisasi dengan metode yangsesuai.  Memahami pengaruhdepresi, deplesi, danamortisasi terhadappajak.
  • 39. Yuli Kusumawati, Catatan Kuliah Ekonomi Mineral - 33 Keterangan: Ayat (1) menggunakan metode depresiasi straigth line, ayat (2) menggunakan metode depresiasi double decliningbalance. Metode perhitungandepresiasiantara lain:  Metode garis lurus (sraigth line)  Metode keseimbanganmenurunganda (double decliningbalance)  Metode jumlah digit tahun (sum ofyear digit)  Metode sinkingfund  Metode unit produksi (productionunit) 7.1.1. Straigth line (SL) depreciation Metode ini mengalokasikan depresiasi aset secara konstan setiaptahunnya selama umur manfaatnya.Dengan metode ini nilai buku (BV) pada akhir umur aset sama dengan nilai sisa (S). Tingkat/laju depresiasi per tahun: d = 100% / N Besarnya depresiasi dari tahun pertama sampai tahunke–t adalah tetap: Sedangkan nilai buku(bookv alue) pada akhir tahun ke-t adalah: Dimana: Dt = depresiasi pada tahunke-t (t = 1,2,...,n) BVt = book v alue (nilai buku) aset d = tingkat/laju depresiasi tahunan P = nilai awal aset S = salv age value (nilai sisa) aset N = umur aset (dalam satuan tahun) t = akhir tahun depresiasi yangditinjau Contoh 7-1: Perusahaan ABC membeli satu unit mesin seharga $ 10,000. Diharapkanmasa pakai mesin tersebut selama 5 tahun.Pada akhir umur ekonomisnya mesin tersebut diperkirakan memiliki nilai sisa $ 1,000. Hitung nilai depresiasi dengan metode straigth line dan nilai buku pada akhir tahun ke-1 sampai tahun ke-5 dari mesin tersebut. Jawab: P = $ 10,000 S = $ 1,000 N = 5 tahun t = akhir tahun depresiasi yangditinjau Tingkat/laju depresiasi per tahun: d = 100%/5 = 20% Nilai depresiasi tiap tahun: Dt = (P-S)/N = ($ 10,000 – $ 1,000)/5= $ 1,800 Nilai bukuakhir tahunke-1 sampai ke-5: BV1 = $ 10,000 – $ 1,800 = $ 8,200
  • 40. Yuli Kusumawati, Catatan Kuliah Ekonomi Mineral - 34 BV2 = $ 8,200 – $ 1,800 = $ 6,400 BV3 = $ 6,400 – $ 1,800 = $ 4,600 BV4 = $ 4,600 – $ 1,800 = $ 2,800 BV5 = $ 2,800 – $ 1,800 = $ 1,000 Besarnya depresiasi dan nilai bukudisajikan padatabel berikut: 7.1.2. Double declining balance (DDB) depreciation Metode ini memakai tingkat/laju depresiasi duakali (double) dari metode straigthline terhadapsisa nilai buku. Tingkat/laju depresiasi tahunan adalah: d = 200%/N Besarnya depresiasi tahunke–t adalah: Sedangkan nilai buku(bookv alue) pada akhir tahun ke-t adalah: Dimana: Dt = depresiasi pada tahunke-t (t = 1,2,...,n) BVt = book v alue (nilai buku) aset d = tingkat/laju depresiasi tahunan P = nilai awal aset S = salv age value (nilai sisa) aset N = umur aset (dalam satuan tahun) t = akhir tahun depresiasi yangditinjau Contoh 7-2: Perusahaan ABC membeli satu unit mesin seharga $ 10,000. Diharapkan masa pakai mesin tersebut selama 5 tahun. Pada akhir umur ekonomisnya mesin tersebut diperkirakan memiliki nilai sisa $ 1,000. Hitung nilai depresiasi dengan metode declining balance dan nilai buku pada akhir tahun ke-1 sampai tahun ke- 5 dari mesin tersebut. Jawab: P = $ 10,000 S = $ 1,000 N = 5 tahun Tingkat/laju depresiasi per tahun: d = 200%/5 = 40% Nilai depresiasi tahunke-1 sampai ke-5:
  • 41. Yuli Kusumawati, Catatan Kuliah Ekonomi Mineral - 35 D1 = 0,4 ($ 10,000 – $ 0) = $ 4.000 D2 = 0,4 ($ 10,000 – $ 4.000) = $ 2.400 D3 = 0,4 ($ 10,000 – $ 6.400) = $ 1,440 D4 = 0,4 ($ 10,000 – $ 7.840) = $ 864 D5 = 0,4 ($ 10,000 – $ 8.704) = $ 518,4 Nilai buku akhir tahun ke-1 sampai ke-5: BV1 = $ 10,000 – $ 4.000 = $ 6.000 BV2 = $ 6.000 – $ 2.400 = $ 3.600 BV3 = $ 3.600 – $ 1.440 = $ 2.160 BV4 = $ 2.160 – $ 864 = $ 1.296 BV5 = $ 1.296 – $ 518,4 = $ 777,6 Besarnya depresiasi dan nilai bukudisajikan padatabel berikut: 7.1.3. Sum of years digits (SOYD) depreciation Metode ini mengalokasikan beban depresiasi ast berdasarkan jumlah digit tahun pemakaiannya (SOYD), sehingga alokasi biaya depresiasi tidak konstan. Dengan metode ini alokasi biaya depresiasi dibebankan lebih besar pada awal-awal periode. Jumlah digit tahun adalah: 𝐒𝐎𝐘𝐃 = 𝐧(𝐧+𝟏) 𝟐 Besarnya depresiasi tahunke–t adalah: 𝐃𝐭 = 𝐧−(𝐭−𝟏) 𝐒𝐎𝐘𝐃 (𝐏 − 𝐒) Sedangkan nilai buku(bookv alue) pada akhir tahun ke-t adalah: Dimana: SOYD = jumlah digit tahun Dt = depresiasi pada tahunke-t (t = 1,2,...,n) BVt = book v alue (nilai buku) aset P = nilai awal aset S = salv age value (nilai sisa) aset N = umur aset (dalam satuan tahun) t = akhir tahun depresiasi yangditinjau Contoh 7-3:
  • 42. Yuli Kusumawati, Catatan Kuliah Ekonomi Mineral - 36 Perusahaan ABC membeli satu unit mesin seharga $ 10,000. Diharapkan masa pakai mesin tersebut selama 5 tahun. Pada akhir umur ekonomisnya mesin tersebut diperkirakan memiliki nilai sisa $ 1,000. Hitung nilai depresiasi dengan metode sum of years digits (SOYD) dan nilai buku pada akhir tahun ke-1 sampai tahun ke-5 dari mesin tersebut. Jawab: P = $ 10,000 S = $ 1,000 N = 5 tahun Jumlah digit tahun: SOYD = n(n + 1) 2 = 5(5 + 1) 2 = 15 Nilai depresiasi tiap tahun: Dt = n − (t− 1) SOYD (P − S) D1 = 5 − (1 − 1) 15 (10.000− 1.000) = 5 15 (9.000)= 3.000 D2 = 5 − (2 − 1) 15 (10.000− 1.000) = 4 15 (9.000)= 2.400 D3 = 5 − (3 − 1) 15 (10.000− 1.000) = 3 15 (9.000)= 1.800 D4 = 5 − (4 − 1) 15 (10.000− 1.000) = 2 15 (9.000)= 1.200 D5 = 5 − (5 − 1) 15 (10.000− 1.000) = 1 15 (9.000)= 600 Nilai buku padaakhir tahunke-1 sampai ke-5: BV1 = $ 10,000 – $ 1,800 = $ 8,200 BV2 = $ 8,200 – $ 1,800 = $ 6,400 BV3 = $ 6,400 – $ 1,800 = $ 4,600 BV4 = $ 4,600 – $ 1,800 = $ 2,800 BV5 = $ 2,800 – $ 1,800 = $ 1,000 Besarnya depresiasi dan nilai buku disajikan padatabel berikut: 7.1.4. Sinking fund depreciation Metode depresiasi sinking fund membebankan depresiasi tahunan secara seragam dengan cara mempertimbangkan faktor suku bunga, sehingga nilai aset yang akan didepresiasi dikalikan dengan equal payment series sinking fund factor.
  • 43. Yuli Kusumawati, Catatan Kuliah Ekonomi Mineral - 37 Besarnya depresiasi tahunke–t adalah: Sedangkan nilai buku(bookv alue) pada akhir tahun ke-t adalah: Dimana: Dt = depresiasi pada tahunke-t (t = 1,2,...,n) BVt = book v alue (nilai buku) aset P = nilai awal aset S = salv age value (nilai sisa) aset N = umur aset (dalam satuan tahun) t = akhir tahun depresiasi yangditinjau Contoh 7-4: Perusahaan ABC membeli satu unit mesin seharga $ 10,000. Diharapkan masa pakai mesin tersebut selama 5 tahun. Pada akhir umur ekonomisnya mesin tersebut diperkirakan memiliki nilai sisa $ 1,000. Hitung nilai depresiasi dengan metode sinkingfund dan nilai buku padaakhir tahun ke-1 sampai tahun ke-5 mesin tersebut, jika i =20%. Jawab: P = $ 10,000 S = $ 1,000 N = 5 tahun i = 20% Nilai depresiasi tiap tahun: D1 = ($ 10,000 – $ 1.000)(A/F,20%,5) = $ 9.000 (0,1344) = $ 1.209,6 D2 = ($ 10,000 – $ 1.000)(A/F,20%,5) = $ 9.000 (0,1344) = $ 1.209,6 D3 = ($ 10,000 – $ 1.000)(A/F,20%,5) = $ 9.000 (0,1344) = $ 1.209,6 D4 = ($ 10,000 – $ 1.000)(A/F,20%,5) = $ 9.000 (0,1344) = $ 1.209,6 D5 = ($ 10,000 – $ 1.000)(A/F,20%,5) = $ 9.000 (0,1344) = $ 1.209,6 Nilai buku akhir tahun ke-1 sampai ke-5: BV1 = $ 10,000 – $ 1.209,6 = $ 8,790,4 BV2 = $ 6.000 – $ 1.209,6 = $ 7.580,8 BV3 = $ 3.600 – $ 1.209,6 = $ 6.371,2 BV4 = $ 2.160 – $ 1.209,6 = $ 5.161,6 BV5 = $ 1.296 – $ 1.209,6 = $ 3.952,0 Besarnya depresiasi dan nilai bukudisajikan padatabel berikut:
  • 44. Yuli Kusumawati, Catatan Kuliah Ekonomi Mineral - 38 Dt = (Produksi/Ʃ Produksi) (P-S) BVt = P(t-1) - Dt 7.1.5. Production unit depreciation Metode ini dipakai untuk peralatan yang menghasilkan layanan atau produksi berdasarkan kebutuhan per unit produksi (misalnya, truk, excavator, mesin pompaair) dan tidaktergantungpadafungsi waktu. Besarnya depresiasi tahunke–t adalah: Sedangkan nilai buku(bookv alue) pada tahunke-t adalah: Dimana: Dt = depresiasi pada tahunke-t (t = 1,2,...,n) BVt = book v alue (nilai buku) aset P = nilai awal aset S = salv age value (nilai sisa) aset t = akhir tahun depresiasi yangditinjau Contoh 7-5: Perusahaan tambang pasir PT Galunggung membeli satu unit excavator seharga Rp 700 juta. Berdasarkan spesifikasi teknis excavator tersebut mampu menambang pasir sebanyak 50.000 m3 dan nilai sisa pada akhir umur ekonomisnya diperkirakan Rp150 juta. Perencanaan produksi pasir perusahaanABC adalah sebagai berikut: Tahun ke- Produksi (m3) 1 4.000 2 6.000 3 10.000 4 10.000 5 15.000 6 5.000 Ʃ 50.000 Hitungnilai depresiasi tahunanexcavator tersebutdenganmetode unit produksi. Jawab: P = Rp 700 juta S = Rp 150 juta Nilai depresiasi tahunke-1 sampai tahun ke-6: Dt = (Produksi/Ʃproduksi)(P-S) D1 = (4.000/50.000)(Rp 700 juta – Rp 150 juta) = Rp 44 juta D2 = (6.000/50.000)(Rp 700 juta – Rp 150 juta) = Rp 66 juta D3 = (10.000/50.000)(Rp 700 juta – Rp 150 juta) = Rp 110 juta D4 = (10.000/50.000)(Rp 700 juta – Rp 150 juta) = Rp 110 juta D5 = (15.000/50.000)(Rp 700 juta – Rp 150 juta) = Rp 165 juta D6 = (5.000/50.000)(Rp 700 juta – Rp 150 juta) = Rp 55 juta Nilai buku tahun ke-1 sampai ke-6: BVt = P - Dt BV1 = Rp 700 juta – Rp 44 juta = Rp 656 juta BV2 = Rp 656 juta – Rp 66 juta = Rp 590 juta
  • 45. Yuli Kusumawati, Catatan Kuliah Ekonomi Mineral - 39 BV3 = Rp 590 juta – Rp 110 juta = Rp 480 juta BV4 = Rp 480 juta – Rp 110 juta = Rp 370 juta BV5 = Rp 370 juta – Rp 165 juta = Rp 205 juta BV6 = Rp 205 juta – Rp 55 juta = Rp 150 juta Hasil hitungan depresiasi dan nilai bukudisajikan padatabel berikut: Tahun ke- Produksi (m3) Depresiasi Nilai buku 1 4.000 Rp 44 juta Rp 656 juta 2 6.000 Rp 66 juta Rp 590 juta 3 10.000 Rp 110 juta Rp 480 juta 4 10.000 Rp 110 juta Rp 370 juta 5 15.000 Rp 165 juta Rp 205 juta 6 5.000 Rp 55 juta Rp 150 juta Ʃ 50.000 7.2. Deplesi Sumber daya alam juga termasuk aktiva tetap. Seperti halnya aktiva tetap lainnya, sumber daya alam setelah dieksploitasi kemudian diolah terus-menerus maka lama kelamaan sumber daya alam akan habis. Oleh karena itu perlu ada pembebananbiayayangmirip denganpersediaan. Deplesi adalah suatu bentuk potongan pajak atas berkurangnya sumberdaya alam (cadangan mineral atau kekayaan hutan) yang diberikan selama umur ekonomis cadangan tersebut. Perhitungan deplesi dilakukan setiap tahun. Perbedaandepresiasi dan deplesi adalahsebagai berikut:  Deplesi merupakan penyusutan terhadap pengurangan kuantitatif yang terjadi dalam sumber daya alam, sedangkan depresiasi merupakan penyusutan terhadap pengurangan service (manfaat ekonomi) yang terjadi dalam aktivatetap.  Deplesi digunakan untuk aktiva tetap yang tidak dapat diganti langsung dengan aktiva yang sama jika sudah habis, sedangkan depresiasi digunakan untuk aktiva tetap yang pada umumnya dapat diganti jika sudah habis.  Deplesi adalah pengakuan terhadap perubahan langsung dari suatu sumber alam menjadi barang yang dapat dijual, sedangkan depresiasi adalah alokasi harga perolehan ke pendapatan periode yang bersangkutan untuk suatu serv ice yang dihasilkan (kecuali dalam perusahaan di mana depresiasi dihitung berdasar hasil produksi). Dalam menghitungbesarnyadeplesi perlu memperhatikan faktor-faktor:  Harga perolehan aktiva,  Taksiran nilai sisa jika sumber alam sudah selesai di eksploitasi  Taksiran hasil yangsecara ekonomis dapat dieksploitasi. Ada dua metode perhitungan deplesi untukmineral, minyak, dangas, yaitu: a. Cost depletion b. Percentage depletion Sedangkan untukmenghitungdeplesi hasil hutanbiasanyamenggunakanmetode cost depletion.
  • 46. Yuli Kusumawati, Catatan Kuliah Ekonomi Mineral - 40 Prosedur menentukan metodedeplesi: 7.2.1. Cost depletion Basis perhitungancost depletion untukproyekminyakdan pertambanganadalah:  Hakpenguasaan.  Bonus penyewaan.  Biaya surv ey geologi/geofisika.  Biaya penaksiran danlegalitas.  Biaya-biayasejenis yangdiketahui dengan pasti. Biaya eksplorasi penambangan ataupun biaya pengeboran minyak yang tidak nyata biasanya jarang digunakan sebagai basis perhitungan cost depletion. 𝐂𝐨𝐬𝐭 𝐝𝐞𝐩𝐥𝐞𝐭𝐢𝐨𝐧 = 𝐁𝐚𝐬𝐢𝐬 𝐩𝐞𝐧𝐲𝐞𝐬𝐮𝐚𝐢𝐚𝐧 𝐱 𝐌𝐢𝐧𝐞𝐫𝐚𝐥 𝐲𝐚𝐧𝐠 𝐝𝐢𝐚𝐦𝐛𝐢𝐥 𝐝𝐚𝐧 𝐝𝐢𝐣𝐮𝐚𝐥 𝐬𝐞𝐥𝐚𝐦𝐚 𝐭𝐚𝐡𝐮𝐧 𝐛𝐞𝐫𝐣𝐚𝐥𝐚𝐧 𝐌𝐢𝐧𝐞𝐫𝐚𝐥 𝐲𝐚𝐧𝐠 𝐝𝐢𝐭𝐞𝐦𝐮𝐤𝐚𝐧 𝐩𝐚𝐝𝐚 𝐚𝐰𝐚𝐥 𝐭𝐚𝐡𝐮𝐧 Dimana: Basis penyesuaian = Basis biaya ± Penyesuaian – Deplesi kumulatif. Contoh 7-6: Suatu perusahaan mempunyai lahan pertambangan emas dengan cadangan diperkirakan 1.000.000ounces. Biaya untuk mendapatkan IUP yang dikeluarkan pada tahun ke-0 adalah $150.000. Penambanganemas diproyeksikan mulai tahun ke-1 dengan volume tetap sebesar 50.000 ounces/tahun.Hitunglah cost depletion tahun ke-1 dan tahun ke-2. Jawab: Cost depletion tahun ke-1 = ($150.000)(50.000 ounces / 1.000.000 ounces) = $7.500 Cost depletion tahun ke-2 = ($150.000-$7.500)(50.000ounces / 950.000 ounces) = $7.500 7.2.2. Percentage depletion Percentage depletion adalah persentase tertentu dari pendapatan bersih (pendapatan kotor setelah dikurangi royalti) dari penjualanmineral ataumigas yangdiambil dari lahan pertambangan selama tahunpajak. Persentase deplesi yang diijinkan biasanya tergantung pada jenis bahan tambang dan kapasitas produksi perusahaan. Contoh 7-7: Suatu perusahaan mempunyai lahan pertambangan emas dengan cadangan diperkirakan 1.000.000 ounces. Biaya untuk mendapatkan IUP yang dikeluarkan pada tahun ke-0 adalah $150.000. Penambanganemas diproyeksikan mulai tahun ke-1 dengan volume tetap sebesar 50.000 ounces/tahun dengan harga jual $29/ounce setelah royalti.Diasumsikan tingkat deplesi untuk emasadalah 15% dari pendapatan bersih.Hitunglah percentage depletion tahun ke-1 dan tahunke-2. % deplesi Batas % deplesi: 50% untukmineral 100% untukminyakdan gas Hasil yang lebih kecil merupakan percentage depletion yangdiijinkan Cost depletion Pilih metode deplesi yang menghasilkan pengurangan pajaklebih besar.
  • 47. Yuli Kusumawati, Catatan Kuliah Ekonomi Mineral - 41 Jawab: Percentage depletion tahun ke-1= 15% (50.000 ounce x $29) = $217.000 Percentage depletion tahun ke-2= 15% (50.000 ounce x $29) = $217.000 7.2.3. Memilih metode deplesi Contoh 7-8: PT Agung Linuwih mempunyai lahan emas dengan biaya hak penguasaan $150.000. Cadangan emas yang ditemukan diperkirakan satu juta ounces. Produksi diproyeksikan dimulai tahun ke-1 sebesar 50.000 ounces/tahun dan dijual dengan harga $29/ouces setelah royalti. Biaya operasi dan overhead tahun ini $180.000, dan depresiasi aset adalah $120.000/tahun. Diasumsikan tingkat produksi, biaya operasi, dan harga jual akan sama pada tahun depan. Tingkat deplesi untuk emas adalah 15% dengan batas persentase deplesi untukperusahaan tersebut adalah 100%, pajakpertambangan $30.000/tahun, danpajakpenghasilan40%. Jika perusahaan boleh memilih metode deplesi untuk mengurangi pajaknya, bandingkan besarnya percentage depletion dan cost depletiontahun ke-1 dantahun ke-2. Jawab: Tahun ke-1 Pendapatanbersih (50.000 ounces x $29) $1.450.000 - Biaya operasi -180.000 - Pajakpertambangan -30.000 - Depresiasi -120.000 Penghasilan kena pajaksebelum deplesi $1.120.000 - Batas persentase deplesi (1,0)($1.200.000) 1.120.000 - Persentage depletion(0,15)($1.450.000) -217.500*) - Cost depletion 7.500**) Penghasilan kena pajak $902.500 - Pajak40% -361.000 Penghasilan bersih (keuntungan) 541.500 + Depresiasi 120.000 + Deplesi yangdiambil 217.500 Cash flow penjualantahun ke-1 $879.000 Keterangan: *)Karena hasil percentage depletion 15% adalah $217.500 lebih kecil daripada batas 100% percetage depletion, maka diperbolehkan menggunakan metode persentase deplesi. Pada tahun ke-2 jika pendapatan dan pengurangan diasumsikan sama seperti tahun ke-1, maka batas percentage depletion juga sama, sehingga diperbolehkan menggunakanmetode persentase deplesi. **)Cost depletion tahun ke-1 = ($150.000)(50.000 ounces / 1.000.000 ounces) = $7.500, lebih kecil daripada percentage depletion sehinggaperusahaanmemilih percentage depletion untukpengurangandeplesinya. Basis biaya untuk perhitungan cost depletion tahun ke-2 harus disesuaikan untuk pengurangan aktual deplesi yangdiambil. Cost depletion tahun ke-2 = ($150.000-$217.500)(50.000/950.000) < 0. Sehingga pada tahun ke-2 metode cost depletion juga tidak digunakan untuk pengurangan deplesi. Dengan kata lain percentage depletion digunakanlagi untukmenghitungdeplesi padatahun ke-2. 7.3. Amortisasi
  • 48. Yuli Kusumawati, Catatan Kuliah Ekonomi Mineral - 42 Amortisasi adalah proses penyusutan untuk aset tidak berwujud (intangible) dan digunakan untuk pengurangan pajak. Menurut pajak, harga perolehan aset harus diamortisasi jika aset tersebut digunakan untuk mendapatkan, menagih, dan memelihara penghasilan dan memiliki masa manfaat lebih dari satu tahun. Harga aset atau biaya yang bisa diamortisasi misalnya biaya untuk pengembangan (development) tambang, intangible cost padapengeboran migas, hakpatent,franchaise, dsb. Cara mengitung amortisasi sama seperti perhitungan depresiasi garis lurus ataupun declining balance, hanya saja amortisasi diberikan dengan tingkat (rate) tertentu dan selama periode tertentu, tergantung kebijakan pajak masing-masing negara. Di Indonesia tarif amortisasi ditetapkan dalam pasal 11 A (2) UU PajakPenghasilan (PPh) No.36 Th 2008, seperti yangdisajikan padaTabel 7.1. Tabel 7.1. TarifAmortisasi berdasarkan UU No. 36 Tahun2008 Kelompokharta Masa manfaat (tahun) Tarif amortisasi Garis lurus Saldo menurun Kelompok1 4 25% 50% Kelompok2 8 12,5% 25% Kelompok3 16 6,25% 12,5% Kelompok4 20 5% 10% Jika biaya pengembangan seluruhnya (100%) digunakan untuk pengurangan pendapatan sebelum pajak, maka tidakada amortisasi. Contoh 7-9: Seorang investor akan mengembangkan suatu pertambangan. Biaya pengembangan sebesar $1 juta yang diadakan pada bulan juli (bulan ke-7) tahun ini (tahun ke-0). Jika amortisasi yang diijinkan adalah 30% dari biaya pengembangan dan periode amortisasi di atas 60 bulan, hitunglahpenguranganpajakyangdiijinkan. Jawab: Basis amortisasi = 30% x $1.000.000 = $300.000 Jangka waktu dari bulan ke-7 sampai bulan ke-12 tahun ke-0 adalah 6 bulan, sehingga amortisasi yang diambil pada tahun ke-0 adalah6/60, tahun ke-1 sampai ke-4 adalah 12/60, dantahun ke-5 adalah6/60. Tahun ke- Penguranganamortisasi 0 (6/60) ($300.000) = $30.000 1-4 (12/60) ($300.000) = $60.000 5 (6/60) ($300.000) = $30.000
  • 49. Yuli Kusumawati, Catatan Kuliah Ekonomi Mineral - 43 8. PENGARUH PAJAK DALAM ANALISIS LIFE CYCLE COSTING 8.1. Pengertian pajak Pajak(tax) adalahsalahsatu sumber keuanganpemerintah yangberasal dari rakyat. Kegunaan pajakantaralain:  Membiayai kegiatan mengelola negara (membayar gaji PNS, subsidi, pembangunan fasilitas umum, dll).  Pemerataan pendapatan (orang yang memiliki pendapatan lebih besar akan dikenakan pajak lebih banyakdibandingkanorangdenganpenghasilan lebih kecil).  Mengurangi pemakaianbarangyangsecara umum tidakbermanfaat (misalnya: miras, rokok).  Memproteksi industri nasional, industri kecil, industri ramah lingkungan. Beberapa jenis pajakyangdikenakan padaperusahaan antaralain:  Pajak pendapatan, yaitu pajak yang dipungut dari pendapatan usaha atau perorangan yang besarnya dihitungberdasarkan persentase tertentudari pendapatan bersih.  Pajakkekayaan, yaitupajakatas kepemilikan tanah, bangunan, kendaraan, mesin/peralatan, dsb.  Pajak penjualan, yaitu pajak yang dipungut atas penjualan barang atau pemberian pelayanan dan tidak ada kaitannya denganpendapatan bersih ataukeuntunganperusahaan. Dalam industri pertambangan dikenal adanya pungutan yang bisa digunakan sebagai pengurang/potongan(deduction) pendapatan sebelum pajak, antaralain:  Royalti, yaitu suatu bentuk kompensasi atas diberikannya hak istimewa untuk mengembangkan atau memproduksi mineral atau migas. Kompensasi berupa pembagian keuntungan dari nilai mineral yang diekstraksi.  Severance tax (mining tax), yaitu pajak negara atas pengambilan sumberdaya alam yang perhitungannya berdasarkan nilai/jumlahproduksi aktual.  Property tax (ad valorem tax), yaitu retribusi yang dipungut oleh otoritas perpajakan atas kekayaan baik yangberupa barangbergerakmaupuntidakbergerak. Di samping itu potongan pajak juga berasal dari penyusutan atau penurunan nilai sebuah aktiva yang memilki umur manfaat. Penyusutan adalah biaya non-kas yang berpengaruh terhadap pajak pendapatan, karena merupakan potongan tahunan terhadap pendapatan sebelum pajak sehingga pengaruh waktu dan penggunaanatas nilai aset dapatterwakili dalam laporan keuangansuatuperusahaan. Jenis-jenis penyusutandalam industri pertambanganyaitu: a. Depresiasi digunakan untuk aktivatetap. b. Amortisasi digunakan untuk aktiva tidakberwujud. c. Deplesi digunakan untukaktiva sumber dayaAlam. Dari sudut kepentingan pajak investor, metode tercepat dari pengurangan biaya adalah untuk mengeluarkan atau mengurangi pajak secara penuh pada tahun dikeluarkan. Hal ini disebabkan semakin cepat diperoleh pemotongan pajak, semakin cepat diperoleh keuntungan dari pemotongan tersebut sehingga akan memperbaiki ekonomi proyek. Akan tetapi hukum pajakseringkali tidakmengijinkan hal yangdemikian. Tujuan:  Memahami konsep pajak.  Mampu menyusun cashflow setelah pajak  Mampu memahami pengaruhpajakdalam analisis life cycle costing
  • 50. Yuli Kusumawati, Catatan Kuliah Ekonomi Mineral - 44 8.2. Cashflow setelah pajak Contoh 8-1: PT Agung Linuwih bermaksud menguasai dan mengembangkan lahan pertambangan yang diyakini mengandung 500.000 ounces emas. Biaya hak penguasaan tambang tersebut $900.000, biaya pengembangan $1.200.000, biaya pembelian peralatan tambang $1.000.000, dan biaya modal kerja $ 300.000, semuanya dikeluarkan pada awal tahun ke-0. Umur pemakaian peralatan adalah 5 tahun, dan depresiasi peralatan dihitungdengan metode straigth line dengannilai sisa padaakhir umur pemakaian peralatanadalahnol. Produksi emas diproyeksikan dimulai tahun ke-1 sebesar 100.000 ounces/tahun selama 5 tahun.Harga jual emas pada tahun ke-1 adalah $30/ouces, dan diperkirakan mengalami kenaikan 10%/tahun. Biaya operasi tahun ke-1 adalah $1.000.000, dan mengalami kenaikan 8%/tahun.Diasumsikan tingkat produksi, biaya operasi, dan harga jual akan sama padatahun depan. Royalti yang harus dibayarkan adalah 15% dari pendapatan (revenue). Pajak pertambangan $30.000/tahun, dan pajak penghasilan 30%. Perusahaan boleh memilih metode cost deplesi ataupun persentase deplesi untuk mengurangi pajaknyadengan tingkat deplesi adalah 15% dan batas persentase deplesi untuk perusahaan tersebut adalah 50%. Hitunglah: a. Cashflow sebelum pajak b. Cashflow setelah pajak jika 100% development cost digunakan untuk pengurang pajakdan deplesi dihitung dengan metode cost depletion. c. Cashflow setelah pajak jika 100% development cost digunakan untukpengurang pajakdan deplesi dihitung dengan metode percentage depletion. d. Cashlow setelah pajak jika tingkat amortisasi 30% selama 5 tahun dan deplesi dihitung dengan metode cost depletion. e. Cashlow setelah pajak jika tingkat amortisasi 30% selama 5 tahun dan deplesi dihitung dengan metode percentage depletion. Diasumsikan negatiftaxable income sebagai saving (expensed against other income). Jawab: Tahun ke- 0 1 2 3 4 5 Produksi (ounces/th) - 100.000 100.000 100.000 100.000 100.000 Harga jual ($/ounce) - 30 33 36,30 39,93 43,92 Biaya produksi ($/th) - 1.000.000 1.080.000 1.166.000 1.260.000 1.360.000 a. Cashflow sebelum pajak (x$1000): Tahun ke- 0 1 2 3 4 5 Rev enue 0 3.000 3.300 3.630 3.993 4.392 -Royalti (15%) 0 -450 -495 -545 -599 -659 Net rev enue 0 2.550 2.805 3.086 3.394 3.733 -Operatingcost 0 -1.000 -1.080 -1.166 -1.260 -1.360 Net income 0 1.550 1.725 1.919 2.134 2.373 +Working capital return 0 0 0 0 0 300 -Capital cost1) -3.400 0 0 0 0 0 CASH FLOW -3.400 1.550 1.725 1.919 2.134 2.673 Keterangan: 1)Capital cost = working capital+acquisitionrigths +depreciable equipment+development. = $300 + $900 + $1.000 + $1.200 = $3.400
  • 51. Yuli Kusumawati, Catatan Kuliah Ekonomi Mineral - 45 b. Cashflow setelah pajak jika 100% development cost digunakan untukpengurang pajakdan deplesi dihitung dengan metode cost depletion. Tahun ke- 0 1 2 3 4 5 Rev enue 0 3.000 3.300 3.630 3.993 4.392 -Royalty (15%) 0 -450 -495 -545 -599 -659 Net rev enue 0 2.550 2.805 3.086 3.394 3.733 -Operatingcost 0 -1.000 -1.080 -1.166 -1.260 -1.360 -Miningtax 0 -30 -30 -30 -30 -30 -Dev elopment cost -1.200 0 0 0 0 0 -Depreciation1) 0 -200 -200 -200 -200 -200 -Cost depletion2) 0 -180 -180 -180 -180 -180 Taxable income -1.200 1.140 1.315 1.510 1.724 1.963 -Tax (30%)3) 360 -342 -394,5 -453 -517,2 -588,9 Net income -840 798 920,5 1.057 1.206,8 1.374,1 +Depreciation 0 200 200 200 200 200 +Depletion 0 180 180 180 180 180 +Working capital return 0 0 0 0 0 300 -Capital cost4) -2.200 0 0 0 0 0 CASH FLOW -3.040 1.178 1.300,5 1.437 1.586,8 2.054,1 Keterangan: 1)Deprecition tahunke-n = (P-S)/n = (1.000.000 – 0)/5 = $200.000 (tetap selama 5 tahun) 2)Cost depletion = Basis penyesuaian x Mineral yang diambil dan dijual selama tahun berjalan Mineral yang ditemukan pada awal tahun Cost depletion th-1 = ($900.000)(100.000 ounces / 500.000 ounces) = $180.000 Cost depletion th-2 = ($900.000-$180.000)(100.000 ounces / 400.000 ounces) = $180.000 Cost depletion th-3 = ($720.000-$180.000)(100.000 ounces / 300.000 ounces) = $180.000 Cost depletion th-4 = ($540.000-$180.000)(100.000 ounces / 200.000 ounces) = $180.000 Cost depletion th-5 = ($360.000-$180.000)(100.000 ounces / 100.000 ounces) = $180.000 4)Tax (30%), jika taxable income negatifmaka dianggapsebagai saving. 5)Capital cost = working capital+acquisitionrigths +depreciable equipment. = $300.000 + $900.000 + $1.000.000 = $2.200.000 c. Cashflow setelah pajak jika 100% development cost digunakan untukpengurang pajakdan deplesi dihitung dengan metode percentage depletion. Tahun ke- 0 1 2 3 4 5 Rev enue 0 3.000 3.300 3.630 3.993 4.392 -Royalty (15%) 0 -450 -495 -545 -599 -659 Net rev enue 0 2.550 2.805 3.086 3.394 3.733 -Operatingcost 0 -1.000 -1.080 -1.166 -1.260 -1.360 -Miningtax 0 -30 -30 -30 -30 -30 -Dev elopment cost -1.200 0 0 0 0 0 -Depreciation1) 0 -200 -200 -200 -200 -200 -Percentage depletion2) 0 -382 -421 -463 -509 -560 Taxable income -1.200 938 1.074 1.227 1.395 1.583 -Tax (30%)3) 360 -281,4 -322,2 -368,1 -418,5 -474,9 Net income -840 656,6 751,8 858,9 976,5 1.108,1 +Depreciation 0 200 200 200 200 200 +Depletion 0 382 421 463 509 560 +Working capital return 0 0 0 0 0 300 -Capital cost4) -2.200 0 0 0 0 0 CASH FLOW -3.040 1.238,6 1.372,8 1.521,9 1.685,5 2.168,1
  • 52. Yuli Kusumawati, Catatan Kuliah Ekonomi Mineral - 46 Keterangan: 1)Deprecition tahunke-n = (P-S)/n = (1.000.000 – 0)/5 = $200.000 (tetap selama 5 tahun) 2)Percentage depletion diperbolehkan jika nilainyalebih kecil dari batas deplesi maksimal: Batas deplesi maksimal = 50% x taxable income before depletion Batas deplesi th-1 = 50%(2.550.000 - 1.000.000 – 30.000 – 0 – 200.000) = $660 Batas deplesi th-2 = 50%(2.805.000 - 1.080.000 – 30.000 – 0 – 200.000) = $747,5 Batas deplesi th-3 = 50%(3.086.000 - 1.166.000 – 30.000 – 0 – 200.000) = $845 Batas deplesi th-4 = 50%(3.394.000 - 1.260.000 – 30.000 – 0 – 200.000) = $952 Batas deplesi th-5 = 50%(3.733.000 - 1.360.000 – 30.000 – 0 – 200.000) = $1.071,5 Percentage depletion = 15% x net revenue Percentage depletion th-1 = 15% x 2.550.000 = $382.000 Percentage depletion th-2 = 15% x 2.805.000 = $421.000 Percentage depletion th-3 = 15% x 3.086.000 = $463.000 Percentage depletion th-4 = 15% x 3.394.000 = $509.000 Percentage depletion th-5 = 15% x 3.733.000 = $560.000 3)Tax (30%), jika taxable income negatifmaka dianggapsebagai saving. 4)Capital cost = working capital+acquisitionrigths +depreciable equipment. = $300.000 + $900.000 + $1.000.000 = $2.200.000 d. Cashflow setelah pajak jika amortisasi 30% selama 5 tahun dan deplesi dihitung dengan metode cost depletion. Tahun ke- 0 1 2 3 4 5 Rev enue 0 3.000 3.300 3.630 3.993 4.392 -Royalty (15%) 0 -450 -495 -545 -599 -659 Net rev enue 0 2.550 2.805 3.086 3.394 3.733 -Operating cost 0 -1.000 -1.080 -1.166 -1.260 -1.360 -Miningtax 0 -30 -30 -30 -30 -30 -Dev elopment cost1) -840 0 0 0 0 0 -Amortization2) -72 -72 -72 -72 -72 0 -Depreciation3) 0 -200 -200 -200 -200 -200 -Cost depletion4) 0 -180 -180 -180 -180 -180 Taxable income -912 1.068 1.243 1.438 1.652 1.963 -Tax (30%)5) 273,6 -320,4 -372,9 -431,4 -495,6 -588,9 Net income -638,4 747,6 870,1 1.006,6 1.156,4 1.374,1 +Amortization 72 72 72 72 72 0 +Depreciation 0 200 200 200 200 200 +Depletion 0 180 180 180 180 180 +Working capital return 0 0 0 0 0 300 -Capital cost6) -2.560 0 0 0 0 0 CASH FLOW -3.126,4 1.199,6 1.322,1 1.458,6 1.608,4 2.054,1 Keterangan: 1)Dev elopmentcost = 70% x $1.200 = $840 2)Amortisasi tahun ke-n = 30% (dev . cost)/periode amortisasi = 30% (1.200.000)/5 = $72.000 (tetap sampai tahunke-5) 3)Deprecition tahunke-n = (P-S)/n = (1.000.000 – 0)/5 = $200.000 (tetap selama 5 tahun)